You are on page 1of 87































































































‬ ‫ﺗﻢ ﺑﻔﻀﻞ ﷲ ﺗﺠﻤﯿﻊ اﻻﺳﺎﻟﺔ واﻻﺟﻮﺑﺔ اﻟﻤﺘﺸﺎﺑﮭﺔ ﻟﺘﺴﮭﯿﻞ ﺣﻔﻈﮭﺎ‬



‬ ‫ﻣﻠﻒ ﺷﺎﻣﻞ ﻟﻠﺒﯿﺪﯾﺎ ‪ ،‬ﻣﺼﺪر اﻻﺳﺎﻟﺔ ﺟﻠﻮري ﺗﯿﻢ ﻓﻘﻂ‪.‬‬












Antidiarrhealagentshouldbe
avoided ifthereis fEVER BlooDinstool

AbxGererallynotindicatedlexceptforcidifficilelevancomycinefmetronianazole

1Supportive management

Diarrhe

SxoffoodpoisoningiNausea vomiting

Abdominal cramps

Tx supportive

Gastroenteritis vomitingwatery

SxotRotavirus diarrhea Highgradefeve

Tx supportive

!Child patient after eat steak has fever and abdomen pain what is the management?

A-Conservative ✅ B _antibiotic
Rotavirus


my
# 3years old presented with watery diarrhea,cramps , dehydration after being exposed to colleague

with same presentation at day care , mx: === Supportive treatment

‫ اﻟواﺿﺢ اﻧﮫ‬، ‫ﯾﻌﻧﻲ طﻔل ﺗﻌرض ﻟﻠﻌدوى ﻻن زﻣﯾﻠﮫ ﻛﺎن ﻋﻧده اﺳﮭﺎل ﻓﺎﻻﺧﺗﯾﺎرات ﯾﻘوﻟك ﻛل اﻧواع اﻟﻣﺿﺎدات اﻟﺣﯾوﯾﺔ و ﻻ ﺑس ﻋﻼج دﻋم‬

supportive care ‫ﻓﺎﯾرس ﻓﺧﻼص ﺑس‬

# 4-year-old child presented with several episodes of passage of frequent non-bloody, watery stool and

vomiting. Management?

A. Metronidazole B. Doxycycline C. Amoxicillin. D. Supportive management✅✅

# Child with Hx of eating in a restaurant with abd pain, nausea and vomiting, 10 days later bloody

diarrhea, urinalysis shows 10 RBCs , ttt? Mr HUS

A. Anti diarrhea medication. B. Metronidazole. C. Supportive therapy D. Antibiotics

…..

# Child develops petechia in the legs, he has history of URTI. Labs showed: normal CBC except platelet

count was : low. What is the next step in management?


ITP Just Rash and Thrombocytopenia >>ITP

A. Platelettransfusion B. Immunoglobulin ( if sever bleeding like untracranial hemhhrage

C. Steroid ( if mild bleedingbe


intoleranttosteroid

) D. Supportive ( if plattlet more than 30.000 and there is no bleeding ) ✅✅

….. Ptt230,000

!Child develops petechia in the legs, he has history of URTI. Labs showed: normal CBC except platelet

count was : low. What is the next step = Supportive ! supportive


730 ooo steroidor

….. hrdependsonPLTLv S oooo VIG

!Child with watery diarrhea , history of sick contact = Supportive management !

..

!5 years old boy has RUQ pain + normal bowel sounds ,lap show severe drop in hemoglobin and

increased reticulocytes count. Blood smear report: target cells and inclusion bodies. diagnosis?

Sickle cell disease ( Not appendicitis)

… testhydration Analgesia
Child with a long history of watery diarrhea abdominal bloating and pain, what’s the Dx?
Bodies
mansion

A. chronic giardiasis ✅ Can Giardia last for years? (chronic) giardiasis, which causes persistent or

repeated bouts of diarrhoea that can last for up to two years.

…. Aspirin

!treatment of dengue fever? Supportive and avoid red NSAIDs!

….

!dengue mosquito time of activity?! early morning ✅

!2-3years old presented with watery diarrhea,cramps , dehydration after being exposed to colleague

with same presentation at day care , mx:

A-Ciproflaxacin B-Metronidazole. C-Supportive treatment✅. D-Fourth antibiotic

Child with watery diarrhea bloodless management: is Supportive ‫ دم‬bd c ‫ ﻣﺎدام ﻣﺎ‬hd i ‫ ﻣﻨﻄ‬.

.....

!Rota virus: = reassure !

......

Neonate with vaginal mucoid discharge and concerned mother what to tell her?

A-Reassure ✅ B-Take vaginal smear C-Canʼt recall other choices

Feverish child T 38, cough, Bilateral infiltrated lung. Nothing else mentioned. Management?

A- Reassure cuz viral ✅. B- Discharge and oral ABx C- Admission and IV ABx

……almost RSV >> Reassure

$ 8-month-old with asymmetrical breast enlargement and no other symptoms what is the cause =

reassure $
Hoo E II How to at
$A child with birth weight 3.5kg now weighing 3.1. Breastfeed about 3 times every 15 minutes. Advise to

mum? A. Improve matemal nutrition B. Reassure that all is well✅ C. Admit for full work up

….

!Two weeks year B old child suddenly had erythematous rash on his body (the child was not febrile and he

was stable) what to do = Assure mom. ons Erythema toxicum neonatorum is a common rash in neonates. It appears in up to half

Erythema um Toxic
of newborns carried to term, usually between day 2–5 after birth; it does not occur

… outside the neonatal period. Erythema toxicum is characterized by blotchy red spots

r
Innocent heart murmur (functional or physiological)
on the skin with overlying white or yellow papules or pustules.
• Most commonly mid-systolic or continuous murmur

!Mucus vaginal discharge in newborn baby: reassure the mother • Innocent Heart
Due to increased blood mnormal
flow across urmer aortic and/or

… •t
pulmonary valves (ejection murmur)
position dependency
Position-dependent;
Early
murmur varies in intensity
ormidsys
2 or disappears

! Child presented to pre-diagnosis clinic with systolic ejection murmur , no sx ? • murmer


Asymptomatic
3 Asymptomatic

A. refer to cardio for cath B. reassure and discharge C.start ACE and

It

....

! Child with 15 months, can only babble and his mother is afraid because he cannot say 2-3 word

sentences, hearing test done when he was 5 months old and was normal: d

A. reassure as this is a normal variant B. re-evaluate at 24 months C. Refer to ENT

....

Child presents with fever, vomiting and diarrhea on exam of chest there is reduced air entry to right side

and murmur was heard , the child CVS and chest exams were previously normal (prior visits) what

is the management of murmur?

A. Urgent echo B. Reexamine after these symptoms subsides ✅✅✅ C. Refer to cardio pediatrician

....

!3 year are going for dental operation had murmur when he stand and disappear when sitting What to

do? A. Reassuring ✅ B. Reevaluate C. Cardio consultation

....

! Asymptomatic child scheduled for a dental procedure, on examination found to

have a murmur that changes with position, what to do?

A. consult cardio

B. cancel procedure and for further testing

C. innocent murmur✅‫ ﻫﺬا اﺳﻢ اﻟﻤﺎرﻣﺮ وﺻﺢ‬+ no need to worry and cont✅‫ﺬا اﻟﻤﺎﻧﺞ‬P‫و‬

! Child 3 years old, fell from bed, immediately cried afterwards, vomited twice, headache, physical

exam and neuro exam are normal: A- Brain MRI B- Brain CT D- Observation

If normal Neuro exam and pt vomitted less

than 3 times observe


if the patient vomitted 3 or more CT

%Child was brought to hospital with airway swelling and skin lesions all over the body the mother stated

that he was in a party at his friends house: Food allergy%

..

&If language delay > do hearing test.&

# what to be screen in neonates = metabolic diseases, HYPOTHYROIDISM, HEARING.

#Clear case of congenital prolonged QT syndrome Jervell and Larged-Neilson

Syndrome associated with : sensorineural hearing loss 1 congenitaldeafness


2 Ventricular1achy
arrhythmias

!2 years old child can't talk probably and doesn't understand, he have decreased hearing, what 1st test

you want to do Hearing loss examination. ✅

..

Baby abdominal distended what is the first investigation = a. Xray. b. Ct. c. Us✅✅

….

Newborn what to examine?m. A- Hearing✅✅. B- Vision. C- Both ‫ اذﻧﻪ ﻗﺮان‬ä‫ ﻧﻘﺮء ﻋ‬Ñd Ö ‫ اول ﻣﺎ ﻳﻮﻟﺪ اﻟﺒﻴ‬:)

….

A child has kernicterus sign what you have expect that you tell his parents ?

ARetina detachment B.Blindness C.Hearing loos ✅

‫ﻂ‬è‫ = ر‬kids = ker = ‫ﺎء‬ùú‫ء ﻳﺰﻋﺠﻨﺎ ﻓﻴﻬﻢ ﻫﻮ اﻟﺼﻮت واﻟ‬î ï ì


d íë‫ ا‬hearing .

..............

….

!15 month old , parents concerned about his development that only babbles and cant say 2-3 words yet,

he has normal auditory examination, what should you do? Reassurance and come at 24 months

… • Re-evaluate at 24 months

!3 years old boy came with his mother, she's concerned he might have abnomial development. Ho goes

to day care, understand only 2 words command (order) and strangers can understand onlyNormal 75% of his

talk. Your action? A. Reassure B. Refer ta ENT C. Delayed speech disorder ✅

… GdRevalute at 48months

!Newborn examinations = vision and hearing (vision for red eye reflex )

surfactanttreatment
z Esf

!Aspiration meconium treatment = Surfctant ! Tx surfactantlavage

….

ventilatorsupport

Sever Nitricoxide
$mom comes to your clinic with her neonate who is 1 week of age, she is worried because her neonate

lost 1.5 kg from his birth weight? reassure

!Infant with erythematous macules on erythematous base in the back and trunk?

Reassure

!case of watery diarrhea ( what is the electricity abnormalities will find ) ?

Answer is : HYPONATREMIA, HYPOKALEMIA, METABOLIC ACIDOSIS

‫ة |ﺎﻻﺳﻬﺎل‬sz‫ﺖ ﻛﺜ‬v‫وﻻ‬str‫ﺪ =ﻓﻘﺪ اﻟ‬n‫ﻂ =اﺳﻬﺎل =اﺳ‬i‫ر‬

.....

spitting
!Primimother worried about her 6week baby , he stepping all the formula since birth , what most likely:

A. HSD B. pyloric stenosis C. physiological reflex

...

! And there was another Q near to the previous, ask about the next step :

A. burping the baby and semi setting while feeding

B. immediate call surgeon

! 9 year boy with growth pains, management? -Reassurance paroxysmal episodes ofexcessive

andunexplainedcrying inhealthy
1Kneetochestposition
infantile colic
r

andwellnourished infant
2 fistclenching3 stiffening
4...Abdominaldistension 5leg

ArchingdBack 6facial

flushing

!Pt with infantile colic scenario, mother mentioned (normal feeding) didn’t say if it was breastfeeding or

formula, treatment: The options were similar to something like this:

A. Change his milk formula because he might be allergic B. Colic drops C. Reassure mother that it’s normal

start on 6 weeks (N.B. Infantile colic onset is 6 weeks of age and goes away by 6 months of age)

.‫ ﺧﻣﺳﺔ = زودوا واﺣد وﯾﻛون ﺳت اﺳﺎﺑﯾﻊ‬colic = ‫رﺑط = ﻋدد ﺣروف‬

.....

! Infantile colic mc occur:

A. First 6 weeks B. First 3 weeks C. First 3 months D. First 6 months

! Most common time for presentation of infantile colic ?

A. 6 months B. 2 years C. 6 weeks D. 12 months

# Case describe infantile colic ... what is the usual duration ?1st ٦ month

….

! Scenario of infantile colic without mentioning the dx. The age of this condition usually? First6 month

..

! consul mother regarding infantile colic?

A. 80% of children have infantile colic B. will resolve at 6 month C. gharib water have evidence in treat

infantile colic

…..

A child with excessive crying and can't stop crying. There is history of *bottle feeding*. On examination,

the child is restless, and abdomen distended. No other significant finding on examination. Dx?

A. Infantile Colic✅ B. Infant constipation C. Cow milk intolerance

….

$2 months baby the mother complained about that he cried excessively since 2 weeks they mentioned

that the mother indroduce 1 bottle feeding at night recently. his growth on 50% What is the cause :

A. Infantile colic✅ B. Cow’s milk allergy

!What’s true about infantile colic = w large passage of flatus í™‫§ﻜﻮن ﻋﻨﺪﻫﻢ ﻏﺎزات ﻛﺜ‬

….

!Typical symptoms of infantile colic, all normal, what is most appropriate management?

A. Colic drops for colic B. Changing formula C. Behavioral adaptation

….

!6 week baby he spilt out whatever he drink he is now on cow milk his weight in birthday : 2,7 kg and

now he is 5.3 kg : A. observation B. precautions of reflux and reassure

.... Had

Baby 4 months, distrnded abdomen, stool yellow and it becomes the color more ‫ﻌﺪ ﻳﻮم‬- ‫ﻟﻮﻧﻪ اﻓﺘﺢ ﻳﻮم‬

,after birth he passed stool. Dx? A- allergy to formula given(formula intolerance )✅ B- Volvulus C-

Diverticulum inflammatory

….

!Infant on breastfeeding after that take bottle feeding and complaining from constipation and

.distension dx? A-Cow milk intolerance B-Infant colick Answer is A

….

cowsmilk leadtoAnemia tiron

!Child 12 month ago on breastfeeding for first 9 months then use bottle feeding for another 3 months.

Came with symptoms and signs of anemia with splenomegaly severeirondef


+ hypochromic microcytic anemia with retIculocytes number normal *RBC* count Peripheral blood

2
smear *Target cell* What should be restored = Iron

… can seen be
!Mother C/O child spitting after each feed cow's milk, Mx?

A- Elevate head during feeding✅. B- U/S C-Reassure

‫ﺼﻖ‬ú‫ق او ﻳ‬²ï± ‫ﺔ ﻟﻮ ﻣﺎرﻓﻌﻨﺎ رأس اﻟﻄﻔﻞ راح‬¹‫ﻌ‬¹‫ اﻟﺮﺿﺎﻋﺔ اﻟﻄﺒ‬Ñi‫ ﺣ‬، ‫ ﻣﺎﻟﻪ ﻋﻼﻗﻪ‬، ‫ﻘﺮ‬ú‫ﺐ اﻟ‬¹‫ﻄﻚ وﻛﺘﺐ ﺣﻠ‬è¬‫ اﻟﺴﺆوال ﺣﺎول §ﺨ‬، ‫ﺐ‬¹‫ﻌﺪ »ﻞ رﺿﻌﺔ §ﻄﻠﻊ اﻟﺤﻠ‬- ‫ﻃﻔﻞ‬

‫ﺐ ﻓﺎاﻟﺤﻞ ﻧﺮﻓﻊ رأﺳﻪ اﺛﻨﺎء اﻟﺮﺿﺎﻋﺔ‬¹‫ﺣﻠ‬

….

! baby with “spitted up” after feeding his weight at birth 3.4 and now 5.5 how should you council the

mother ? let the mother time spent purpine and elevate the head of baby after feeding

‫ﻌﺪ اﻟﺮاﺿﺎﻋﺔ‬- ‫ اﻻم ﺗﺮﻓﻊ راﺻﻪ‬äd‫ﺸﻨﺞ =ﺧ‬± ‫ﺴﺘﻔ¬غ او‬± ‫ﻌﺪ »ﻞ رﺿﻌﺔ اﻟﻄﻔﻞ‬- Ñd c ‫§ﻌ‬

!1 month old baby with sob, low grade fever, tachypnea,prolonged expiratory phase and in cxr there is

bilateral infiltration symptoms is the most common to be presented in this condition ?

1. Cough 2.Poor feeding ✅ 3.Peripheral cyanosis 4.Nasal flaring


‫ﻼء‬- ‫ﻪ‬¹‫ )اﻋﺮف ﻟﻮ ام ﻗﺎﻟﺖ ﻟﻚ ﻫﺬە اﻟ”ﻠﻤﺔ ﻣﻌﻨﺎﻫﺎ اﻟﺮﺿﻴﻊ ﻓ‬. ‫ﻪ ﻫﻮ (ﺗﻘﻮﻟﻚ اﻻم رﺿﺎﻋﺘﻪ ﻗﻠﺖ ﻣﺎﺳﺎر ﻳﺮﺿﻊ‬¹‫ء ﻳﺘﺎﺛﺮ ﻓ‬îï

d ‫ﻋﻤﻮﻣﺎ اي رﺿﻴﻊ اذا ﺗﻌﺐ اول اول‬

… duetoviral

%Child shifted to cow’s milk, pale with wt loss? Iron def.%


,,,

!Child with cow's milk compline of palor = iron deficiency anemia

!Child with drinking 3 large glasses of milk, he is bicky in food choices, what type of anemia he has? Iron

deficiency

!vitamin black color= Iron! ferroussulfate


Blackstool

..

child took tablets, came with black vomiting, most likely material ingested?

A. iron ✅ B. hypervitaminosis C. Wrong choice

$Child who is lethargic and losing his concentration, Hgb is 10.5, what to give?

A. IM iron✅ ‫ﺣﻞ ﺟﻠﻮري‬. B. Oral ferrous sulphate. C. Forti cereal. D. Vit,b12

… '''''

! Neonate in first routen visit had low hemoglobin 105 and give ora ferrous sulfate, in this visit also Iow

hg 10.3, what next investigation to reach dx?

A. level of serum iron and serum ferritin B. bone marrow C. give it oral ferrous sulfate.

….

!Breast milk =: rich in protein

....

!Another breast milk q content: A- lgA ✅ B-lg E C-lgG

....

!Toddler with pigmentations in his teeth, they describe the location of the pigments blah blah,

diagnosis:?Sleep with milk bottle M Riskof toothdecay

!colostrum high in?! A. Protein B.fat

›c ™ ‫ﻢ ﺑﺮوﺗ‬¹‫ =ﻧﻌﻤﻞ رﺟ‬¤‫ول ﻋﺎ‬íi‫ام =ﻛﻮﻟﻮﺳ‬íi‫ﻂ =ﻛﻮﻟﻮﺳ‬è‫ر‬


d

..

$9 days newbon come with jaundice only in the face not extended to the rest of

the body.. otherwise he is healthy was delivered by NVD with no completing. and

he was breastfeed immediately. what is the cause of his jaundice?

A. Breast milk jaundice✅✅

N.B: Breastfeeding jaundice appear in the first week. Breast milk jaundice appear after the first week and

peaks around 2 weeks.

…..

!Baby with tooth discoloration, he sleep with bottle at night What the cause

A. Syphilis B. Bottle dental caries

...

!2 months old baby wakes at night and cries for 1-2 hrs was happening several times he’s fine at what

age this usually occurs? A. 6 weeks B. 2 months C. 3 months D. 6 months

....... TV
$8 yr boy wet his bed at night, the cause is? A. detrusor muscle$ B. Pelvic muscles

‫ﻂ‬è‫ = ر‬Wet ‫ ﻣﻊ‬Det

% Night bed wetting considered normal till ?

Ñ
‫ اي ﻋﻤﺮ §ﻜﻮن‬¤‫ ا‬bed wetting ‫ﻪ‬n‫ﺤﺘﺎج ﻓﺤﻮﺻﺎت ﻣﺮﺿ‬v ‫ﻨﺎ‬P ‫ﻞ‬n‫ اﻟﻠ‬ÉÖ ‫ﻠﻞ ﻧﻔﺴﻪ‬ô‫ﺎ ﻟﻮ ﻣﺎزال ﻳ‬P‫ ﺳﻨﻮات |ﻌﺪ‬٥ ‫ ﻋﻤﺮ‬ê‫ ؟ ا‬، ç
Ñ
Ö n‫ﻞ ﻋﻨﺪ اﻻﻃﻔﺎل ﻃﺒ‬n‫ اﻟﻠ‬ÉÖ

A. 5✅

… to role out bacterial infection (E-Coli)

!most important study to do in cases of enuresis in child: a.Urine analysis✅ b. urine culture c. VCUG

.....

!Most worrying sign of Child abdominal pain ? Late night pain ( not Pain for more than 10 min or Early
This is mostly congenital adrenal hyperplasia but there is dehydration first ABC

morning pain)
should be Hydration iv fluid.

… we will chose steroid if there is no dehydration


!A baby girl complains of dehydration and clitorymegaly ,signs of dehydation. next step :Steriod

… 17Hydroxyprogesterone
CAH

! Old male present with acute confusional state and postural hypotension .. investigation just

hypercalcemia what is the next ?? Hydrocortisone


… fluid
treatment of hypercalcemia IV fluid and Diuretic
please check for all answers before choosing

Rheumatic fever

Diagnosis of RF based on Jones criteria

1heart

diseas Rheumaticfeverdue to
fi
next step not Echo if there is ASO chose it
Echo its additional finding (may show mitral or aortic regurgitation)
streptococcal pharyngitis
!A young boy complains of arthritis , rashes

, nodule subcutaneous.He had pharyngitis two weeks back =
• ↑ Antistreptolysin O titer (ASO)

next step Echo ( Dont choic steroid ) turn • ↑ Antistreptococcal DNAse B titer (ADB) e

…. valvulardiseases Erythema • Positive rapid streptococcal antigen test for GAS

!Boy collapsed during sport On Ex Jerky carotid pulse. what’s Dx?HOG'M Mostcommoncause suddendea of

A. HCOM B. wolff Parkinson’s white syndrome

... inyoungAthletes

!Pt came with HCOM Waht tx =Metoprolol • BB to decrease the heart rate the heart can fill the champers

!child with radiofemoral delay pulse, Coartication of Aorta

$child/baby has no distal pulse what you need to check before reduction? pulses in other side

.......

!Baby will be prepared for open heart surgery, mother is very worry about him what is the best way to

deal with her worry? explain what will happen before and after the surgery ✅!

......

Post MI

!What is Drug decreases the mortality in HF = ACEI/ARB + beta block ! p • ACEI that end with (pril)
ARB like sartan

!Management of pericarditis = NSAID like Ibuprofen !


no

!Sharp pain relieved by leaning forward, pericardial friction rub ==pericarditis!

..

PT pos-inferior MI, few hours developed hypotension, raised JVP, clears lungs on auscultation ? Right

ventricular infarction

right ventr ‫ ﻓورااااا ﻓﻛروا ﻓﻲ‬hypotenstion ‫ و اﻟﻛﻠﻣﺔ اﻟﻣﮭﻣﻣﻣﺔ ﺟدااااا‬posterior infarctio ‫( ﻻن داﯾم ﻟﻣﺎ ﻧﺷوف ﻛﻠﻣﺗﯾن‬

infarcrion )

!ECG show 2:1 heart block

A. 1st degree B. 2nd degree C. 3rd degree D. 4th degree

heart block2:1 ‫ ﯾﻌﻧﻲ‬Absent ‫ﺑس ھﻲ‬QRS . ‫اﻟدواﺋر اﻟﺣﻣراء ھذه ﻣﻔروض ﯾﻛون ﻓﯾﮭﺎ‬

....

First degree = ‫ §ﻜﻮن ال‬PR pronlong just.

Scond degree 1 = ‫ §ﻜﻮن‬PR Irregular

Scond degree 2 = ‫ §ﻜﻮن‬rythm ‫ ﻣﻨﺘﻈﻢ‬í™‫ﻏ‬

2:1 heart block=

= Absent‫ ﻏﯾر ﻣوﺟود‬QRS ‫ﯾﻛون ال‬

....
!Heart failure due to left ventricular hypertrophy

A. Systolic dysfunction B.Diastolic dysfunction ✅!

..

! Pediatric patient with sob and productive cough with white sputum for one week that became yellow 3

days ago, on examination there's bilateral crackles increased in the right middle lobe. On x ray report

there's bilateral pleural effusion with

consolidation in right middle lobe. What is the most appropriate cause:? exacebated infection

A. Excabitation of heart failure✅ • if there no cardiac hint in Q chose pneumonia if its in the choices

!2 months old baby with congestive heart failure and the mother asking about

nutrition requirement

A. Less than requirement for healthy baby

B. Greater than requirement for healthy baby

C. Same as healthy baby

D. Same as baby of 2 years old

… Diastolic

!Congestive heart failure due to systolic left ventricular hypertrophy = Beta block +diurtic!

!Baby with congenital heart disease present with symptoms of heart failure (pulmonary edema, crackles,

etc ) Heart rate a250-300 what is the Dx? í™‫ﺐ او ﻓﻼﺗ‬¹‫ﻂ اﻟﻘﻠﺐ ﻧﺤﻄﻂ اذا ﻓ‬¹‫ﻞ ﺗﺨﻄ‬ù‫ ﺣﺴﺐ ﺷ‬ä‫ﻋ‬

Atrialflutter

Atrialfib Go 180

A.Atrial flutter. B.Atrial fibrillation. ✅✅. C.Ventricular fibrillation. D.SVT.

How does AFib lead to heart failure? ... Blood can “back up" in the pulmonary veins (the vessels that

return oxygen-rich blood from the lungs to the heart.) which can cause fluid to back up into the lungs.

When AFib causes heart failure,

........ Failure thrive


$14M infant with had recurrent syncopal attacks worsened the HF and i think FTT on examination she
to

has diffuse crepitations on ECG she has cardiac arrhythmia narrow complex QRS and Hr 250-300 Bpm

what is the most probable cause: supraventriculrtachycardia

A. Atrial fibrillation. B. Atrial flutter. C. Ventricular tachy. D. Supraventicular arrhythmia✅✅

cyanoticHeartdisease pulmonarystenosis

I
Toffmostcommonl2transpositionoth f Right Aorta
1TOFIguffgriding
ventricularHypertrophy

3 Truncusarteriosis

Baby after every feeding develop apnea and loss of conscious, what is the cause of admission in hospital?

a-Seizure b-Apnea c-Syncope ✅✅

‫ ﻣﺎﻧﺨﺎف‬، ‫ﺎب ﻣﻌﻴﻨﺔ‬ú‫ﺎﻧﺎ §ﻜﻮن ﻋﺎدي ﻟﻪ اﺳ‬¹‫ اﺣ‬، ‫ﻌﺪ اﻟﺮﺿﺎﻋﺔ‬- ‫ ﺑ‚ﻨﻤﺎ اﻧﻘﻄﺎع اﻟﻨﻔﺲ‬، ‫ﺎﻟﻘﻠﺐ‬- ‫ة‬í™‫ﻠﺔ ﻛﺒ‬ù‫ ﻣﺸ‬bd c ‫ﺎﻟﺬات ﻫﺬا ﻣﻌﻨﺎﻫﺎ‬- ‫اﻻﻏﻤﺎء‬

‫ﺐ‬È‫ ﻧﻌﺮف اﻟﺴ‬Ñi‫ و ﻓﺤﻮﺻﺎت ﺣ‬hc‫ﺸ‬Ë‫ﻌﺪ اﻟﺮﺿﺎﻋﺔ ﻫﺬا ﻻزم ﻣﺴ‬- ‫ﻪ‬¹‫ ﻋﻠ‬Á‫ ﻟﻤﺎ ﻃﻔﻞ §ﻐ‬í™‫ ﻏ‬، ‫ﻣﻨﻬﺎ‬.


children with cyanosis and systolic ejection murmur best heard at the left sternal border. Dx?
PS

A. Tetralogy of fallot ✅✅ B.Transposition of great vessels C.Down syndrome

….

!Child noticed having cyanosis with feeding, with Physical exam a Holosystolic murmur was noted:

A. VSD B. TOF C. ASD D. Transposition of great arteries

….

!TOF 5 yo kid with history of surgical correction at 6 months. Now came with new left parasternal

decrescendo diastolic murmur with single S1 with left parasternal impulse, no radiation. What is the dx?

smbiastoic
A. Mitral onsdiastolic
stenosis B. pulmonary mssystolic
regurg C. tricuspid regurg

tricuspid=right....‫ﻟو‬pulmonary
e ‫ﻲ‬ ‫ﻧ‬‫ﻌ‬ ‫ﯾ‬ left ‫ وﻻن ﻗﺎل‬regurg ‫ دوروا ﻋﻠﻰ ﻛﻠﻣﺔ‬diastolic murmur T ‫اذا ﻗﺎﻟﻛم‬

…..

!Diastolic decresendo murmur in left sternum woith prevous TOF = pulmonary regurge ,

..... .....

Right axis deviation and Rt BBB

! case of TOF . How does it appear on ECG? right axis deviation! because VSD let the blood transport from Lt to Rt

! Pt with Hx of TOF repair present with murmur in parasternal area it increase with inspiration =

pulmonary regurgitation, becz TOF has pulmonary stenosis.

.‫ ﻧطﻠﻊ ﺑرة ﻧﺷم ﺷوﯾﺔ ﺟو وﻧﻔس‬para = ‫رﺑط= ﯾزﯾد ﻣن اﻟﺷﮭﯾق ﺗﻧﻔس = رﺋﺔ‬

…..

! TOF( Tetralogy of Fallot )mangment . ?

A-Dirotic iv B-pain relif and sedation ... ✅✅ C-no oxygen in answer

NmorphineforsedationFluids

!classic case of DiGeorge syndrome (check the manifestations please) asking about the heart
awom.me

abnormality: catch22 cardiac


Abnormal
cleftpalate
face HypocalcemiaHypopara

A.TOF ✅ B.PDA C. TGA D. Endocardium cushion defect ThymicAplasia

foot.............‫ ﻟل‬. ‫ = ﻧﺑﺣث ﻓﯾﮭﺎ ﻋن ﺷوز ﻛوﯾس‬Googl ‫ = ﺟري‬Gerog = ‫ = ﻧﻌﻛﺳﮭﺎ ﯾطﻠﻊ ﻓوت‬TOF ‫رﺑط‬

.....
profoundcyanosis IS'evercases Tx IV prostaglandin

%Child with peripheral and central cyanosis? TOF

.. . . . . . until SX

270- Child with cyanosis central and peripheral what is cardiac anomaly? A- TOF✅ B- PDA
‫ اﻻﻃﺮاف واﻟﻔﻢ §ﻜﻮن ازرق‬bd c ‫ »ﻞ ﺟﺴﻤﻪ‬äd‫ﺔ اﻧﻪ §ﺨ‬¹‫ﺎﻟﻘﻠﺐ »ﺎﻓ‬- ‫ﺸﻮﻫﺎت‬Ó ‫ ـﻊ‬è‫
ار‬hd i ‫ =ف ﻣﻨﻄ‬Ï c
d ‫ﺎ‬è‫ ر‬Ñd ‫ =§ﻌ‬ÎÖd ‫اﺗﻮﻟﻮ‬í™‫ ﻣﻦ اﺳﻤﻬﺎ ﺗ‬.
Presentation of Di-George syndrome
• Neonate with tetanus has hypocalcemia and recurrent infection and thymus aplasia or hypoplasia

• Chromosome abnormality deletion in 22 q11

• Heart abnormality in digeorge (TOF-VSD-ASD-PTA)


.... .... ...

!Family known about CongenitaL heart disease of baby but not known what it by examination central

and peripheral cyanosis = Tetralogy Of faLlot !

c $Neonate developed cyanosis (2nd or 3rd week after delivery) and there is finding on auscultation I
cannot remember exactly but they didn't mention about machinery murmur the qs about the

them

management? A. NSAID B. Steroid C. Prostaglandin✅ Incase severeRTOTI likeinthiscasewe


of give
The “5 Ts” DO cyanotic Congintal Heart Disease = prostaglandinto keepPDA

Tetralogy of Fallot, Transposition of the great vessels, Tricuspid atresia, Total anomalous

pulmonary venous return, and Truncus arteriosus.

‫وﺳﺘﺎﺟﻼﻧﺪﻳﻦ‬s®‫ﻠﻬﻢ ﻋﻼﺟﻬﻢ |ﺎﻟ‬-

!child with transposition of great vessels. Further evaluation of his mother could reveal? Elevated fasting

TGA
blood glucos !
TP06T
• Neonate of diabetic mother

… Infant to
born diabeticMother • Cyanosis in first 24 hours of life
• Confirmatory Echo

mtyphs sub glottic Tx : Prostaglandin E1 it keeps the ductus arteriosus patent

...........

% Same Q and ask about concern sign (croup ) ? cyanosis %% indicatesevererespiratorydistress

‫ﻂ‬è‫ = ر‬croup = ‫ ﻛﺮب اذا ﺷﻮﻓﻨﺎ وﺟﻬﻪ زرق‬bd c ‫ ﻛﺮوب =ﻧﻜﻮن‬%%

.....
!Concerning symptom in croup? • Blue lips✅ • Tachypnea. • Expiratory stridor

We will give inhaled Epinephrin if no response we will give again Epinephrin if no response we will add steroid
............

# croup given epi what's next step? steroids ✅(no epinphron on choic)
Moderateto severeRacemic

epineph
… r

fasterthe steroid

! Egg shaped heart shadow. What's the congenital heart disease? TGA✅!

%% Pt congenital heart disease and discretion in x ray egg shape what diagnosis ? Transpostion Great

arterios
eftx.in

!Neonate developed cyanosis (2nd or 3 rd week after delivery) + there is finding on on auscultation

machinery murmur = PDA!

. ‫ ﯾﻼ اﻟﻣﺣﺎﺿرة ﺑدﺋت = ﻣﺎﺷﯾﯾن اﻻن ﻟﻼﻣﺣﺎﺿرة‬، ‫ ﻣﺎﺷﯾﯾن‬machin = ‫ﻣﺎﺷﯾن‬PDA =‫رﺑط= ﺑدء‬


estate

.............
of
Q about cyanotic spell, Mx ? knee-chest position, oxygen

….
Sedation Fluids

!child crying and cyanotic=


..
sedation and relieve the pain
TTF

!baby cyanosed with parasternal heave no murmur what to give? Prostaglandin

..

‫د‬Child pt cyanotlc . can't complete one sentence= intubation

..

!Child, k/c CHD , presented with cyanosis , progresing, o2 sat 85, looks ill, cyanosed, crying

A. immediate catheterization B. diuretic IV C. sedation + analgesia

.. Leg firstor

!2 months old baby mother complaining of apnea usually happens after feeding with 10 mins of

cyanosis, why will u admit this case?

A. acute life threatening event B. Seizures C. sepsis

....

ARS

Baby born at 27 weeks GA developed SOB, tachypnoea. No X-ray. Diagnosis? apnea of premature

‫ﺎ‬P‫ﻜﻮن ا¸ﺘﻤﻞ ﻧﻤﻮ‬v‫ع ﻟﺴﺔ ﻣﺎ‬µ‫ واﻟﺮﺋﺔ ﺑﻬﺬا اﻻﺳﺒ‬، ‫ ﻻن وﻟﺪ |ﺪري‬.

80

! Mother brought her 2 years old child to the ER with a history of upper respiratory tract infection for the

last 3 days with mild respiratory distress. This evening the child started to have hard barking cough with

respiratory distress. Which of the following are the most worrisome signs in this condition.

croup
A. Nasal flaring B. Barking cough Cyanosis if not in choices > A

..

!Neonate cyanotic, low o2, Dx : A. hypoxic ✅ B.Hypocapnic C.Hyoercapnic.= ›c ™ ‫ اوﻛﺴﺠ‬bd c ‫ﻣﻦ اﺳﻤﻬﺎ ازرق ﻟﻮﻧﻪ ﻻن ﻣﺎ‬

….

What is the least physical activity duration required in pediatrics: 60 min

‫ﺎﺿﺔ‬Ò‫ﻌﻤﻠﻮا ر‬Ò‫ﺎﻟﻴﻮم و‬- ‫ اﻻﻃﻔﺎل ﻟﻠﺰم §ﻤﺸﻮا ﺳﺎﻋﺔ‬Ñi‫ﺣ‬

%Patient playing sports and frequently developing sudden attacks of LOC, HOON
examination: mid-systolic murmur in left side, What’s dx? Hypertrophic cardiomyopathy %

‫ﻠﻌﺐ‬¹‫ اذا ﺷﻮﻓﺘﻮا »ﻠﻤﺔ اﻧﻪ ﺷﺎب ﺑ‬sports ‫ﺎﻟﻘﻠﺐ‬- ‫ﻠﺔ‬ù‫ﻮا ﻟﻤﺢ ﻟ”ﻢ اﻧﻪ ﻋﻨﺪە ﻣﺸ‬Ò‫ واﻟﺴ‚ﻨﺎر‬، ‫ﺎﻟ”ﻢ ﻣﺮض‬- bd c ‫ﻪ ﻻاازم ﺗﺤﻄﻮا‬¹‫ ﻋﻠ‬Á‫وﻣﺎت او اﻏ‬

، ‫ ﻣﺮض‬bd c ‫ ﻃﻮل‬ä‫ ﻓﻜﺮوا ﻋ‬Hypertrophic cardiomyopathy ، ‫ وﻫﻢ‬í™‫ﺎب ﻋﻤﺮﻫﻢ ﺻﻐ‬ú‫ ﺑﺘﻼﻗﻮا ﺷ‬، í™‫ﻘﺔ ﺑﺘﺤﺼﻞ ﻛﺜ‬¹‫وﻟﻠﻌﻠﻢ ﻫﺬا ﺣﻘ‬

‫ §ﻠﻌﺒﻮا‬sports ‫ﻦ ﺗﻈﻬﺮ ﻣﻊ‬ı‫ ﻟ‬، ‫ﻈﻬﺮ اي ﻋﻼﻣﺎت‬¹‫ اﻟﺮاﺣﺔ ﻣﺎ ﺑ‬bd c Ùd ì ‫ ورا‬hd i ‫ اﻟﻘﻠﺐ ﺧﻠ‬bd c ‫ ﻋﻨﺪە ﺗﻀﺨﻢ‬¤d‫ﺐ اﻧﻪ ا‬È‫ واﻟﺴ‬، ‫ ﻋﻠﻴﻬﻢ‬Á‫ﻓﺠﺎة §ﻐ‬

‫ﺔ‬¹‫ﻘﻞ »ﻤ‬¹‫ﻪ ﻓ‬¹‫ ﻓ‬¤d‫ﺐ اﻟﺘﻀﺨﻢ ا‬È‫ §ﺪﺧﻞ ﻟﻠﻘﻠﺐ ¯ﺴ‬bd c ‫ﻪ ﺗﻀﺨﻢ ف ﻻ ﻳﻮﺟﺪ دم »ﺎ‬¹‫ﻘﻮة وﻓ‬- ‫ﺾ‬úˆ‫ وﻗﺘﻬﺎ اﻟﻘﻠﺐ ﻣﺎ§ﻘﺪر ﻳ‬، ‫ﺬل ﻣﺠﻬﻮد ﻗﻮي‬-

ï
‫ء ﻳﺘﺎﺛﺮ‬î d ‫ اول‬¤d‫ﺎﻟﺘﺎ‬è‫ و‬، ‫ﻀﺔ‬ú‫ﻘﻞ ﻗﻮة اﻟﻨ‬Ò‫ ﺗﺪﺧﻞ ﻟﻠﻘﻠﺐ و‬¤d‫ اﻟﺪم ا‬brain ‫ ﻣﻊ اﻟﻤﺠﻬﻮد ﻣﺎ§ﻘﺪر‬í™‫ وﻣﻤﻜﻦ ﻟﻮ اﻟﺘﻀﺨﻢ ﻛﺒ‬، ‫ﺤﺼﻞ اﻻﻏﻤﺎء‬Ò‫و‬

‫ﺾ اﻟﻘﻠﺐ ﺧﻼص ف ﻳﺘﻮﻓﻮا‬úˆ‫ ﻳ‬.

.. . . . .. . . . .

!Preschool checkup for asymptomatic 7 years male, with grade 3 systolic murmur best heard in the

lower left sternal border, intensity increases with standing, the most likely diagnosis: HOCM or MVP !

…. holosystolic murmur over left lower sternal border ( VSD)

!Pediatric with holosystolic murmur in left 3rd intercostal space = ASD ( Not PDA)! holosystolic

murmur usually caused by ventricular septal defect, mitral regurgitation or tricuspid regurgitation,

...............

!8 year , with late systole ,mid sternum ,crescendo decrescendo, high pitched : systolic regurgitation

… stenosis early
$ baby with Ejection systolic murmur with click heard in the left second intercostal and left parasternal

heave distended JVP and ejection systolic murmur increase with inspiration:

A. AS. B. coarctation of aorta. C. VSD. D. congenital pulmonary stenosis ✅✅ Mor Tof


Click + Ejection systolic murmur +scond intercostal + left = pulmonary stenosis

.............

! Pistol shot murmur in pediatrics case : aortic regurgitation

%VSD picture , symptomatic = refer for surgery %

!child with mother death + he is not concentrantion in his lecture = dissociation


‫ﺖ و˙ﺣﻨﺎان‬ë‫ﻌﺪ ﻣﺎ ﺗﻮﻓﺖ اﻣﻪ ﺟﻠﺲ ﺳﺎاا‬- ، ‫› =ﺳﻜﻮووت‬ c ™ ‫ﻂ =داي ﺳﻮ»ﺎﺷﻦ =داي =ﻣﻮت ﺳﻮ»ﺎﺷ‬è‫ر‬

1laryngomalacia M

I
$Child with noisy breathing in prone position decreased when the child is sitting, the mother is worried
A. tell her his condition wills get worse B. -He needs surgical intervention
stridorworsen
inspiratory

C. He will get better spontaneously at the age of 1 year old✅✅


laryngomalacia

… z ylo

$Pediatric male pt presented with sign of “ laryngomalacia “ what’s the TTT:

A. Referral to surgical department B. reassure and wait for 1 year✅

Answer: no need for intervention so B It will resolve within 2 years

..

! larngyomalicia what you well do? reassure its will resolve in her first birhday

NB :If there's in2 years more accurate

….

!Infant with airway obstruction that Increase with supine, decrease with prone= Laryngomalacia

!baby has breathing sounds his age is 3 months mother is worried

a. normal b. will outgrow it at age 1 year 90% resolve with time✅ N.B. laryngomalacia

supine pionee
typical case of laryngomalacia, dyspnea worse when prone and better when supine
h
of

w*o*rse when pr*o*ne

… X
!Baby crying when change position decreased what couse? laryngomalacia

….

!3 months old mother's complain of noisy breathing sounds during sleep and disappeared when he

prone position and increased in supine position What is diagnosis?

A. Nasal atria B. Laryngomalacia C. Vocal cord syndrome D. Subglottic stenosis

Baby developed SOB , whezzing DR start to give broncodilator then no response, think of laryngomalacia

dx test is ? Bronchocsopy ( no laryngioscopy in choices)

… FlexibleLaryngioscopy I
!Laryngomalacia case and ask about the best investigation ? laryngoscope

# tracheomalacia on children = What is diagnostic image ? By Bronchoscopy

‫ ﻣﺗﻛرر وھو اﻧﮫ ﻟﻣﺎ ﯾﺎﺧذ ﻧﻔس اﻟﻘﺻﺑﺔ اﻟﮭواﺋﯾﺔ ﺗﺗوﺳﻊ وﻟﻣﺎ ﯾطﻠﻊ اﻟﮭواء ﺑﺎﻟزﻓﯾر ﺗﺗوﺳﻊ‬wheezing ‫ھذا اﻟﻣرض ﯾﻛون ﺑﺎﻻطﻔﺎل ﯾﺗﻣﯾز ب‬

‫اﯾﺿﺎ اﻟﻘﺻﺑﺔ اﻟﮭواﺋﯾﺔ ﻟﻛن ﻣﻊ اﻟﻣرض ھذا ﺑﺎﻟﺻوؤة ﻟﻣﺎ ﯾطﻠﻊ اﻟﮭواء اﻟﻘﺻﺑﺔ ﺗﻛون ﻣﺎزاﻟت ﺿﯾﻘﺔ ﻣﺛل اﻟﺻورة ھذه‬

……………… Croup

!barking cough=Laryngotracheobronchitis

….

Physical exam findings (auscultation) in croup ( patient was having nasal congestion , barking cough) :

A-expiratory wheeze and prolonged expiratory phase ✅✅

B-inspiratory sounds die subglottic obstruction Croup : inspiratory stridor with sub-glottic narrowing

# barking cough = stridor = croup = expiratory wheeze. ***INSPIRATORY PHASE***

….

%A child with inspiratory stridor, barking cough, most likely diagnosis?

- laryngotracheobronchitis%%

Epinephrine 2nd dose steroid

…. CesTxinhdedRacemic
x
!croup ++ barking cough = expiratory wheeze and prolonged expiratory phase ✅

%barking cough and respiratory distress: croup

‫رﺑط = ﻛروب‬croup = ‫ ﻛرب = ﻟﻣﺎ ﯾﻛون اﻟواﺣد ﻓﻲ ﻛرب ﯾﺣﺗﺎج ﺑرﯾك راﺣﺔ‬bark = ‫ﺑرﯾك‬

!Barking cough ( croup ) what will listion on auscultation ?

Increase inspirantion sound due to subglottic narrow ( inspirantion stridor ) (Not expirantion ❌)

.. . . . . . .

Barking cough ( croup ) what will give ?


Ÿ
Ms
Inhaltion epiniphrin and oral steroid ✅ We will give inhaled Epinephrin if no response we will give again

Ÿ Inhlation steroid ❌ Epinephrin if no response we will add steroid (oral)

Ÿ Empric anticiotc ❌

..

!Croup classical case barking cough + inspiratory stridor + Auscultation: inspiratory stridor DX : croup

Croup : Larybngotrachobroncitis
cause : parainfluenza • Inspiratory stridor- Barking cough

• Parainfluenza Virus ‫ ﻧﺷم ﺷوﯾﺔ ﺟو‬para ‫ﻛروب = ﻧطﻠﻊ ﺑرة‬

• Diagnosis clinically
......

• Lateral Neck X-ray (steeple sign)


!child is having barking cough = viral croup Tx: -Conservative

- Inhaled Epinephrin

- Steroid oral
!3 years old Patient with hx of mild atopic dermatitis, presented with Barking cough and stridor, what's

the dx: A- bronchial asthma B- spasmodic croup

....

! Kid with inspiratory stridor, mild respiratory distress, hoarseness of voice, barking cough which of the

following is concerning symptoms: - A. Tachypnea B. Expiratory stridor. C. Nasal flaring D. Blue lips

…..

!hoarseness of voice in Croup ?

inspiratory stridor with subglottic stenosis (Not expirantion)

i
‫ﻂ =ﻛﺮوب‬è‫ ر‬croup = ‫ﺮب‬ı‫˝ واﻗﻠﻞ ﺷﺪة اﻟ‬ d ‫ ارﻓﺢ ﻋﻦ ﻧﻔ‬Ñ‫§ﺤﺘﺎج اﺧﺬ ﺷﻬﻴ‚ﻴﻖ ﻋﻤﻴ‚ﻴﻖ ﺣ‬

...

$CROUP given epinephrine and after 30 min the symptoms came agine = manag by = Reapate again

# croup given epi what's next step? steroids ✅(no epinphron on choic)

….

&Croup----------steeple sign on X-ray&


‫ ﻛﺮوب =ﻛﺮب =ﻻزم ﺗﻜﻮن ﺻﺎﺑﺮ‬steepl

..... ...... ......


Epigbttitismy aosds.to

qg
● Fever, Severe sore throat, high-pitched sound when breathing in (stridor), Difficult and painful
swallowing, Drooling, Anxious, restless child . Feeling better when sitting up or leaning forward what is the

y
diagnosis ?? A. Epiglottis. B. bronchiolitis. C. Pharyngitis

… indications tointubate

!drolling saliva? A-CRUP B-EPiglottis ✅

&Epiglottis-----------X-ray thumb sign& ‫ﺐ‬¹‫ ﻋ‬Epi= ‫) اﻻﺻﺒﻊ‬

….

!Child with fever ,sob , drooling what next = intubantion and mutiblspichil team = Epiglotitis ( ( Dont

choice x ray, bez he ask nest step we will not wait the x.ray)) .!

…..

Child preschool age has VSD 2mm,asymptotic,what will you do= Watchful and waiting ✅

.‫اﻻھم ھو ﻟو ﻓﯾﮫ اﻋراض ﻻزم ﻋﻣﻠﯾﺔ‬mm 5 ‫ اذا اﻟﺛﻘب ﻛﺑر ﻋن‬، ‫ ﯾﻌﻧﻲ ﺻﻐﯾرة وﻻن ﻣﺎﻓﻲ اﻋراض اذا ﻧﻘول ﻧﻧﺗظر ﻋﻠﯾﮫ‬mm 2 ٢ ‫ﻻن‬

.....
!child have vsd 2mm asymptotic == watchful waiting✅!‫ء‬æ ø ‫ﺔ وﻻ‬n‫ة واﻗﻞ ﻣﻦ واﺣﺪ ﺳﻢ ﻓﻤﺎ ½ﺴﻮي ﻟﻬﻢ ﻋﻤﻠ‬sz‫ﺻﻐ‬

Ñ Ö

‫ اﻋﺮاض‬ÉÖ ‫ وﺧﺎﺻﺔ ﻣﺎ‬.

…. Txsurfactant treatment
surfactantlavage

Infant mouconim aspirations management: A-Nitric oxide? B-lavage surfactant support


ventilator

Sever Nitricoxide

‫ اﻟﻲ‬mecnium ‫ ﻋﺷﺎن ﻧﺳﺣب ﻟل‬trachia ‫ ﻣن‬tube ‫ ﺑﻌظﮭﺎ ﻟو ﻣﺎﻧﻔﻊ ﻧدﺧل‬، ‫ ﻟﻠﺑﯾﺑﻲ‬suction ‫ﻟﻛن اﻟﻲ ﻣﻔروض ﻧﺳوﯾﮫ اول ﺷﻲء ﻧﻌﻣل‬

vetilantion .‫ ﻧﺿﻌﮫ ﻋﻠﻰ‬، ‫ وﻟو ﻟﺳﺔ اﻟطﻔل ﻣﺎ اﺳﺗﺟﺎب‬، ‫ وﻧﻌﻛﯾﮫ ﻣﺿﺎد ﺣﯾوي‬، ‫ ﺑﻌدھﺎ ﻧﺿﻊ ﻟﮫ ﻣﺎﺳك اوﻛﺳﺟﯾن‬، ‫دﺧل ﻟﻠرﺋﺔ‬

meconium aspiration syndrome:

MAS occurs, your newborn will need immediate treatment to remove the meconium from the upper

airway. After delivery, your doctor will immediately suction the nose, mouth, and throat.

If your baby isn’t breathing or responding well, a tube may be placed in your newborn’s
windpipe (trachea) to suction the fluid containing meconium from the windpipe. The suctioning may

then continue until no meconium is seen in the material removed. oxygen therapy to make sure there is

enough oxygen in the blood. the use of a radiant warmer to help your baby maintain body temperature.

antibiotics such as ampicillin and gentamicin to prevent or treat an infection. the use of a ventilator (a

breathing machine) to help your infant breathe.

. . .. . . . .

Pediatric patient has fallen from tall building, presented to ER with SOB and right chest pain,

investigations showed multiple fractures multiple sites on his body, CXR showed: Right pneumothorax

with mediastinal shift, your management:

= its Tenstion pnumothorax = A.Thoracotomy. B.Thoracostomy C. Intubation and mechanical ventilation

Thoracotomy vs. thoracostomy

*thoracoStomy is a procedure that is used to drain the space between the lungs and chest wall of excess

fluid, blood, or air.

Thoracotomy is surgery that makes an incision to access the chest. It's often done to remove part or all of a

lung in people with lung cancer.

What is Needle Thoracostomy?

urgent needle thoracostomy (also called "needle decompression") is performed. Simply, it is the insertion

of a large-bore needle or cannula through the chest wall and into the pleural space to allow air within the

pleural cavity to escape.

‫ ﻓﻴﻬﺎ ﺣﺮف‬¤d‫اذا اﻟﺠﻮاب ا‬

….

!child with erythema marginatum , knee pain , fever = what inv =ECG !
*erythema marginatum = its rash with Rheumatic fever ,we order ecg to see any muse heart
Acheck
other choices

damage * • ↑ Antistreptolysin O titer (ASO)

• ↑ Antistreptococcal DNAse B titer (ADB)

1Virus
• Positive rapid streptococcal antigen test for GAS

+Child presented with ulcers on mouth and gingiva erythematous based and pale in the center. Dx?

A. Coxaci ✅+vB.EBV vC.Herpes

....... ...... conjunctivitis


firstfevercoughcoryza
fo

spotsonmouth
Then iRastwhite

Rash on the face and inner cheeck there’s whits spots: = Measles wainspots
‫ﺾ‬¹‫ﻂ =ﻣﻮز ﻟﻤﺎ ﻧﻔﺘﺤﻪ وﻧﺎˇﻠﻪ ﻟﻮﻧﻪ اﺑ‬è‫ = ر‬wight spot ‫ اﻟﻔﻬﻢ‬bd c .

.......

again
!2yrs old child Not vaccinated, had fever for 3-4 days then rash appear, on examination there was white

spot in mucosal membrane. What is dx? A. measles B. Rubella

Pediatric patient with coryza, conjunctivitis, and white spots in the mouth, what is the diagnosis?

A-Measles ✅✅ B-Rubella C-Mumps

$Measles should mention the 3Cs (Cough coryza conjunctivitis) mouth koplick spots

$Child with sore throat and coryza 2 days ago came with difficulty swallow food what is investigation

you will order? A. Chest x ray B. Ct scan✅‫ ﺣﻞ ﺟﻠﻮري‬C. Lateral neck x ray
CTXof

Measlesissymptom a
.. leswedontdoctinpediatric
Age

!Child with fever conjunctivitis, coryza cough, wheezing Tachypenic what is the optimal ttt =O2 therapy

(not Steroid) supportivetherapy

! Child with acute onset fever, Coryza, sore throat, difficulty swallowing solid food. Ex showed cervical LN

2.3 cm. Next step? . CT neck Lateral Neck XRay

....
to

earsthen trunkandextremities.orNgopyefmf.ge
Measles(rubeola) : - Highly Contagious

rash start behind


Rubella
causative organism >Paramyxovirus
Presentation :

Prodrome , 3 C (Cough,Coryza,Conjuctivitis) , Koplik's spot , rash on face >>leg

trunkandextremitiesInvolvesol
to

Measlesrashstartbehindearsthen
No arthritis
Tx : -Supportive therapy only
-Vitamin A reduce Morbidity and mortality
andpalmes

Prevention By Active immunization


Rubella

Rash start at behind the ears >>Peripheral **sparing palms and soles***, Forchheimer spots on soft palate, occipital &post cervical lymphadenopathy , Ployarthritis
Dx : clinically , confirmatory test is serology
Tx : Symptomatic treatment (antihistamines for pruritis and NSAID for poly arthritis)

!Young girl has a recent history of fever, difficulty in swallowing solid food only. enlarged 2cm cervical

LN. investigations will you do ? Lateral neck X RAY. CTScan peritonsillarabscess

… saw
noneed_
!Fever and cough and maculopapular rash in behind ear and face and trunk

A. mubes B. measala C. rubella



r

#7 y/o unvacclnated boy presents with red erythematous irregular patches of rash that is around hls

Roseola infantum(Herpes virus 6)6th disease


neck and spreads down hls back. What does he have?

3 days of fever then resolve completely >>rash start at trunk

A. measles ✅B. Chickenpox C. Rubella D. Pertussis or something weird like that >>Peripheral

fRoseola
Once rash start No more fever

..... Dx : clinically

Tx : Symptomatic treatment (Fluid and possibly acetaminophen to


!A child with runny nose and fever which subsides and then rash appear allover reduce fever)

his body starting from the face. Dx? Rubella, after fever subside it’s roseola, concurrent fever Rubella.

....

!Baby with white eye reflex (Leukocoria) and murmur. Mother mentioned viral infection during

pregnancy: ? A. Rubella ✅ B. CMV C. Toxoplasmosis ‫ﻀﺎ‬¹‫ رﻋﺐ =ﻋﻴﻮﻧﻪ ﺑ‬.

....
!Absent red reflex caused by wich infection?Rubella
OO O as

… V

Face Rash with conjunctivitis spread later to the trunk: = Rubella Measles

‫ واﺣد ﺟﺳﻣﮫ ﻣن وﺟﮭﮫ ل ﺟﺳﻣﮫ اﺣﻣر‬rubella ‫رﺑط = رﻋب‬

.........

%%child with rash started on face and then spread to the trunk. ? Rubella%%

!Newborn with absent red eye reflex and new murmur , what does his mother had when she was

pregnant ? A. Rubella B. Cmv C. Toxoplasmosis Rubella'ssyndrome actuarial anomalies PDA


congenital

….
!pic of skin lesion in child , rash was red and mother tell it start on face the go to trunk 3 ,Deafness
with LN enlarge

of groin , Dx ? A- imptigo B- HSV C- candidia D_rubell


… postauricularandsub occipital t

more geos wlmeasies O

Leukocoria

Kawasaki
o

Baby presents as shown in pic, what would you do? Immediate referral to ophtha✅

!Kawasaki management? A.intravenous gamaglubulines✅ highdose Aspirin

intravenous immunoglobulin (IVIg) can treat include: Immune deficiencies like immune thrombocytopenia.

Kawasaki disease.
.............
Feetswelling
Gunjunctivitis Handle

!Kawasaki sign? bilateral red eyes Rash


Adenopathy
fever 5days

…. tongue
strawberry

!Case about kawasaki how you will assess the coronary artery disease? A. iIVIG+ASA B.ECHO ✅

..

!Kawasaki disease assess for heart complication? Echo Anurysim

!kawasaki case how you will assess the coronary artery disease? by 2D echocardiography (2DE) or

coronary angiography. !

......

confirm

$A child presented with 5 days of fever, oral mucosal lesions, cervical lymph node enlargement and limb

edema. Lab results essentially normal. Drug of treatment?

A. Acyclovir B. Cefotaxime C. Ampicilin D. Aspirin✅kawasaki case if there is IVIG in choices choose it its the mainstay

treatment of Kawasaki
.......

!The best treatment of Kawasaki disease? Aspirin and IVIG

…….

!kawasaki best treatment= a. aspirin b. IVIG Aspirin if no IVIG in choices or Q mean what is the best for fever

N.B. Aspirin for fever, continued for 2 months. High dose IVIG is ultimate treatment


Child with Conjunctivitis Rash cervical lymph nodes fever what to give as a treatment?

- Aspirin✅ - NSAID - No IVIG in the choices

….

!poor prognosis in treating Kawasaki with IVIg? High CRP

..

!A patient with kawasaki features,what is the best indicator as poor response to IVIG?

A. Neutropenia B. High CRP✅ C. Albumin

!A child is complaining of 5 days of fever , bilateral non-purulent conjunctivitis , rashes in palms and soles

( case of kawasaki ) =Aspirin

… Neutrophilia

#Case of Kawasaki ask about what will reduce affect of lvlg =A. Anemia B. Neutropenia ✅

!Which indicated poor response to IVIG In Kawasaki = Neutrophilia, hypernatremia, high CRP
Hyponatremia I

$Kawasaki on IVIG how to know if bad prognosis : C reactive protien ✅ A. Anemia B. Neutropenia

!Kawasaki case asking about which of the following is one of the criteria:

A. anterior uveitis B. myocarditis C. injection conjunctivitis with no exudate D. arthritis

.....

Fever 5 days, conjunctivitis, lymphadenopathy, high ESR and CRP. Dx? A-Rubella B- Kawasaki ✅

…….

!child with fever and then rash and peeling on hands and edema with peeling lips = kawasaki

....

Fever 5 days, conjunctivitis, lymphadenopathy, high ESR and CRP= Kawasaki


True Kawasaki diagnosed Clinically *****

….
but if say confirmation for coronary artery aneurysm Angio if initial Echo

Confirmation of Kawasaki? Clinical ✅ ‫ﻣﺎ§ﺤﺘﺎج ﻓﺤﻮﺻﺎت »ﻠﻬﺎ ﻟﺴﺎن اﺣﻤﺮ وﻋﻴﻮن ﺣﻤﺮاء وﺧﻠﺼﻨﺎ‬

….

!Child with vesicle at lip and gum and proximal tongue and hard palate. = gingivostomatitis ‫ﻣﻦ اﺳﻤﻬﺎ‬

… Fever

!Pedia with Egg allergy contraindicantio A. Yellow fever B. Influenza initialphaser opharyng Tonsil

... GISx

!4yrs old fully vaccinated child, came to the ER with on day history of fever and sore throat which started 3

on the same day. What is dx? A. scarlet fever B. Kawasaki C. Measles

$Pt with pharyngitis for 2 days , what’s the possible complication : sclar fever infectious

Pharyngitis/Tonsillitis;

Definition = inflammation of the pharynx, especially the tonsils if present, causing a sore throat =

… Herpetic gingivostomatitis (HSV 1)


Mainly in children (1-6 years)and Immunocompromised Pt

$peritonsillar abscess = Quinzi Presentation : Prodrome (Fever - malaise) - Pharyngitis - Gingivitis (erythema and painful ulceration especially on the inner cheek,soft palate and tongue )

Dx :-Clinically with confirmation through following test


-Viral culture (gold standered for definitive diagnosis ) ‫ﻂ =ﺑﺮ‬i‫ ر‬peri = ‫ﺎر‬ô‫ﻞ اﻻﺧﺘ‬ô‫ﻚ ﻗ‬v‫ = ﺑ ﺮ واﻟ ﺪ‬Quiz


Tx : Duration (7-10 days)
- First Line : Oral Acyclovir

in severe cases or Immunocompromised IV Acyclovir
-Topical maybe helpful if used early

E I

E
........

Complications

= immune-mediated complications =

'scarlet fever' ,acute rheumatic fever,. ' post-streptococcal GN '. , reactive arthritis, ,

1_SCARLET FEVER

acute onset of fever, sore throat, strawberry tongue

• 24-48 h after pharyngitis, rash begins in the groin, axillae, neck, antecubital fossa; Pastia’s lines + may

be accentuated in flexural areas

• within 24 h, sandpaper rash becomes generalized with perioral sparing, non-pruritic, non- painful,

blanchable

• treatment is penicillin, amoxicillin, or erythromycin x 10 d

……

A 12-year-old girl presents to her pediatrician for a sore throat. Her symptoms began approximately

1 week ago after she attended a sleepover.

Since then she experienced a sore throat and noted a temperature of 101°F (38.3°C). She denies a cough

but has noticed increasing fatigue and difficulty swallowing due to pain. On physical exam, she has

anteriorcervical lymphadenopathyandpatchy tonsillar exudates and swelling. What complication

could he develop? -Scarlet fever✅

Scarlet fever is a disease which can occur as a result of a group A streptococcus (group A strep) infection,

also known as Streptococcus pyogenes. The signs and symptoms include a sore throat, fever, headaches,

swollen lymph nodes, and a characteristic rash.

‫ اول ﻧﺮﻛﺰ اﻧﻪ ﻗﺎل‬patchy tonsillar exudate ‫ ﻣﻦ‬، ‫ﺎس‬-‫ﺘﻮا¸ﻮ‬ÕÃst‫ﺎ ﺳ‬Ãsz‫ﻜﻮن |ﻜﺘ‬v ‫ﺎ‬ô‫ﺬا ﻏﺎﻟ‬P‫ي و‬sz‫» اﻟﺘﻬﺎب |ﺎﻟﺤﻠﻖ |ﻜﺘ‬Ö Ñ ‫ﻌ‬v

‫ اﺣﺪ ﻣﻀﺎﻋﻔﺎت‬group A strep ‫ اﻧﻪ §ﺤﺼﻞ‬Scarlet fever i ‫ ان اﻟﺸﺨﺺ §ﺼﺎب ب ((ﺣﺮارة ))ﻫﺬە اﻫﻢ »ﻠﻤﺔ وﻣﻦ‬Ñd c ‫اﻟ”ﻠﻤﺔ ﻫﺬە ﺗﻌ‬
Ñ

‫ﺎ »ﻠﻤﺔ‬¹‫ اﺳﻤﻬﺎ ﺛﺎﻧ‬Scarlet »Ö Ñ ‫ﻌ‬v Rash ‫ﺔ‬Ã‫ اﻟﻐﺪد اﻟﻠﻤﻔﺎو‬ÉÖ ‫ﻤﺎن –ﺸﻤﻞ اﻧﺘﻔﺎخ‬-‫ و‬lymphoadenopathy .

‫ﺎﻟﻐﺪة و»ﻤﺎن‬- ‫ﻌﺪﻫﺎ ﺣﺮارة ﻣﻊ اﻧﺘﻔﺎاااخ‬- ‫ﺎﻟﺤﻠﻖ وﺟﺎء‬- ‫ ﻓﺎذاا ﺷﻮﻓﻨﺎ واﺣﺪ ﻋﻨﺪە اﻟﺘﻬﺎب ﺻﺪ§ﺪي‬rash ‫ ب‬Á‫ﺴ‬± ‫ ﻧﻔﻜﺮ ان ﻫﺬا‬scler fever

‫ي ﻟﻠﺤﻠﻖ‬í™‫ﻜﺘ‬ú‫ﺐ اﻟﻬﺠﻮم اﻟ‬È‫ ¯ﺴ‬.

Scarlet fever : it might be Post streptococcus GN

maybe they asking about feared complication


if no GN choose scarlet fever

read more about this Q

.... . . . . .. . .

pt with fever ,cervical lymphadenopathy swollen tonsils (without?) patch Possible complication = -

Pharyngitis. - Scarlet fever✅✅

Scarlet fever is a disease which can occur as a result of a group A streptococcus (group A strep) infection,

also known as Streptococcus pyogenes. The signs and symptoms include a sore throat, fever, headaches,

swollen lymph nodes, and a characteristic rash.

withexudate

…. fTonsilitis
! pediatric patient with fever, on examination, there is a white membrane covering the tonsils, the most

likely complication to be happened is: A. Scarlet fever ✅. B. Pharyngitis C.Glomerulonephritis Complications of Acute tonsillitis:-

Suppurative complications :-

… - Peritonsillar abscess

- Parapharyngeal abscess

$Complications of tonsillitis= A. Scarlet fever✅ B. Pharyngitis. C. Glomerulonephritis - Otitis media

- Sinusitis

… feared complication
- Cervical lymphadenitis

- Mastoiditis
Nonsuppurative complications:
!14-yrs had fever, pharyngeal exidate, enlaged LN, most common complication = Scarlet fever - Rheumatic fever

RSV: Broncholtitis on
- Scarlet fever

… -Initially presents with upper respiratory tract symptoms (e.g.,


- Poststreptococcal glomerulonephritis

rhinorrhea), fever, and cough


-Respiratory distress (usually occurs in infants)
$Case of infant have cough and low grade fever ,rash ,runny nose : all choices are correct but you should know

Tachypnea, prolonged expiration

Bronchiolitis
the feared complication from chronic

A. RSV✅ B. rubella Cough + Running nose + Fever T


Nasal flaring, intercostal retractions
tonsillitis PS Glomerulonephritis
Cyanosis

upper RTI followed by symptoms of respiratory distress and wheezing


they always like to ask in exam

…. it could be measles in a child < 2 years of age should prompt evaluation for bronchiolitis!

$Child with fever and vomiting and rash on 2nd day rash become over All body
as

A. Meningococcemia. B. Rocky mountain fever✅. C. Kawasaki. D. Measles Begininwristandtrunk

…….
spreadtotrunk

!Prophylaxis for contact with pertussis: 14 days O

The macrolide antibiotic = erythromycin, clarithromycin, and azithromycin !

pertussis = ‫ﻌﻤﻞ‬v ‫ = ﺟﺪاا ﻣﻌﺪي‬whooping cagh



‫ﻂ‬i‫ = ر‬per = ‫ﺠﺔ‬n‘‫ﺎﻟﻚ اﻟﻨ‬n‫» –ﺸﻮﻓﻪ |ﻌ‬t‫ﻚ =ﺣ‬v‫ = ﺑﺮ واﻟﺪ‬My son = mycin
.............

ons Bilk
C i
$pedia pt not vaccinated,
present with sore throat and cervical lymphadenopathy what’s your dx:

imf

A. diphtheria✅ B. streptococcus = not related to vaccin . C. pertussis = croup

… u
% How many years the pertussis vaccine last? 10 years% Pertussis

$1-year-old, never vaccinated, presented with "hacking" cough and inspiratory something, What's the

organism? Pertussis

….

!Pertussis with severe vomiting, most complications? Pneumonia ( not Pneumothoroax)

!4 months old with proven pertussis infection on macrolide. His 3 and 5 years old siblings are vaccinated

up to date. What is the proper action to prevent the siblings from getting the infection?

A. prophylactic macrolide. B. booster vaccination against pertussis

C. observe them for the possibility of developing the infection.


….
r

!3 month years old present with paroxysmal cough with deep inspiration between

the cough, conjunctivitis, diarrhea, he is up to date with his vaccination.. What is the causative organisms?

A. Chlamydia B. Adenovirus C. Pertussis or ifhe'snotupto evaccinated

… x Ift
15 Investigation:
1I s

! Pertussis case “whooping cough”Ask about diagnosis and

A. Nasopharyngeal swab B. Blood culture

.. ‫ ﻓﺗﺣﺗﯾن اﻻﻧف‬nose =OO‫ ﻛﺎﻧﮭﺎ‬whooping ‫ ﻣن ﻛﻠﻣﺔ‬OO ‫ﺣرف‬

……

Patient 50 yrs. Old. Came for. Routine check. Up visit, by CXR you. Find.Solitary Cheast nodule. About 2*3

patient is asymptomatic ,what is most useful thing to ask. -Ask about old CXR. ✅

!diffuse ST elevation (ECG pic) ttt? aspirin ✅✅ Pericarditis !

if ask about Tx choose NSAID if no NSAID choose Aspirin J

NSAID

….

!Child with Sx of varicella. Has immunodeficient brother. Action with the immunodeficient child?

A. Avoid skin contact B. Immunoglobulins ✅ C. antiviral meds Meningitis :-

infant : S.Pneumonea and Neisseria meningitidis (most common)

… Children and teenagers : Neisseria meningitidis (most common)

Adults 20–60 years : Streptococcus pneumoniae (most common)


________________

1Bacteria infection HMM


Blood cultures should be performed before antibiotic therapy is started
Lumbar puncture (LP): essential in all patients with suspected meningitis, unless there
are signs of increased intracranial pressure (ICP)

If LP is contraindicated: initiate empiric antibiotic treatment immediately and conduct CT

Meningitis with 1 month or less choice (.ampicillin + gentamicin).Or cefotaxime


before LP to rule out ↑ ICP

————
1 month old Tx: Ampicillin + cefotaxime or Genta

Meningitis with more than 1 month age choice ( ceftriaxone and vancomycin). >1 month and <50 years : Vancomycin+3rd generation cephalo (cefotaxime or
ceftriaxone)

Meningitis with Bacilli catalase choice (Ampicillin) . >50 years : Vanco+3rd generation cephalosporins
Complications :-

…. coagulasechoice staph van co Headache- Dizziness- Nausea -Hearing loss (late complication)- tinnitus

$Neonatal lumbar puncture + diplococci Management?? B.ampicillin + gentamicin✅

0….‫ﺑﺳﺔ)ﺑﯾﺳﯾﻠﯾن( = اﻟﺟن ﯾﺗﻛﺷل ﻋﻠﻰ ﺑﺳﺔ ﻋﺷﺎن ﯾﺧرب اﻟﺧطﺑﺔ‬.☠ + ‫ﺟن‬- + ‫ دﺑﻠﺔ‬:‫…رﺑط‬.

…..

3 days old , csf culture showed gram + bacilli catalase + beta hemolytic , how to treat? ampicillin ✅✅
a

$15 months with meningitis, Gram stain G+ double coccus tt=

A. vancomycin and gentamicin B. ceftriaxone and vancomycin✅ C. ampicillin

….

!3 day neonate with B hemolytic and catalse +ve what antibiotic give:

I )ampicillin 2)gentamicin 3)ceftriaxone neonate and children :

…. Dipplococci gram + bacilli >> Lesteria monocytogens


if there is( Ampicillin and cefo) or( Ampicillin and Genta) choose it
if like this choose Ampicillin

! !Child 15 month with meningitis lp show gram+.cocci mange?A. Vanco+ceftr B. ampicillin +gentamicin

….

! 15 m old baby with sign of meningitis on csf analysis you found gram positive diplococcai what is the

treatment ? A.ampocilib B.voncomycin C. ampicilibe with guntamycin D. vancomycin with ceftraixon

viral

pre glucose normal protein ,increases


% 8 years old child came with fever ,neck stiffness ,irritability, normal

WBC what's abx used ? A- Ampicillin. B- Ceftriaxone. C- Vancomycin. D- Ceftriaxone and vancomycin. ✅

….

!7Y with meningitis , Gram stain G postive diplococci tt: = ceftriaxone and vancomyci ‫ﻌﺾ‬- ‫› ﻣﻊ‬ c ™ ‫ ﻻزم اﻻﺛﻨ‬.

….

$ Child with pain that last for 10-15 mints (intermitted pain) crying , postive kering sign ? What is the

best treatment : Ceftrixon + Vanco %Kernig's sign = meningitis %

viral
7 y/o with meningeal irritation, headache, and fever. CSF (normal protein and normal glucose and

lymphocytosis). What you will give the child?

A.Ceftriaxone and vancomycin and steroids B.ceftriaxone and steroid

C. antiviral ✅✅ Bez( normal glucose ) O

.......

#Pt 7 yrs old with nausea and vomiting dehydrated comatose acidotic CSF : high protein , normal

glucose = viral meningitis ✅

%5 y/o with fever, lethargy, positive Brudzinski sign. CSF showed lower limit of glucose with high protein.

Gram stain revealed gram positive cocci in chains. Your management? dexamethasone administered

A. Ceftriaxone, vancomycin, and steroid* ✅ B.ceftriaxone and steroid C.antiviral with the antibiotics reduces the
risk of long-term complications

......... ..... such as deafness.

7 y/o with meningeal signs, headache, and fever. He and his family came from Africa recently. He also has

sore throat and lymphadenopathy. CSF (normal protein and normal glucose and lymphocytosis). What you

will give the child? A.Coronavirus B.CMV C. EBV ✅

.......

!Child after returning from Africa. Presented with fever, sore throat followed by meningitis symptoms.

CSF shows normal protein and glucose but high lymphocytes = organism? EBV (not CMV)

….
one typhoid fever because has sore throat and lymphadenopathy
Fever for 6days and tender splenomegaly, which culture is most importantly needed?

-urine and stool culture -repeated blood cultures ✅ -bone marrow smear culture

‫ﺪ‬Ò&‫ اﻟﺘﺎ§ﻔ‬bd c ‫ ﻧﻔﻜﺮ‬، ¤d‫ﺠﺎ‬¹‫ﻠﻴﻨﻮﻣ‬ú‫ﻪ ﺳ‬¹‫ ا§ﺎم و §ﻜﻮن ﻓ‬٦ ‫ﺪ ﻋﻦ‬Ò$‫ ﻟﻤﺎ اﻟﺤﺮارة ﺗ‬.
aya

Baby mele fever wet his diapers what is suggests uti in that baby wetting dippers boy == fever✅
‫ ﺧﺎﺻ
ﺔ ﻓﻲ‬urinary tract infection ‫ ﻻن ﻣن اﻋراض‬، ‫ وھو وﺟود اﻟﺣرارة‬UTI ‫اﻟﺳؤوال ﻗﺻده اﯾش اﻟﻲ ﯾﺧﻠﯾﻧﻲ أﺷك اﻛﺛر ان ﺳﺑب ﻛﺛرة اﻟﺗﺑول ھو ﺑﺳﺑب‬

we should see the other choices .‫اﻻطﻔﺎل ھو اﻟﺣراااارة ﻣﻌﮭﺎ ﻛﺛرة اﻟﺗﺑول‬

...... we can say fever but maybe there are another choice better than fever

# boy 15 years old fever, abdominal pain, splenomegaly? multiple blood cultures✅

… simplecystits

pre
!Girl 7 years old with suprapublc pain No rebound no guarding. Tx?

A-observ B-admits iv abx C- discharge with oral Abx ✅

,,,

$7 year old child, presented to the emergency by his partners with 2 days history of fever and vomiting,

child is comatose dehydrated with acidosis. CSF report: Cells 20 (above normal) , Protein 0.45 (above

normal) ,Glucose (Normal)

A. Tubercular meningitis. B. Salicylate toxicity. C. Diabetic coma. D. Viral meningitis. ✅

…..

!child with chill ,fever ,irritability , on exam there was neck rigidity and positive Kernig’s,CSF showed

clear color, lymph and neutrophils high, protin and glucose normal, dx?

A. pneumococcal meningitis B. Meningococcal meningitis ‫› ﻧﻮرﻣﺎل‬ c ™ ‫وﺗ‬íÖ‫ﻻ ﻣﺎﻧﺨﺘﺎرە ﻻن اﻟ‬

C. aseptic meningitis ✅ D. pyogenic meningitis

….

!Girl confused with fever + sign of meningitis = encephalitiis !(Bez confustion somthing happen to

brain , bactiria goto brain) if no meningitis in choice choose encephalitis

$child meningitis, what is the complication ? A. blindness B. hearing loss✅

….

!Chime with meningitis came with his parents and has papilldema , parents are afarid of ?

A-Hearing loss ✅ B-Vision loss

….

in
Meningococcal meningitis
cutaneouspetechiae

Meningococcemia
child with Irritability ,headache ,nausea, lethargy and rash all over the body what is dx : A.

meningococcemia ✅ B. kawasaki Pyogas p

! 3 mo old boy with pic of bacterial meningitis What’s most common pathogen?
or Nesseriamengig strep Atelectasis 13
f
A. Moraxella catarrhalis. B. Streptococcus pneumonia. C. Streptococcus pyogen

!pediatric patient have meningitis, with close contactor to his brother recently, Asking for what to give to

Meningitis :-

his brother: A. Rifampicin B. IVIG infant : S.Pneumonea and Neisseria meningitidis (most common)

Children and teenagers : Neisseria meningitidis (most common)

…. Adults 20–60 years : Streptococcus pneumoniae (most common)

! 3 y/o boy with maculopapular rash on this limbs and buttocks is brought to the ER by his partners

saying he is lethargic and irritable. On examination, neck stiffness is noted. LP shows diplococci parents

are concerned about his 6 y/o brother. What prophylaxis will give= Oral rifampicin

!Most common cause of ear infection or otitis media in children ? A. Bacteria B. Virus C. Fungal

! Child came from africa. complaining of weakness, he couldn’t move his head and legs especially when

he is prone. What is the dx? A. Polio B. Cmv

Bacterialsuperinfection following

viral013Tinfection Rhinovirus
Bacterial 0M 5 pneumonia
ee
….

!At school age what we tend to prevent?

g
A. Japanes encephalitis B. Hemophilus influenza encephalitis✅ C. Herpes encephalitis

….

!Children with 2 days history of ear pain, exam reveals perforated ear drum with fluid passing through

it= A) Chronic otitis media B)Acute suppurative otitis media ✅ C) Otitis externa

….

!Most common virus cause of acute otitis media in pedia =Rhinovirus

‫ﻂ‬è‫ = ر‬rhino = ‫ اﻧﻒ‬Ñd c ‫ = §ﻌ‬ottits ‫ﻌﺾ‬ú‫› ﺑ‬ c ™ ‫ﻄ‬ú‫› ﻣﺮﺗ‬ c ™ ‫اذن اﻻﺛﻨ‬

...

!Child hasfever with perforated tympanic member and pus in the external canal = Acute OM ( Not

Otitis media with effusion ) stuppurative

! Pediatric with fever, ear pain ruptured tymp

A- Acute otitis media. B- Otitis media with effusion. C- Otitis externa

child came with limping and non-weight bearing, not allowing anyone to touch his leg, most common

organism is: staph. Aureus ✅+(septic arthritis)

.‫ ﻻ ﺣد ﯾﻠﻣﺳﮫ‬، ‫اوه ﺑس ﯾﺄﻟم‬aureus = ، ) ‫رﺑط = ﺳﺗﺎف = )ﻋﻠﻰ اﻟﺣﺎﻓﺔ ﻣﺎﯾﻘدر اﺣد ﻣﺟرد ﺑس ﯾﻠﻣﺳﮫ‬

….

$child with fever and left knee pain and swelling. Most important single investigation?

A. Blood culture. B. FBC. C. Joint aspirate✅✅. D. Xray Impetigo:


It’s cause usually by Staph aureus

… Other cause: streptococcus pyogenous


Clinically present with honeymoon ( honey color crest)

& Conductive hearing loss could be because of recurrent OM.& Dx: clinically
Treatment;

…. Vesico or nonbullous: muprocin

Impetigo 5 Aureus
!honey crust infection= A) staph aureus ✅. B) GA streptococcus. C) herpes
Bullous or severe non-bullous impetigo : first generation cephalosporing : cephalexin or
dicloxacillin

Complications :
In GAS infections:

….. Acute poststreptococcal glomerulonephritis (PSGN)

!Pediatric has unilateral hip joint pain Flexing Refuse to be touched Mostly causative organism?

A. Staph aurus ✅✅vB.Group b strep vvvC.H influnza

….. septicArthritis

child with limping for 2 day and abnormality in hip what's most likely organism=A. staph ✅. B. GBS C. Hib

......

$Child with pneumonia, indication of hospitalization?

A. Vital signs( hypotension and tachycardia) B. not able to take orally ✅ one of the indication for intubation (unable tolerate orally )

4 years old with pneumonia, with vomiting for any oral food .. vitaly stable; reason for admission:

A. pneumonia B. unable to tolerate orally

….

!headache, stiff neck, and vomiting, coughing and breathing with breathing difficulty , causative

organism ? strep. Pneumonia if infant or 720ylo

P
dBBRpends

!Pediatric Pneumonia = A. Iv B_ oral Abx

…..

!13 months old girl present with fever 38 , bilateral lung infiltrate , she looks mildly ill , what is the likely

vviral
as pneumoniae
organism==A. Moraxella catarrhalis B. Strep C. Hib influenza

…. Most likely Viral >RSV, Adenoovirus

! Baby with tachypnea, cough, hemoptysis and bilateral lung infiltrates. What is the treatment ?

A. Steroid. B. Antibiotic✅. C. Surgery. D. Betablocker

Hemoptysis is the coughing up of blood or blood-stained mucus from the bronchi, larynx, trachea, or lungs.

In other words, it is the airway bleeding. This can occur with lung cancer, infections such as tuberculosis,

bronchitis, or pneumonia, and certain cardiovascular conditions

!Pneumonia ask about treatment

! child with flu like symptoms+ fever + has middle lobe crackles+ stony dullness, chest x.ray will show =
pleural effusion bez dulness not pnimothorax e

… cough coughs
withdeepinspirationist
in
as
paroxysmal

!Staccato cough : . chlamydia pneumonia

….

!Children with cough, fatigue, 2 time bloody vomiting, low grade fever, with dullness in percussion dx?

A. parapneumonic effusion B. pleural effusion C. TB overweeKs ***when we say parapneumatic effusion ?

…. nofever if Q give me symptoms of pneumonia then it become


effusion

!Absolutely contraindicantion in penicillin allergy? Pipracillin/tazobactam

....

!Child with poor feeding since 2 days have oral thrush and dipper dermatitis what you will give

A. Topical antifungal. B. Oral antifungal. C. Topical and oral anti fungal D. Systemic antifungal

….

!Pediatric pt with Rash in cheeks trunk and upper limb:

A. dermatitis herpetiformis (location typic on extensor surfaces like elbow)

B. Impetigo C. Candida D. herpes simplex


…Voiding cystourethrogram (VUR) Eczema

I
Herpeticum Head upperBody
Cystourethrogram of a pediatric with recurrent
UTI =

VesicoUrethral reflux is a condition in which urine flows backward from the . bladder to one or both

ureters and sometimes to the kidneys

.. . . ...... ...... .....

case of febrile neutropenia what next? cluture form sputum,urine,blood and Iv antibiotics ‫ﺎن ﻧﺎﺧﺬ‬ù‫ﻣﻦ »ﻞ ﻣ‬

AA ‫ﺸﺎر‬Ë‫ »ﺎﻟ‬.

AA Patient presented with typical picture of malaria infection, blood smear showed no

parasite, what’s your next step?

o
e

A. Repeat thin blood smear. B. Repeat thick blood smear C. Repeat every 8 hours for two days ✅

….

!Tonslitis = amoxacillin / clavi .!

..

!Neonate with sign of sepsis what is empirical antibiotic ?. Ampicillin!

..

!Q about treatment of uncomplicated cytits in child ?

A.iv ceftrixone B.im ceftrixone C. oral amoxicilln ✅✅. D.oral carbonate

....

!Pediatric prophylaxis for maningitis? rifampicine !

….

…..

Baby with septic arthritis (scenario not directly saying the diagnosis) what is the best study:

A) fluid aspiration from the joint✅ B) blood culture C) CRP


urethritis chlamydialpostenteritis shigellassalmonell
........ infectionpost

reoccureatterottor

!Young girl with diarrhea came with left knee swelling, right elbow, left Achilles tendon. Stool analysis

shows +ve clostridium toxins. What is the dx? arthritisasymmetricalmigratory


A. JIA B. Reiter's syndrome C. septic arthritis D. Reactive arthritis
poly

....N.B: Reactive typical sites > knee and ankle.. Reactive arthritis = Reiter's syndrome

!Baby has swollen tender joint, limited passive movement, the most important test: Examination of

synovial fluid
‫ﻪ‬¹‫ ﻧﻔﺤﺺ اﻟﺴﺎﯨﻞ ﻋﺸﺎن *ﺸﻮف اذا ﻓ‬wbc ‫ﻌﺪ‬úË‫ء *ﺴ‬î ï c

d ‫ اﻫﻢ‬Ñd ‫ §ﻌ‬، ‫ﺔ او ﻻ‬¹‫ ﻋﺎﻟ‬septic arthritis . ‫ﻞ اﻟﺴﺎﺋﻞ‬¹‫ﻖ ﺗﺤﻠ‬Ò¬‫ﻜﻮن ﻋﻦ ﻃ‬¹‫ ﻓ‬.

......

When resuscitating a child with septic shock, which of the following has the least evidence of benefit in

treatment? A. ABX B. inotropes C. steroids ✅✅


forHypotensivepatients
Ped URTI lethargy , confuse , Tem39, rr35 , hr>100 = A.Sepsis ✅ B. Septic shock

If hypotension and not respond to ivf = septic shock

Severe sepsis = organ damage

.....

$Commonest cause of shock in children? .sepsis✅

!baby with septic shock, what is less indicated? steroid

..

!Neonate with High fever, developed petechial rash and was hypotensive 70/55, with cold extremities

and poor feeding. What is the dx: A. septic shock B. ITP

..

!Case of child with leukemia , after 17 days of chemo coming with fever , neutrophil is normal, tx? blood

culture , urine culture , broad spectrum iv abx !

!Child with fever, vomiting and diarrhea. ABG: Normal pco2 + Normal bicarb Base access -4 “normal

range from -2 to 2” Ph 7.3 What type of shock does he have?

early compensated shock ✅ Not late but early = bez evry thing normal .

+ve sexual history 6 years ago , labs WBC 0.3 Serology: cryptococcosis positive What is the best diagnostic

test = HIV serology

............ ....... invasivefungus

consideredAnAIDSdefiningillness

!Pedia pt under went tonsillectomy, the father notes that surgery take more time than usual but pt

recurved well, thre was bleeding during surgery they ask what you tell the father? Explain to the father

what happened !

….

$Child with flu symptoms on examination he has membranous exodus in tonsils, causative organism


A
A. streptococcal✅. B. EBV ‫ ﻓﺎﻳﺮس‬Scarletfever Group Streptococcus yogasproducingToxin
Ip

$child present to the clinic with pain in micturition and lower abdominal pain

all symptoms of UTI and his parents observe in his urine foul smell, which organis can cause this

condition? A. E coli gram negative✅. B. E coli rod in shape. c. Proteus gram negative bacilli

70 ‫ﺎ‬Òí™‫ﻜﺘ‬- , d ‫ﺔ ﻣﻦ ﺣﺎﻻت اﻟﺘﻬﺎب اﻟﻤﺴﺎﻟﻚ‬¹‫ﺎﻟﻤ‬- e.coli


$Child Foul smelling (urine) which organism?E. Coli. WaterydiarrheaGiardia sis

Ambiasis

… Bloodydiarrhea

..... ......

%% 10y boy with hx of 2wk of bloody diarrhea and abdominal pain, tenesmus: Amebiasis%%

Child c/o fever, bloody stool, and tenesmus, abdominal exam showed abdominal distention, Dx?

A. Ascaris. B. Amebiasis✅. C. Giardiasis ( watry diarrhia ) D. Rotavirus

!bloody stool, and tenesmus, fever = Amebiasis! bewatery

!child has bloody diarrhea and oliguria, vomiting, nausea, abdominal pain, fever for 7 days before that

family think this from restaurant What's ,treat= A.- antibiotics✅ B.- supportive C.- steroid bacterial
Gif
(shegilla ,amebiasis ,hemorrhagic

…. moresevere non
Bloody commone coli) need Abx
q
2

amore
Child with bloody diarrhea fever 39c dx : A. Salmonella B. Shigella C. Campylobacter ✅ if no bloody diarrhea and eat from
resturant think about staph no

Sheigella = neurolgical Sx = ‫ﺔ‬¹-‫ﺐ ﻋﺼ‬È‫ﺴ‬±= ‫ﺷﻘﺎوة‬ need abx just supportive

… Bloodydiarrhea
!6 years with sore throat + difficulty swallowing + painful cervical lymph
foodBorne
Bothnodes. the organism =

streptococcus pyogenes scarlet fever

…. S.Pyogenes = Group A streptococcus

! Child with group A strep pharyngitis. What will you do with his brother?

A. Observation ✅ B. Throat culture C. Throat swab for rapid antigen test D. Antibiotics

..

!Child with chronic diarrhea and labs indicative of macrocytic anemia asks which of the following is

important in past giardiasis infection

#Baby with mass in umbilical and developed veslculopustular rash grape like organism

A. coxsackie B. Hsv C. Group A strep✅

best to choose and most accurate

agents with central anticholinergic

1Neuro
activity (e.g. diphenhydramine,

benztropine)

cholinergic
if not in choices choose domperidone

# child took hyoscine butylbromide and metoclopramide for gastroenteritis and fulud the he develop

jerky movement (not sure about the presentation it was wired to me ) what to give ? domperidone

Thid drug improves symptoms of nausea, vomiting, bloating, and feeling of fullness. e

!child starting brief sizer (less than 30 seconds) , EEG (generalized 3-Hz spike-and-wave activity.)

Treatment = ethosuximide ( Absence seizure) Brief loss of consciousness

….
or
Interrupted motion or activity, blank stare, unresponsiveness

usually ceases after 5–20 seconds


Very subtle automatisms (often go unnoticed!): lip smacking, eye fluttering, or head nodding are common

Consciousness returns rapidly, without any impairment

Patient often unaware of interruption


!Child fall from his bed complain of headache and 2time vomit everything normal like physical exam

neurological and he is conscious next? A. close observation B. CT head

… ifvs.miled3times
! An 8 year old boy’s parents complain that he has episodes where he blinks multiple times and becomes

okay after that.. he is conscious and responsive during those episodes. The most likely diagnosis is ,,

A. Tics B. Tourette syndrome. C. Blinking disorder

Addictive found in deep coma and cyanosed what do to?

a-Mechanical ventilation ✅ b-History details from family c-Lavage gastric ‫ﻞ ﻣﻌﺪة‬¹‫ ﻏﺴ‬Ñd c ‫§ﻌ‬

‫ اول ﺷﻲء ھل راح ﻧﺧﻠﻲ اﻟﻣرﯾض وﻧﻣﺳك اﻟﻌﺎﺋﻠﺔ وﻧﺎﺧذ اﻟﮭﺳﺗوري ﺑﺎﻟﺗﻔﺻﯾﯾل واظل اﺳﺎل‬coma ‫ ﻣرﯾض ﺟﺎﻧﺎ ﻣدﻣن وﺷوﻓﻧﺎ وﺟﮭﮫ ازرق و ﻓﻲ‬: ‫ﻟو ﺗﺧﯾﻠﻧﺎ اﻟﻣوﻗف‬

first ABC then you can give antidote .Mechanical ventilation ‫واﺳﺎل واﺗرك اﻟﻣرﯾض ؟ ﻣﻧطﻘﻲ اول ﺷﻲء اﺳوي اﻧﻘذه واﺳوي ﻟﮫ‬

….

!3 y old boy fall down from his bed No LOC But vomiting 3 times + headache Neurological ex > normal

What next ? A-Brain MRI B-Brain Ct C-Close observation✅ D-Neurologist consult


closeeye

!Fracture of the left stylomastoid foramen during delivery of a baby:ons cant

A. Decreased blood supply to the left ear B. Loss of sensation of the left side of face

C. Loss of anterior 2/3 sensation of tongue most accurate answer (cannot close eye) ptosis

… if not in choices choose Loss of anterior 2/3 sensation of tongue

!child unable to feed herself with a spoon , hx of head trauma 10 days ago where is the lesions?

cerebellum

‫ﻘﺪر ﻳﻮازن اﻟﻤﻌﻠﻘﺔ‬v‫ ﻣﺎ‬. .....

pt BMI 18 but think of oneself as obese thus dieting, Dx? A. bulimia‫ﺎﻟﻌﻤﺪ‬- ‫ﺴﺘﻔ¬غ اﻻˇﻞ‬Ó ‫ﻌﺪﻳﻦ‬- í™‫ﺗﺎˇﻞ ﻛﺜ‬

B. anorexia✅

….

1-2months old , full term , diabteic mother birth weight is 4.8kg and no other complications during

delivery absent moroʼs reflex on right side what is the cause: Erbʼs palsy ✅

‫¯ﺲ *ﺸﻮف »ﻠﻤﺔ‬: ((absent hand motor reflex )) bd c ‫ ﻃﻮل‬ä‫ ﻧﻔﻜﺮ ﻋ‬Erbs palsy

absent hand motor reflex === Erbs palsy

......

& Earb’s palsy, happened to macrosomia baby Wight more than 4500&

!unilateral moro's reflex = Erb's palsy

….

!Child has pneumonia+ fever then developed seizures. Treatment:

A- Amoxicillin B- paracetamol C- diazepam

…. - Child develop seizure

IV Lorazepam → Buccal midazolam or Rectal diazepam → Nasal diazepam --> phenytoin --> phenobarbital
Child with febrile illness and seizur

A. IV lorazepam B. Nasal lorazepam. C. Rectal diazepam✅✅

E
‫ ﻻن ﻋﻠﻰ ﻛﻼم‬، ‫ وﻟﻛن طﻠﻊ ﺧطﺎ‬، ‫ھذا ااﺳؤوال ﺣﻠﮫ ﺟﻠوري ﺳﺎﺑﻘﺎ اﻧﮫ ﻧﻌطﯾﮭم ﺑﺎراﺳﯾﺗﺎﻣول = ﻋﻠﻰ اﺳﺎس ﯾﻧزل اﻟﺣرارة وﯾﻌﺎﻟﺞ اﻟﺗﺷﺟﻧﺎت‬

‫ وﻟﯾس‬anticonvultion ‫ ﻓﺎﻟﻌﻼج ﻧﻌطﯾﮭم‬، ‫اﻻﺑﺣﺎث ﺣﺗر ﻟو ﻋطﯾﻧﺎھم ﻋﻼج ﻟﻠﺣرارة اﻟﻲ ﻋﻣﻠت اﻟﺗﺷﺟﻧﺎت ﻣﺎ راح ﯾﺟﯾب ﻓﺎﯾدة‬
iv lorazepam First Line if IV access is not possible Rectal diazepam

Diazepam‫طﯾب اﻟﺳؤوال اﻟﺛﺎﻧﻲ اﯾش اول ﺷﻲء اﺑدءه ﻣﻌﮭم ﺑﺎﻟﻌ


ﻼج ؟‬paracetamol ,
here is no evidence that antipyretics reduce the risk of
subsequent febrile convulsions in at risk children.

Prescription of paracetamol following febrile seizures may provide comfort and symptomatic relief, but

should not be recommended to prevent further febrile convulsions.

plus taking a dose of diazepam (Valium) gel that’s administered rectally. You can be taught to give the

treatment at home if your child has recurrent febrile seizures.

.......

!Continuing >35 minutes Seizure epilepticus Given lorazepam iv what's next? Iv phenytoin
e

phenytion ‫ ﺛم ﻧﺑدء ب دواء‬lorazepam ‫ ﻣﺛل دواء‬diazepam ‫ول ﺷﻲء ﻧﺑدء ب‬

‫ ﻻن‬. ‫ ﻋﻧدﻧﺎ وﻣﺎ ﻛﻔﻰ اﻟوﻗت ﻧﻌﻣل ﻟﮭم اﻛل ﯾﺳﻌﻔﻧﺎ اﻟﻔﯾﻧوا‬zepam ‫ ﺟﻣﯾل ﻋﺷﺎن ﻟﻣﺎ ﯾﺟﻲ زﺑﺎﯾن‬pheno 11 ‫رﺑط = ﺧﺑز اﻟﻔﯾﻧوا‬

2‫ﺳﺎﻧدوﯾﺗﺎش ﺳﮭل‬

.....

Child with fever and active seizure now. TTT? A- Phenytoln B- Paracetamol C- Diazepam ✅

… Generalizedtonicclonic

!Child k/c of grand mal seizure on Depakine comes with breakthrough seizure

what to give initially after intubation on ER presentation: Diazepam

!Pt status epilepticus for 5 min, with iv access, what is the first line:

A. lorazepam B. diazepam rectal C. buccal midazolam

!kid with seizure for more than 5 mins, iv line secured = IV lorazepam

!pediatric pt with tonic clonic seizure before 5 min resolve ,what Rx. to give ( availability of IV access) all

options was anti-epileptic.

%Status epilepticus (more than 5m) > Lorazepam

Generalized tonic clonic > Carbamazepine

Partial tonic clonic > Phenytoin

Absent seizure > Ethosuximide Athetosis


… kinetictype
Dl's

mainlydueto
Fania
Coffs Kerncterus

!Cerebral palsy in kernicterus?

Answer is: Athetoid cerebral palsy or dyskinetic cerebral palsy.!

Kernicterus is a type of brain damage most often seen in babies. It's caused by an extreme buildup of

bilirubin in the brain. Bilirubin is a waste product that's produced when your liver breaks down old red

blood cells so your body can remove them.

..... Kernicterus is caused by unconjugated


hyperbilirubinemia that develops either as a result of

A case of icterus ‫ﺎ ﺟﻮﻧﺪ×ﺲ‬P‫ >=ﻣﻌﻨﺎ‬Kernicterus a hemolytic disease (Rh incompatibility, hereditary

1-ABO incompatibility 2-RH incompatibility spherocytosis, other hemolytic disorders) or because of

… inability of the liver to conjugate bilirubin.

$Fist hand +feet crossed seen in : A. down syndrome B. cerebral palsy✅

6w or month old with unilateral absence of red reflex what to do next? First
fundoscopy

A-Mri brain and optic. B-Funduscopy

!best time to do red reflex examinations =at birth and age of 6 weeks !

….

15 months old baby presented to the clinic with developmental delay, on examination patient was having

spasticity, crossed leg, lower limb were involved more than the upper limb but both were affected, what’s

the type of cerebral palsy he has: A. Quadriplegia ✅. B. Hemiplegia. C. Diplegia. D. Athetoid


Diplegia : involve Upper and lower but lower limb

… involved more than upper limb

$girl with bruises and fecal incontinence after being fully toilet trained: dx sexual abuse✅
very common Q sexual abuse location 6 clock

$Parents came with there child complaining of that there child always blinking on rest and activity on

examination there is no pain and tearing what is the most likely diagnosis? ‫ﻘﻮة‬- ‫ §ﻐﻠﻖ ﻋﻴﻮﻧﻪ‬Ñd c ‫§ﻌ‬

A.conjunctivitis B.dry eye syndrom C.tourette syndrome D. : tics disease ✅

.‫ ﯾﻐﻠق ﻋﯾﻧﮫ ﺑﻘوة‬، ‫ ﺑﻘوة‬tight ‫ = ﻣن‬tics = ‫رﺑط‬

.......

$child came with continued seizure for 5 minutes ,intravenous line is secured, first line management?

A. rectal diazepam B. IV lorazepam ✅

!Child with rapid eye, he is active with parents during the attack= Tic disorder

….

$Child with repetitive eye movements. Otherwise normal no pain nothing. On exam no redness or

tenderness or edema. Dx? Tics syndrome

. 3434. ‫ = ﺟﺎﻟس ﯾﻧظر ﺑﻛل ﻣﻛﺎن ﯾدور ﻋﻠﻰ ﺗﺎﻛﺳﻲ‬repetitive eye movements ‫ = ﺗﺎﻛﺳﻲ‬Tics = ‫رﺑط‬

…..

5-7years old presents with inability to stand or sit unsupported and clumpsy gait and resistance to neck

flexion after chicken pox infection weeks ago:dx: Acute cerebellar ataxia ✅

2 c
‫ ﻫﻮ‬¤
d ‫ اﻟﺘﻮازن ﻟﻠﻤﺨﻴﺦ ا‬bd ‫ﻌﺪوى ﺣﺼﻞ ﻟﻪ ﺧﻠﻞ‬- ‫ﺔ‬-‫ﻌﺪ اﺻﺎ‬- ‫ﺎﻟﺴﺆوال §ﻘﻮﻟﻚ‬- ‫ رﻛﺰ‬celebrum ‫ﻌﺪوى‬- ‫ﺐ اﺻﺎﺑﺘﻪ‬È‫وﻫﻮ ¯ﺴ‬

Acute cerebellar ataxia of childhood is a childhood condition characterized by an unsteady gait, most likely

secondary to an autoimmune of postinfectious cause, drug induced or paraneoplastic

....

Case of abuse, subdural hematoma and retinal hemorrhage , the childʼs mother is mentally retarded but

father is normal = shaken baby syndrome ✅ ‫ﺔ‬Ò&‫ واﺿﺢ اﻧﻪ ﻫﺰة ﻗ‬Ñd c ‫› =§ﻌ‬ c ™ ‫ﺎﻟﻤﺦ و اﻟﻌ‬- ‫ﻒ‬Ò$‫ﻻن ﻗﺎل ﻟﻚ ﻧ‬

....

fetus delivered by Vacuum instrument, has swelling that doesn’t cross the sutures? Cephalohematoma

% Child Head trauma subarachnoid, hge, periorbital edema, bruises and LOC Father said he found her like

this, what to do? Call child protection% Child Abuse

First ABC after the child be stabilized call child protection


%%A fetus delivered by Vacuum instrument, has swelling that doesn’t cross the sutures?

Cephalohematoma %%

.......

c ™ ‫ﻣﻦ اﺳﻤﻬﺎ ﻋﺼﺐ اﻟﻮﺟﻪ =ﻋ‬

Can’t close his eye inone sidewhat nerve affected? -Facial Nerve ✅ ›

...... Duchenne muscular dystrophy (DMD)


- Onset before age 5 years old

Duchenne muscular dystrophy sign? Gowers maneuver ✅ ✔and atrophy starting in the proximal lower limbs (pelvic girdle), later spreading to the upper body and distal areas
- Paresis
- Weak reflexes

- Waddling gait (bilateral Trendelenburg's sign)


- Gower maneuver: To stand up, the patient supports himself on his thighs and uses the hands to essentially "walk" up the

body until reaching a standing position. It is a classic sign in DMD but also occurs in e.g., Becker muscular dystrophy,
dermatomyositis

- Inability to walk by approx. 12 years of age


- Cardiac and respiratory muscle involvement

- Cognitive impairment
Dx:

- Genetic analysis (confirmatory test): detect dystrophin gene mutation

- Muscle biopsy Only performed if genetic analysis is inconclusive


Tx:

- Medical therapy :-
DMD: glucocorticoids (e.g., prednisone)

BMD (Becker muscular dystrophy): Glucocorticoids may be used, although their efficacy is low.
- Supportive therapy :-

Physiotherapy

Orthopedic assistive devices (wheelchair, walkers)


Psychological support

… Ventilation support

child with febrile seizure since 3 days, tonic clonic lasting an hour, he still have fever. what to do? A-give

phenytoin. B diazepam Ö C-Abx D-Paracetamol choose Diazepam if No IV lorfazepam in choices

N.B: Febrile seizure main treatment is to treat underlying cause, but if patient came to you in ER seizuring

then :- If 5 minutes or more --> Diazepam. If < 5 minutes--> Paracetamol.

If non seizuring in ER --> Paracetamol, whatever the duration he seized at home.

Child afraid of going to school, how can the mother deal with the case =

Talk to him about how his favorite super hero would deal with the situation.

!1x2 cm on the forearm since birth, no symptoms =follow up it could be Hemangioma

...

!What improved to decrease the premature baby mortality rate? hypothermia

-Initially presents with upper respiratory tract symptoms (e.g., rhinorrhea), fever, and cough, conjunctivitis

Bronchiolitis the commonest cause of ARDS in children from 6 month to 24 month -Respiratory distress (usually occurs in infants)
Tachypnea, prolonged expiration
most common cause RSV

Nasal flaring, intercostal retractions

treated supportively Cyanosis

upper RTI followed by symptoms of respiratory distress and wheezing in a child < 2 years of age should prompt evaluation for
bronchiolitis!

influenzavirus
occuremainlyinchildren 2Years croup para

ResPdistress

firstfever.gsuffynoseThen cough epiglottis Hib

TxMainlysupportive 1Bronchiolitis Rsv

Case of bronchiolitis (severe) with chest recessions what is the management? -Admit for fluids hydration

and oxygen✅

‫ﻪ ﺟﻔﺎف‬- ‫ ﻻ §ﺤﺼﻞ‬Ñi‫ﺔ ﻟﻠﺠﺴﻢ واﻟﺴﻮاﺋﻞ ﻻن ﻃﻔﻞ ﺣ‬Ò‫و‬íi‫ ﻳﺰود اﻟ‬Ñi‫› ﻋﺸﺎن اﻻ*ﺴﺠﺔ ﻣﺎ ﺗﻤﻮت ﺣ‬ c ™ ‫اﻻوﻛﺴﺠ‬

Oral corticosteroid medications and pounding on the chest to loosen mucus (chest physiotherapy) have

not been shown to be effective treatments for bronchiolitis and are not recommended.

Hospital care

A tiny percentage of children need hospital care to manage their condition. At the hospital, your child may

receive humidified oxygen to maintain sufficient oxygen in the blood, and perhaps fluids through a vein

(intravenously) to prevent dehydration. In severe cases, a tube may be inserted into the windpipe (trachea)

to help the child's breathing.

.........

#Portia pt have tachypnea, runny nose, cough,slightly elevated fever, audible wheezing sound whats is

tho definitive diagnosis: = Nasopharyngeal swabs (case of Bronchiolitis caused by RSV) ✅ pertussisNasopharyng
... aspirate Reminder

!18 months with picture of bronchiolitis, developed several episodes of apnea. What’s the appropriate

management? a. Supportive b. Ventilatory management c. IV methylprednisolone

...
$two cases about bronchiolitis: cause and treatment= RSV. Ttt: supportive rehydration ✅

.....

! pt have arthralgia after viral infection" watery diarrhoea " what is the type of arthritis?! Reactive

..

!Case descripe typlcl symptos of bronchitis / asking about the diagnisis (Chronic productive cough , SOB

Bronchiolitis ) RSV = supportive Reactive arthritis (Reiter syndrome) :-


Features :-

,… - Polyarthritis Acute onset Often asymmetrical with a migratory character


Occurs predominantly in the lower extremities

&Bronchiolitis:---------RSV&respiratory syncytial virus. - Sacroiliitis- Enthesitis- Dactylitis


- Extra-articular symptoms :-

..... Conjunctivitis or iritis- Oral ulcers


Dermatologic manifestations: skin lesions of the glans resembling psoriasis

&Child with Small VSD, asymptotic, Mx: observation and watchful waiting.✅ Symptoms from preceding infection:-
Diarrhea- Urogenital tract symptoms (dysuria, pelvic pain, urethritis, prostatitis
Tx: self-limiting

… RadioFemoral Delay >Coartication of aorta


Arthritis :First line: NSAIDs

!Pediatric with intact radial and reduced femoral, with fhx of htn: Coarctation of the aorta

! best diagnostic for CoA - Coarctation of the aorta for neonate ? A. Us B. Ct C. Echo catheterization
org Coartication of aorta :

… Mechanicalventilation Dx:Best initial test: upper and lower extremity


blood pressure measurement.

$Child need non invasive M.V , Where you will manage him? Echocardiography with doppler (confirmatory test)
Tx: - Initial management: infusion of PGE1

A. Emergent room✅. B. Child ward. ward (not sure of name). C. Infant. D. Intensive care unit (alprostadil)
- Surgery : surgical correction or balloon

N.B:8 months age might be the age. Baby in icu with multiple cong. angioplasty

… PICU Should be performed as soon as possible in critical


cases.

! Child with mild fever and bilateral conjunctivitis and abdominal pain subsided 2 weeks ago now on x-

ray follow up you found bilateral lung infiltration, most common pathogen?
A. Moraxella catarrhalis B. Streptococcus pyogen C. Adenovirus

Bilateral lung infiltration not lobar pneumonia we think about viral

Most commoncause viralcuntunctivit of


if say lobar pneumonia we think bacterial
Adenovirus
and here bilateral conjuctivitis we think also viral

1Pediatric surgery

!2 year old boy with pain over anterior tibial tubercle, diagnosis? A- Osgood Schlatter Syndrome

.. AnteriorkneepainandpointtendernessovertibialtubersityduetoinflammationandAVNlduringjumpingogrunninggftainfbs

!RTA and you find aortic thoracic injuries and splenic abrasion with hypotension what u do ?

A. Thoracic surgery B. Abdominal surgery C. Call center for vascular surgery D. laparotomy

RoadtrafficAccidant

· As your child enters puberty, their growth centers,

that separate bone and cartilage, close, often


#Young boy Just had a growth spurt came with pain in hls leg , dx ? resulting in heel, foot, and ankle pain. With the use

of orthotics, a splint, or a soft cast, you can help to


Ankle sprint = Osgood schlatter if below the knee

prevent pain and injury while strengthening the feet


and ankles during these growth spurts

Debridement and External fixation

but if not in choices choose A


Vit Ddef bca 9PTH 6Phosp

AALA

hypophosp

bphons

! positive rebound tenderness in Macc Barney point (case of appendicitis) the pathophysiology =

peripheral vasoconstriction Peripheral Vasodilation

!Peds 8 yrs old with RLQ pain and rebound tenderness what's confirmatorytest? phosphor

A. US abdomen B. MRI abdomen C. CT abdomen


… primary
PITH 49 J

child having sudden pain at thigh pic of spiral fracture with labs of (PTH Ca) high Secondary N t t N't
A. primary hyperparathyroid ✅. B. secondary. C. parathyroid carcinoma i

...... StonelBonelgroansl psychiatricovertone


Tertiary i 99

!9y old child , PTH high , ca high , came with bone pain tt ?

avoid
B.rehydration + diuretics
thiazide
+ bisphosphonate ✅
dvµ oooo

C.rehydration + bisphosphonate + diuretics + statin


f

......
y
!Child with bowed legs Labs: calcium is high. Phosphate is normal. = familial hypophosphatemia ( not m

nutritional vit D deficiency ) X

O O e O

!Bowing legs, frontal bossing , management: Vit D3

..
r
1Hh4iph O 0 O

VitD Rickets t.ca.pt phosphate

famI Hypophophetemia Rickets bPhosp NcaNpTHTAIKalinephosph


child with rash appeared as vesicles. Some of his other classmates were having the same. He has

immunodeficient brother. Family has concern about their immunodeficient child. What is your action?

A.give oral antiviral B.don’t do anything because this disease is self-limited C. give IVIG ✅✅✅.....

.....

boy came with deep stabbed wound in the anterior right thigh 10 cm in depth. What is your next step?

A.tourniquet B.call vascular surgeon C. direct pressure apply on the wound* ✅

D. directly clamp the femoral artery

….

Child abdominal trauma, investigators show splenic lacerations 2cm w peri-spleen

fluid most appropriate management A. Non OPrative ✅ ‫ﺔ‬¹‫ ﻣﺎ§ﺤﺘﺎج ﻋﻤﻠ‬Ñd c ‫§ﻌ‬ if hemodynamically stable no need for

surgery just conservative

- child fall dawn on his hand ( radial&ulna) fracture,1cm open wound

A.Closed reduction with cast above elbow. B.Closed reduction with cast bellow elbow

C. Innernal fix with cast till elbow. D. surgical debridement& fixation ✅

‫ ﻧﺨﺘﺎر دي‬، ‫ﺎﻧﺔ‬ù‫ﻂ ﻣﺎ ﺗﺤﺮك اﻟﻌﻈﻢ ﻣﻦ ﻣ‬¹‫ ¯ﺴ‬²‫ او ﻛ‬²‫ﻪ ﻛ‬¹‫ ¯ﺲ ﻟﻮ ﻣﺎﻓ‬، ‫ ﺣﺴﺐ اﻻﺷﻌﺔ‬ä‫ﻌﺎ ﻋ‬ú‫ﻫﻮ ﻃ‬

….. open wound : surgical Debridement with external fixation

!Child with humural & ulnar & and un able to move extensors muscle of forearme and hand ? median

Radial V
nerve in cubtal fossa !

….

Child with vascular malformation of lower limb , when to interfere : A- pain✅✅ B- Claudication Heart failure
… Don't circumcised if the neonate has hypospadias because they need this skin in anastomosis

D
! Newborn circumcision , chordee and hooded foreskin + hypospadias ,how to manage?

A.open circumcision B. plastibell circumcision ✅ C.circumcision with *** D.referral to pedia surgeon
For standard hypospadias, the procedure of choice is primary tubularization, and when necessary, incision of the urethral plate. r

%An infant with hypospadias and for circumcision, what’s the procedure?

The surgeon will use a small piece of foreskin to create a tube that

increases the length of the urethra ✅✅

Boys who are born with hypospadias should not be circumcised at birth. The extra tissue of the foreskin

may be needed to repair the hypospadias during surgery. ... During surgery, the surgeon may place a

catheter (tube) in the urethra to make it hold its new shape

….

%Bleeding Circumcision which factor ? 8% ‫ﻂ‬è‫ = ر‬Circumcis= Circumcision= 8 ‫ﻋﺪد ﺣﺮوﻓﻬﺎ‬

......

$Neonate+ prolonged bleeding after circumcision, aptt high, pt, bleeding time and platelet are narmal,

condition is most likely due to deficiency in


Reflected

A. V. B, Vii. C. c Vii ✅factor 8. D. XFactor VIII =Hemophilia A


Viit factors

Bleeding after ciraumcision factor 8 by willeB

Bleeding after umbilical stump> factor 13 Bleedingtimer von

Bleeding after home delivary - Vit K (factor 10)


y

......... Vitamin k Deficiency =Factor X Deficiency

home delivery baby with umbilical bleeding after day 5, dx: Tx: Vitamin K

A- vit k deficiency B- factor X deficiency ✅. C- hemophilia ((umbilical = ❌ fsctor))

.......... .......

infant had bruises on his thigh after delivery at home what is the cause? vit k def

!boy come with gingival hyperatrophy with enlarge and bluish purple friable

gum . What’s vitamin deficiency? A. vitamin A B. vitamin C C. vitamin D D. vitamin E

(Due to vitamin K deficiency bleeding in the newborn) B. factor x deficiency


bleeds out of tooth extraction >Factor VIII or Von willebrand disease

f
in the q they mention +factor 8 that means no defeciency in factor 8 the answer Von willebrand disease

and you should know that von willebrand disease is the most common hereditary bleeding disorders (Autosomal dominant)
PT high , PTT ==prolonged PT means that the blood is taking too long to form a clot.caused by conditions

such as liver disease, vitamin K deficiency, or a coagulation factor deficiency (e.g., factor VII deficiency)

! Boy bleeds out after tooth extraction Then after develop bruises they mention factors Vlll= +ve , Plat=

normal Only slight increase in pt Dx?. von willebrand disease Bleedingtime9

V = III (VON) ‫ﺛﻼث ﺣﺮوف ﻣﻊ ﺛﻼث اي‬

!Baby for circumcision u found urethra midshaft what procedure will u do

A. gomco clamp B. plastibell C. other name can’t recall D. inform surgeon

…..

!Baby for circumcision u found urethra mid shaft what procedure will u do

a. gomco clamp b. plastibell d. inform surgeon✅ Don't circumcised we need that skin

….

pAfebrile NoNIV
! 6 months uncircumcised pediatric UTI how to treat: A. IV ceftriaxone B. IV cefepime C. Oral Ceftriaxone

….
!Months boy Uncircumcised with ggsoramonthsor
….
NAV
fever , labs showed UTI = IV Ceftriaxone✅ cut = ‫ﺳﯿﻒ‬ Z
they will not mention UTI in q they will give you signs and symptoms and Urine analysis then you will think about UTI

• treatment of afebrile UTI if mild—> Oral ceftriaxone


.. • if febrile and severe and there is N/V—> IV Ceftriaxone

&5 y/o boy uncircumcised presented to ER with fever and abdmonial pain, suspected

to have UTI, urine analysis showed: Nitrate positive and high WBC, Which of the following will most likely

indicates he has UTI: A. Leukocytosis. B. Nitrate ✅✅. C. Urine gravity D. Urine RBC
Nitrite not Nitrate

&most likely indicates he has UTI: Nitrate&

….

! 4 months Child with mid-shaft hypospadias, came for circumcision. What u will do?

A- not possible since they will use it for the repair.✅ B- should be delayed.

#Babies who have hypospadias that requires surgery shouldn't be circumcised, because the foreskin may

be for tissue grafts during the operation#


THE sod
!Child circumcised has UTI treated with TMP/SMX and improved, which further should be done?

A- renal ultrasound B- cystourethrography C- reassuring

Pediatric patient presented with abdominal pain and jelly like stools, right quadrant mass (case of

intussusception) what’s the best diagnostic test?

A. Abdominal x-ray. B. Abdominal CT. C. Abdominal U/S. D. Barium enema ‫✅ ﻣﻜﺮر‬

&jelly like stools, = the best diagnostic test = Barium enema&


...
O Dx

GEE fo
stargetsign

two cases of Intussusception, one about next step after stabilizing pt. :

barium enema / radiological reduction✅

Es
... if no air enema or contrast choose barium
- intussusception case, what would you tell the mother?
0
g

A- Shock is most common complication B- Require surgery immediately

C- Recurrence common after surgery✅ D- Enema carried out in case of peritonitis

the probability of recurrence was 100% after the fourth episode of intussusception. After the third

episode, the probability of recurrence and eventual surgery are 68% and 70%,

respectively. Surgical intervention should be considered at the third episode of intussusception.

….

Picture of intussusception : nausea and vomiting Which statement regarding

Diagnosis A. presence of sausage shape in palpation ✅✅ B. Passage of current jelly stool confirm the

diagnosis C. Present in 2% of population

….


best next: ultrasound

if best for intussusception: contrast or pneumatic enema (air) risehislegstothechest

if patient dehydrated the best next step Rehydration first


!Case of baby with intermitted cries that is sever to the point where he is rise his legs and screams for

hours what best nexst step = abdominal ultrasoud !(intussusception )as attacks ofabdominalPain

‫ ﻓﮭو‬، ‫ وﻻ ﺳﻲ ﺗﻲ ﺳﻛﺎن ﻟﻠﺑطن‬، ‫ ﺑﺎﻗﻲ اﻻﺧﺗﯾﺎرات اﻛس راي ﻟﻠﺑطن‬، ‫ ﻟﻔﺗرات ﻣﺗﻘطﻌﺔ‬، ‫ وﯾرﻓﻊ ﻗدﻣﮫ ﻣن ﻗوة اﻻﻟم‬، ‫طﻔل ﯾﺻرخ ﻣن اﻻﻟم‬

US‫ ﻧﺧﺗﺎر طﺑﻌﺎ اﻻراﺳﺎوﻧد‬nexst step ‫ﻗﺎل‬

. . . .. .

!Tt of intussusception in pediatric = hydrostatic enema ! ( Dont choice reduction by ultrasound or

fluroscopic) Air (pneumatic) enema: treatment of choice

Hydrostatic reduction: normal saline (or water-soluble contrast enema)


….

!Abdominal exam sausage shaped mass = Intussusception !

latesign

…….
!intussusception case how you confirm it clinically: A- Sausage mass B- Red currant jelly stool fs

!Intussusception case stable Best initial radiological reduction ( not I.v fluid bez he is stable)

…. T

Child cry when left her hip with mass in upper abdomen what’s Dx: intussusception

!Intussusception child first thing to do = Iv fluid & analgesia✅

! 13 month old with Abdominal tenderness,vomiting,bloody stool,leukocytosis and US doughnut shape.

intussusception
… polyps

% A 3 year old girl with bloody diapers. She has no pain or constipation. diagnosis?
A.Meckel's Diverticulum %% B. Intussusception C. Colon D.Juvenile apresent withinfirsttoyearsoflife

polyps

....... C Theclassic
✅ presentation
Zyl o
ispainless61 Bleeding

! 3 year old girl with bloody diapers. She has no pain or constipation diagnosis

= meckel diverticulum ! D.Juvenile polyps


!target sign by us= Intussusception !

............

!Case of intussuption child very dehydrated what is the next immediate action? Iv fluid !

… initial

!11 months presented with bloody smelling stool how yo confirm dx A.pns Us B. Barium enema

Intussception presentation and q is whats is the initial investigation: = ultrasound abdomin

‫ﺺ ﻫﺬا اﻟﻤﺮض‬¹‫ﺸﺨ‬Ó bd c ‫ﻘﺔ ﺟﺪااا‬¹‫ﻪ ؟ ﻻﻧﻬﺎ اﺳﻬﻞ وا˙ع ودﻗ‬¹‫ اﻟﺴﻮﻧﺎر ﻟ‬,d ‫ ف اول اول اﺷﻌﺔ‬، ‫§ﻈﻬﺮ ﻣﺜﻞ اﻟﺪوﻧﺎت‬

….

!case of intussuception came with clicky pain + doughnuat sign on ultrasound + bloody stool what is

most important step to manage rhis case ?

Ÿ Urgent surgery

Ÿ Nasogastric decomprestion

Ÿ I.v fluid resuction ✅

FI
Ÿ Bariam enema


upperGIseriesisthe
investigation of
choice
valves
in infantswith
suspected Midgut

Distention
..... Midgut vulvulusiBillowsvomiting Hematochezia


Tx NPONOTBroadspectrumAbxsurgery Laddprocedure
$Case of baby 6 days passed stool after birth w constipation wts dx:

A. Meconium ileus B. Hirshpurge C. Volvolus✅



within48 Billoustdistention

to
ails to passMeconium
…. Meco
pass fails
Abdominal distended, vomiting, picture of obstruction. Dx?

A- volvulus ‫ ﺻﺢ ﻻن اﻟﺴﺆوال ﻗﺎل ا*ﺴﺪادد‬B- ischemia. C- UC. D- Crohn


....

!Neonate with bilios vomiting He pass meconiun (Then he pass yellow stool)=>I think Dx?

A-Hirschberg dis B-Mid gut volvulus C-Meconium colitis


...

A child passed meconium within 24 hrs after birth. Two weeks later, child developed bilious vomiting,

abdominal distention and passage of pellet stool. What's the diagnosis?

A. Meconium plug. B. Midgut volvulus✅✅. C. Hirschsprung's disease. D. Intussusception


..... ...... ......


Child with bilious vomiting and pass limited amount stool. 5 days old

A.meconium ileus B. Volvlus ✅. C.Hirshsprung


...............

$baby 3-7 days presented with bilious vomiting , decrease oral intake , this happened after introducing

milk formula He passed meconium after birth and after that yellow stool :

A. Hirshbrung (no bez said pass mecnium at birth so exclud ) B. milk allergy C. Mid gut volvulus✅

….

!I got same picture exactly sigmoid volvulus


…..

b
Coffee bean sign and want the diagnosis? Sigmoid vulvulus thumb sign in GI >bowel ischemia

J
..... thumb sign in neck > Epiglottis

Thumb print sign in abdomen and eant the daignosis ? Bowel ischemia

… Ultrasound dose not have rules to detect intestinal obstruction like x ray

these are signs of obstruction and we should do X ray for intestinal

Abdominal distrnsion, constipation, vomiting. No Xray in choice .Investigation? A- CT obstruction


ï
if no x ray and there is CT choose it

B- US✅ ‫ﺲ راي‬ë‫ء ﻫﻮ ا‬î d ‫ﺎﻟﻄﻮارء اول اول‬- ‫ﻄﻦ ﻓﻌﻼ‬- ‫ﻻن‬


...... ....... ...... Jun dicepruritus feverchills


features of Primary sclerosing cholangitis What to do?

Colonoscopy = Primary sclerosing cholangitis is a chronic cholestatic liver disease. The majority of

patients with PSC have underlying inflammatory bowel disease . Patients with concurrent PSC and IBD have

method of choice for PSC is MRCP
an increased risk of colorectal cancer . TO Symptoms of chronic inflammatory bowel disease, which is frequently associated with PSC the have risk of developing colorectal cancer we usually do

‫ﺎﻟﺘﻬﺎب اﻻﻣﻌﺎء‬- ‫ﻂ‬ú‫ﺪ دا§ﻢ ﻣﺮﺗ‬úı‫ اﻟ‬bd c ‫ ﻫﻮ‬¤d‫ﺸﻔﻮا ﻫﺬا اﻟﻤﺮض ا‬Ëë‫ ﻣﻨﻈﺎر اﻟﻘﻮﻟﻮن واﻟﺠﻮاب ﻫﻮ ا‬bd c ‫ول‬íi‫ﺪ اﻟ”ﻠﺴ‬úı‫ﺶ دﺧﻞ ﻣﺮض اﻟ‬±‫ﺐ ا‬¹‫ﻃ‬
colonoscopy to role out any abnormality

....... ....... ........



Child tripped on a toy and the right leg was trapped within the toy ,and fell on the leg child

complaining of pain ,what type of fracture do you expect:-

-Spiral facture of femur -spiral fracture of tibial ✅ -hairline fracture(not sure) Toddlersfracture

childhoodaccidental
... ...... .. .....

$A child was brought by his parents after he refused to walk and insisted on being spiraltibialfractureAmbulat

children

infantandyoung
Caused twistingwhiletrippingTheypresentwi

by
Femur fracture summary

-less than 6 month >pavlik harness limping 0 refusingtowalk


-6month to 5 years > hip spica


-5 to 11 years > flexible intramedullary nail

-more than 11 years > rigid intramedullary nail

carried always, the parents reported this happened after he was playing and

stepped on a toy and his leg was twisted and fell down = Toddlers =spiral fracture of distal tibia.

….

!Child with X ray of distal radial and ulnar bone fractures = Cast below elbow

‫ﻂ ﻣﻦ اﺳﻤﻬﺎ‬i‫ ر‬distal ‫» ﺗﺤﺖ‬Ö Ñ ‫ﻌ‬v ‫ﺪ‬n‫» |ﻌ‬Ö Ñ ‫ﻌ‬v elbow ‫ﺤﺘﺎج ارﻓﻌﻬﺎ ﻓﻮق‬v‫ة ﻣﺎ‬sz‫ﺪ ﻓﺎﻟﺠﺒ‬n‫ |ﻌ‬¤r‫ ﻻن اﻟ‬elbow .

.... Open reduction and intra medullary nailing

! 6 y.o child with fracture of thigh and 30% angulation. TTT?Hip spica with traction

Giftsylo

!Child with forarm fracture , mx ? closed reduction and cast !Aboveelbow Green stick

e
..

$Distal radial fracture in peds patient (xray shown), partially penetrated the skin (picture). management?

A. internal fixation B. external fixation C. internal fixation with casting above elbow D. internal fixation

with casting below elbow ✅

Child fall from hight presented to you in ER crying, Bleeding from the ear, tympanic membrane bulging

and bleed , imaging confirmed basal skull fracture, The nerve which pass through foramen ovalea injured

whatʼs the function which will be affected ? Mandible nerve , mastication

AnteriorBasilarfracture

‫ﻂ‬è‫ = ر‬Mandible = man


foramen = men = man = (man with men)
T 1 G'Sf rhinorrhea2Raccooneys

posteriorBasilarfracture
...... ..... ....

child with supracondylar fracture, distal pulse not palpable, your management: 1 ESfotorrhea2Homo tympanum

A- K-wires. B- exploratory operation ✅✅. C- hand elevatio 3 HematomaBehindearsBattle


1 sign

..... ..... ... immobilization

twAffenptaabffAngulation
xray of both bone distal forearm fracture, greenstick, your management:

A- closed reduction and cast )*+(exact pic 2 Greater thanAcceptableAngulation

Aboveelbow Close reduction with1 cast


#A 3 y/o fell off his bunk bod and immediately cried, but ho didn't have an apparent loss of

consciousness. Brought to tho hospital and further Investigations show that there l$ no skull fracture.

What is your next action? . Watchful waiting'

prBesttimetoscreen at BirthdAIffweeks

Infant with absent red reflex ( retinoblastoma most likely) What to do:

A-Immediate referral to ophthalmology✅ B-Brain MRI

..

!what is diagnostic for retinoblastoma ? CT


MRI
b of the risk of radiation-induced second cancers in patients with heritable disease
…(CT) is generally avoided in patients with suspected retinoblastoma because
to
%Retinoblastoma on slit lamp examination = urgent referral to ophthalmologist✅✅%
… of retinoblastoma can usually be made based on:
diagnosis

●Ophthalmologic examination under anesthesia (EUA)


●Ocular ultrasonography (B-scan)

●Optical coherence tomography (OCT) – Usually performed during the EUA


●Magnetic resonance imaging (MRI) of the brain and orbits

congenital adrenal hyperplasia 1inheritance

! 17 alpha hydroxylase deficiency= autosomal recessive

...
!child with 17-hydroxylase what type of inheritance?
wait

A. multifactorial B. autosomal recessive C. autosomal dominant D. x linked

...

$Baby with ambiguous genitalia what deficiency: A. 17 hydroxylase B. 21 hydroxylase✅

...‫ ﷼‬21 ‫ ب‬66 ‫ = ھﻣﺑرﺟر‬ambig = ‫اﻣﺑﯾﺞ‬

! inheritance of an ambiguous genitalia= AR (21 hydroxylase deficiency)

AmbigouGenitalia in female 21

AmbigouGenitalia in Male 17

! Pt carry risk of 25% to have genetic dis , What is the type of genetic abnormality ?

A. autosomal recessive B. autosomal dominant

......

! Parents carrier 25% chance of having affected child with "Cystic fibrosis"= autosomal recessive

‫ﻂ‬i‫ =ر‬cystic = ‫ = ﻛ›ﺲ‬receive ‫›ﺲ‬r‫ﻀﺎﻋﺔ وﺣﻄﻴﺘﻬﺎ |ﺎﻟ‬ô‫ اﺳﺘﻠﻤﺖ اﻟ‬:)

.....

!Child with cough,wheezing , recurrent infection, poor feeding and poor weight gain and murmur =Cystic

fibrosis

.... FT

!Inherited of 21 hydroxylase deficiency autosomal recessive. R=2

....

$Congenital adrenal hyperplasia mode of inheritance? AR 21 hydroxylase deficiency

….

!case of Neurofibromatosis type 1 (7 cafe au liat spots, axillary freckles), ask mode of inheritance

e
:autosomal dominant

‫دوﻣﻴﻨﺎت‬. = íc c ‫ ﻣﻦ دوﻣﻴ‬bd c ‫ي »ﺎ‬íi‫ *ﺸ‬.

.......

!What is the type of genetics in pt presented with cafe aule spots dx neurofibromatosis?

A-AD✅ B-AR C-Xlinked AD

.......

!Child with multiple cafe au let spot on his body, his mother mentioned that his relatives also have the

same spots= consul about NF1


....
X
confermdiagnosiswithGenitictesting N

!4 year old boy brought by his mother examination reveals multiple Café au lait spots. The mother says

that “it’s a common birthmark in our family” what is your test action

A. Send her for genetic counseling B. Educate her on NF type 1 C. Confirm diagnosis with genetic studies D.

Reassurance Recurrentinfection Wiskott-Aldrich syndrome triad is? Xlinked recessive

.. eczemaThrobocytopenic rash Dermatitis eczema. Thrombocytopenia(bleeding diathesis)

. Immunodeficiency recurrent pyogenic Infections.(URTI)

!case about wiskott-aldrich( recurrent infections, eczema, 2 healthy sisters, 1 died before the age of 10

months ask about mode of inheritance? x linke recessive

‫ﻂ‬è‫ر‬

Died = X wiskot = ‫ = اﺳﻜﺖ‬X reccurnt infection = ‫ﻞ ﺷﻮي ﻋﺪوى ﻋﺪوى‬- = X


…..

!Case of 14m boy with a history of 4 lung infections, he has to healthy sisters. X-linked

agammaglobulinemia
Recurrentseverepyogenicinfection

…. Brutaglobulinenia

! Parent come with their child who have recurrent chest infection and they have another child who died

from one attack of chest infection ask about Dx ? -X-linked gammaalbuniemia

X-linked agammaglobulinemia (XLA) is a rare genetic disorder that affects the body's ability to fight
infection.

‫ ﻣﺎﻓﻲ اﺣد ﯾﻣوت ﻣﻧﮭﺎ اﻻ اذا اﻟﻣﻧﺎﻋﺔ ﺟدا ﺿﻌﯾﻔﺔ ﻣﺛل ھذه‬chest infection ‫ ﺛﺎﻧﯾﺎ‬x.linked ‫ ﻟذﻟك‬، ‫ﻓﯾﮫ ﻛﻠﻣﺗﯾن اوﻻ اﺧوه ﺗوﻓﻰ ﺑﺳﺑب اﻧﻔﻛﺷن ﯾﻌﻧﻲ وراﺛﺔ‬

completedeficiency mature13lymphocyte
of ........ ....‫اﻟﻣﺗﺎﻻزﻣﺔ ﯾﻛون ﻋﻧدھم ﺟﺳﻣﮭم ﻣﺎﯾﻘدروا ﯾﺣﺎرب اﻟﻌدوى‬

....

!-10 m old infant with pneumococcal infection and repeated infections. His brother died from severe

sepsis. on studies he has few B cells but normal T cells diagnosis=xlinked agammaglobinemia

….

!Case about a male with immunodeficiency has two normal sister andhx of one brother died due to

pneumonia.? X linked immunoglobulin‫!ﻣﻜﺮر‬

....

#Wiskott–Aldrich syndrome has an X-linked recessive pattern characterized by 3 thing : eczema +

thrombocytopenia + immune deficiency or upper resp infection

‫ﺴﻜﻮت‬8‫ ﻣﺘﻼزﻣﺔ و‬bd c ‫ﻌﺾ ﻧﻔﻜﺮ‬- ‫ اذا ﺷﻮﻓﻨﺎ ﺛﻼث »ﻠﻤﺎت ﻣﻊ‬:

eczema + thrombocytopenia +immune deficiency or upper resp infection

‫ ﻣﻊ اﻛزﯾﻣﯾﺎ‬S ‫ = ﺣرف‬wisskott = .‫ ﻣﻊ اﻟﻘﺻﺑﺔ اﻟﮭواﺋﻲ‬o ‫ ﺣرف‬thrombo‫ ﻣﻊ‬T ‫ﺣرف‬

...

%Child with recurrent URTIs , eczema and thrombocytopenia both brother and uncle have the same

condition? Wiskott aldrich!

...... ....
!Wiskot aldreith syndrome= Dermatitis, thrombocytopenia, immunodeficiency E

…..
ADinheritance
$Child with tuberous sclerosis? A. Single gene testing B. Multiple panel gene testing ✅
e

!Tuberous sclerosis in child also his mother have it with TCH1 mutation what to order to child I think?

multiple gene screening

1Endocrine

!What is the recommended screening age for hemoglobin? A. 8 mo B. 12 mo C. 15 mo D. 24 m

..

# eye pigment with green brown =Wilson's disease Kayser Fleisher Greenbrownring
ring

!Mother is concerned about her 5 month old baby that he’s delayed developmentally how would you

relieve her concern? A. baby is waving hi B. baby is setting independently C. baby is holding objects

… amonth Gamonths Reaching

!congintal adrenal hyperplasia , now he is dehydrantion + slight low glucose what will give ? 3 thing (
tx:

normal salin + steroid + glucose ) ! - Glucocorticoid replacement therapy is indicated in all forms of CAH. (Lifelong daily regimen)
- Hydrocortisone in neonates and children
- Prednisolone or dexamethasone in adolescents and adults

… - Steroid stress dosing


-IV Fluid

!High levels of 17-OH progesterone can indicate acongenital adrenal hyperplasia (CAH) =How to manage?

daily hydrocortisone orally

…..

! Female child has several episodes of vomiting and enlarged clitoris on examination. studies are given

and show sodium 120 Low with other electrolyte


2113hydroxylase

imbalance treatment = corticosteroids


first hydration then steroid corticosteroid bsodim

( congenital adrenal hyperplasia case)

CAH crisis > the definitive ttt is hormonal replacement corticosteroids + mineralocortecoids

What is the classic cause of ambiguous genitalia on the Step 2 exam?

Adrenogenital syndrome, also known as congenital adrenal hyperplasia. Ninety percent of cases are

caused by 21-hydroxylase deficiency. Girls present as neonates with ambiguous genitalia. Boys present as

neonates with salt-losing adrenal crisis or as toddlers with precocious sexual development. Patients

with 21-hydroxylase deficiency have salt-wasting (low sodium), hyperkalemia, hypotension, and elevated

17-hydroxyprogesterone. Treat with steroids and intravenous fluids immediately to prevent death.

$ 2 months old develop diarrhea (did not mention how many times or any other information) without

vomiting or any other symptoms ( did not mention the vital signs or the health status of the baby) The

mother concerns about dehydration management ? A. oral rehydration solution ✅ B. IV fluid

… Esifsevereg

!Child was taking oral rehydration solution, present with mild dehydration, what is the reason of his

symptoms? A.lactose intolerance B.fructose intolerance C. glucose intolerance ✅

!Mother came with baby 12 months suffering from recurrent gastritis after introduce normal diet again

according to previous pediatrician give him oral rehydration Now baby came with same feature with mild

dehydration also =Oral rehydration for 24 day then give normal diet ( not for 5 days)

$pedia pt with polyuria , high glucose , what’s next investigation:=. Hb A1C

nmdepletion
… weight loss Hyperglycemia booooopolyuriapolydipsiaNIVVol

q
!Pediatric patient with classical symptoms of DKA + elevated blood glucose . What will you do next?

apt
Urinedipstick ( ‫ﻴﺘﻮن‬ı‫ = ) ﻋﺸﺎن *ﺸﻮف اﻟ‬Dont choice Hba1c bez Q next step! 1serumGlucose2 BMP1 Anions

if came to you acutely in ER unstable with signs and symptoms of DKA then it will bee urine dipstick next step
… 3presence of Ketonlurin dipstick4pH5 Hb Ak

Child lossing 1kg despite he eating and drinking a lot , came dehydrated and irritable What's the

diagnostic test to the reach the diagnosis AlteredmentalstatusLethargy

A) glycosylated hemoglobin B) HLA-DR3 C) urine dipstick ✅✅. D) sorry forgot it the choice Child

!Pediatric presented with hyperglycemia and sx of diabetes In vs RR:60 What is the most important test

to do: Hba1c == Urinalysis ! ( to detect the keton). Hyperventilation

Diabetic mother, how to know if baby will be normal?

A- FBS B- post prandial C- HgA1C ✅ • Type 1 —>screen eye from 3-5 years from the time of diagnosis then annually

… • Type 2 —>screen immediately at diagnosis then annually

!Child diagnosed with T1DM screen eyes how often =after 5 years then annually !

!Pediatric pt recently diagnosed with DM type 1 1 month ago compline on medication when to do

Opthamology screening Immediately =6 y 5yrs

!When do screening for uveitis in SLE patients with (-) ANA. = 6m

… ANA 3 m

child with type 1 DM, what he will use to control the disease? Regular insulin ✅ ✔

!6 years old K/c DM type 1 complain of hypoglycemia best TTT ? decrease Mixture insulin

Rapid ACTING INSULIN ANALOG before meal , no long acting NPH intermediateactinginsuline

/….

!Child didn’t eat anything last 3 h came with severe thirst . Fasting blood glucose 6.3 . Why he is increase

of glucose ? decrease insulin

…..
pfast
acting

!child on glargine + aspart complain of fasting + postprandial hypoglycemia :reduce both

… Long Actinginsuline

%%DKA During the management most important thing to be monitored?cerebral edema%%

….
pens ifhe'snottreated

!DKA treated but still have hypokalemia why ? ( vomting ) ( Dont choice insulin therapy or i.v fluid )

… a mostly due to insulin because it shifting the potassium from extracellular to intracellular
the treatment is the cause
also vomiting cause hypokalemia but not mostly the main cause

$7 y. Newly diagnosed DKA but if there is no insulin choose vomiting

A. go to DM group. B. write management plan in a paper. C. hear what the child said and its concern ✅

......

!child with DKA, PH 7.1 and glucose 20mmol. What is the initial mx step? Iv fluids

..... IV fluid then after 1 hour give insulin

child with Sx of dehydration and lethargy He also has fever. Rapid infusion of normal saline has been

started; after that, the boy start some abnormal movement and went into deep coma which lead the

3.6 5.2

doctors to intubate and transferred to PICU. Hypotensive, Tachycardic, Na 165, K 3.2, Cl 115. What is the

cause behind it?

A.delay in Abx treatment

B. rapid infusion led to cerebral edema ✅✅

C.intraventricular hemorrhage due to the high sodium.*

….

Child with dehydration, depressed anterior fontanel, and decreased skin turgor. What is the percentage of

dehydration? А. 5. В. 10✅. С. 15. D. 20

if asymptomatic> mild> 1-5%

if more > moderate> 6-10 %

if severe (hypotension)> 15 % O G

! in children of diabetic mothers, glucose 12.5% is given in? A. central line B. peripheral line C. NGT

....

!Newborn with hypoglycemia what is the route of 20% dexterous? Central line ( 20 % tacke by central ,

if say 10% priphral =

I =10

.......

!Child came with hypoglycemia what is the infusion rate ? . 10% dextrose + 10ml/hr✅

&Treatment of hypoglycemia &:

• Dextrose given peripherally in D5 and D10 but centrally in D12.5 and D15 and D20.

‫( اﻻﺑرة اﻟﻛﺎﻧﯾوﻻ ﻧرﻛﺑﮭﺎ وﯾن ؟ ھل ﻓﻲ اﻟﯾد وﻻ ﻧرﻛﺑﮭﺎ‬dexstros) ‫ھذا ﻣﻌﻧﺎھﺎ ﻟﻣل ﯾﻛون اﻟرﺿﯾﻊ ﻋﻧده ﻧﻘص ﻣﻣﯾت ﻓﻲ اﻟﺳﻛر وﻧﺑﻐﻰ ﻧﻌطﯾﮫ ﺟﻠوﻛوز‬

‫ ف ﻋﻠﻰ ﺣﺳب اﻟﺟرﻋﺔ اﻟﺟﻠوﻛوز اﻟﻲ ﺑﻧﻌطﯾﮫ ﻟﮭم اذا‬. central line ‫ اﺳﻣﮫ‬femoral ‫( ﯾﻌﻧﻲ ﻋﻧد‬:‫ﻋﻧد ورﯾد ﻛﺑﯾر ﻣﺛل اﻟﻲ ﯾرﻛﺑوﻧﮭﺎ ﺗﺣت اﻟرﻗﺑﺔ او اﻟﻔﺧذ‬

. ‫ اﻻرﻗﺎم ﻣﮭﻣﻣﺔ ﻧﺣﻔظﮭﺎ ﻋﺷﺎن ﻧﺣدد اي ورﯾد‬. ‫ ﻧرﻛب ﻓﻲ ورﯾد اﻟﯾد‬١٠ ‫ او‬٥ ‫ اذا اﻗل‬، ‫ ﻓﻣﺎ ﻓوق ﻧرﻛب ﻟﮭﺎ ﻋﻠﻰ ورﯾد ﻛﺑﯾر‬١٢

• We start ttt of neonatal hypoglycemia by inserting 2ml/kg of D10 *peripherally*

• If no response, persisted hypoglycemia so consider 12.5D through *central line*

• So initially it is peripheral line, not direct central line

....

&Dextrose given peripherally in ===D5+D10 but centrally in ===D12.5 and D15 and D20.&

‫ﻪ‬¹‫ ﻧﻌﻄ‬٢٠ ‫ او‬١٥ ‫ او‬١٢ ‫ ف ﻣﺜﻞ ﻣﺎ اﺗﻔﻘﻨﺎ اذا‬centrall ‫ ف ﻧﺤﺎول ﻧﺤﻔﻆ اﻻرﻗﺎم‬.

central .‫ داﯾﻣﺎ‬١٢ ‫ داﯾم ﯾﻛون ﻓﻲ وﺳط اﻟﺳﺎﻋﺔ وﺑﻌد‬١٢ ‫ = وﺳط واﻟﺳﺎﻋﺔ‬central ‫رﺑط‬

.... ..... .......

#Diabetic kid type I diagnosed since 3 years he is follow g growth with same sex

and age what an annually screening should be done ophthalmology

Diabetic child with growth delay .. what check annually:


celiac
V

A- Ophthalmology ✅ ( dispetic retinopathy) B- Growth hormone

Everyone living with diabetes over the age of 12 will get an invite to a regular eye screening. At first the

screening will be every year.

!When to do ophthalmology screening for child with DM type I

A-lmmediatly B-After one year C-After 3 years D-After 5 years ✅ thenAnnualy

....

DMZ

$newborn with one umbilical artery , what’s the cause: mother with DM$ TPOGV

7 years old child brought to ER with DKA. What is the best to do after ER treatment?

A- Listen to his conserns about DKA and its management B- Supply child and family with a written plan of

care ✅ C- Send the Child to special care services

N.B: They didn't mention history of DM I think It's first diagnosis In this case ) B sure.

….
!18 months girl with asymmetrical breast enlargement other examination normal = Reassure

Abdominal US ( Dont choice C.T pelvic or Brain MRI) (if there is GN Rh stimulation test choice it)

…. its normal Reassure (nothing to do) maybe physiological

if there is milk in breast or other signs and symptoms the answer will not be reassure
!mother complains her daughter is less than her age +parent or one of then short. all lab results normal

except insulin growth hormone is low = growth hormone defincy


constitutional
9 y/o child brought by his mom who’s concerned about his short stature. His friends make fun of Him,

G
Investigations show bone age of 7 years and low insulin like growth factor (IGF-1). How treat him?

… constitutionalA- GH

B- constitutional

Child 9yrs old came with his mother because she thinks he is short stature. mother is short. When hand

bone examiend revealed age of 7 years Investigation All normal including growth hormone. Except insulin

BoneAge usually Normal


like growth hormone was 18 low, What is diagnosis?

A. Chronological B. Failure to thrive C. Constitutional D. Growth hormone deficiency✅

......... Infronwithitted BparenseentAgeafsandff


Tx highcaloriediet
!9 y/o child brought by his mom who’sshort def s tature
concerned about his short stature. His friends make fun of Him,

Investigations show bone age of 7 years and low insulin like growth factor (IGF-1). How would you treat

him? GH (children with GH deficiency , we recommend treating with recombinant GH rather than no

treatment

..... Highcalori intake

! Pt 6 years old with very low weight (25kg), everything is normal in examination and lab, except for low

high calorie diet


IGF-1, what you will do: A. give growth hormone K

..

Child with short stature...parents concerned whether he will remain short in the future as well !?

What is the most important thing in history that would determine your answer

A-sibling hight B- parents hight✅✅ C- wt in relation to his hight and age

...

! Asymmetrical kidneys size on us? A- PCKD .polycystic kidney disease

….

girl 7 years old has pubic hair, developed breast ,ask about which kind of puberty?

• central precocious puberty ✅✅ • ovarian tumor • central lesion • gonadotropin adenoma

Precocious puberty is when the signs of puberty start: before age 7 or 8 in girls. before age 9 in boys.

.... ..... ....

5 yrs female with pubic hair , no clitoromegaly obese, hight above 90 centile ?

DHEA
A- Testosterone. B- Dehydroepiandrosterone Sulfatedehy ✅. C- 17 hydroxylase. D- Lh ? Or fsh ?

I
Dehydroepiandrosterone Sulfatedehy also known as androstenolone, is a male sex hormone (androgen)

that is present in both men and women,

When To Get Tested ?‫ﺎس ﻟﻠﻬﺮﻣﻮن‬¹‫ﻜﺮ ﻧﻌﻤﻞ ﻟﻪ ﻗ‬ú‫ﻠ&غ اﻟﻤ‬ú‫ﺾ ﻋﻨﺪە ﻋﻼﻣﺎت اﻟ‬Ò¬‫ﺔ ﻫﺬا اﻟﻬﺮﻣﻮن ؟ ﻟﻤﺎ §ﻜﻮن ﻣ‬ú‫ﺎس ﻟˆﺴ‬¹‫ ﻧﻌﻤﻞ ﻗ‬Ñi‫ﻣ‬

When a girl or woman has excess facial and body hair (hirsutism), or when shows signs of very early

(precocious) puberty such as deeper voice, pubic hair, or muscle development

….

7 y old female started breast development, and pubic hair start to appear and acne >> dx.

A- ovarian tumor, B- central ✅ C- gonadotropin

Precocious puberty is when the signs of puberty start: before age 7 or 8 in girls. before age 9 in boys

The onset of puberty is normally triggered by the hypothalamus. This area of the brain signals the pituitary

gland،Most commonly, especially in girls, precocious puberty is due to the brain sending signals earlier

than it should

‫ وﺑﻌدﯾن ھﻲ طﻔﻠﺔ‬ovarian tumor ‫ وﻟﻛن اﺑدا ﻣﺎﻟﮫ ﻋﻼﻗﺔ ب‬pitotary ‫ ﻣﻣﻛن ورم ﻓﻲ ﻏدة‬central ‫ ﯾﻌﻧﻲ ﺷﻲء‬brain ‫ﻓﺎﻟﻣﺷﻛﻠﺔ اﻟﺑﻠوغ اﻟﻣﺑﻛر ﯾﻛون ﻣﺷﻛﻠﺔ ﻓﻲ‬

.‫ ﻣﺎﯾﺟﻲ اﻻ ﻣﺗﺎﺧر ﻣﻊ اﻟﻛﺑﺎر ﺑﺎﻟﻌﻣر‬ovarian ‫ﺳرطﺎن‬

..... ..... ....

how you measure the mid hight of child

father hight +mother hight /2 if boy +13 if girl -13

!2 y/o developing breast which case ? premature thelarche !

Tanner adultsize
!Turnner stage 5 breast and pupic hair = Delay causes = consitutional delay!

! 7Y tanner stage 5 (breast, pubic hair,acne) type of puberty? Precocious puberty !

….

!7 year old with pubic hair, no axillary hair, no breast or mensis? Precocious puberty ( Not adrenarche

Bez adrena means > axillary and pubic hair and this case no axilla hair ) !
if just hair >premature adrenarche


if with breast enlargement and acne and change voice >Precocious puberty

!9 y/o boy, her mother concern about short stature, the investigation result the bone density for age 7

years= constitutional ( not Growth hormone)

.....

.. . . .. . .

!male with type hair and dark secrtum = tanner stage 4 ! scrotalskindarker 4

!Boy with pubic hair towards adult distribution and darkening of scrotal skin. Tanner stage? A.II B.IV

C. V
..

!13 years old brought by her mother concerned about her stature,tfHx
BoneAges
patient is normal, on examination, no

signs of breast development and no pubic hair, what is the cause? Constitutional

....

$Mother came with her 9 years old complaining about his height. She's short (didn't specify how short),
Patient is 25kg and 120cm, His bone exams revealed that his actual bone age is 7 years, Diagnosis:

A. Constitutional✅. B. Failure to thrive arms

....

#Case of gonadal agenesls she was 17 years no period mInImal development of breast with axlllary and

pubic hair Outflow obstruction or mullarian agenesis = pelvic US screening confirm MRI
.... orabdominal

#17 years old boy with unilateral gynecomastia: Reassure, it will disappear later

....

! 9 days newborn come with jaundice only in the face not not extended to the rest of the body..

otherwise he is healthy was delivered by NVD with no completing.. and he was breastfeed immediately.
Breastfeeding jaundice appear in the first
what is the cause of his jaundice? breastfeeding jaundice Breast milkJun week.
d ice

.... X Breast milk jaundice appear after


the first week and peaks around 2
weeks.

!44-year-old male married for three years complaining of decreased libido painful tender breasts by

lottery bilaterally breastmilk can be expressed manually he came in for an evaluation of his complaints

physical examination is otherwise normal prolactin level is very high what investigation would you like to

do? A. brain MRI(tumor pitotary) B. Ct scan C. Adrenal levels D. Abdominal us

....

$2 years old her mother noticed development of the breast no other sign of puberty

A. precocious puberty B. premature puberty C. premature breast development ✅

No premature thelarche in the choices

Premature thelarche is the term we use for girls who develop small breasts (often an inch or less across),

typically before the age of 3 years. Girls with premature thelarche do not have other signs of puberty.

.........

4 months on breastfeeding, This is her first baby came with 2 days hx of lethargy constipation, fever,

response weak when light directed to his eyes , cause ?

A. Hypothyroidism B. Infantile botulism C. Guillain barre D. Poliomyelitis

‫ ﺣﺗﻰ ﯾﻛون ﻋﯾﻧﮭم ﻣطﻔﯾﺔ ﻣرة‬botulsim ‫ وھذا ﺑﺎﻟﺿﺑط ﯾﻣﺷﻲ ﻣﻊ‬pupillary paralysis ‫ ﯾﻌﻧﻲ‬light ‫ و ﻣﺎﯾﺳﺗﺟﯾب ﻟل‬fever ‫ﻗﺎﻟك ﻋﻧده‬

botlism‫ اﻟطﻔل اﻣﮫ اﻛﻠﺗﮫ ﻋﺳل ھذا ﯾﺟزم ﻟك ﻣﺋﺔ ﺑﺎﻟﻣﺋﺔ ﻣرض‬baby eat Honey) ‫ وﻟو ﺷوﻓﺗوا ﻛﻠﻣﺔ اﻧﮫ‬ptosis ‫ﯾﻌﻧﻲ‬
.... pupilspoorHeadcontrol
signandsymptoms Lack expression 1
constipation16595 DroolingFloppiness

Nonreactive

of
suckandprolongedfeeding
Hypotonia teye
movements

s fsifNeonete
!25 y/o primigravida , baby flat face , no smile : A- infantile botulism B- congenital hypothyroidism
g
t
….

child has leg abnormal shape and delayed walk, ca high ,alkaline phosphatase high ,normal Pos ?

A- rickets✅✅. B- familial hypophosphatemia ricks. C- renal dystrophy , D- hypophosphatemia


….

$A child (can't recall the age) presented with lower leg long bones angulation. Labs show high CA and

low phosphate. X ray shows distal bone hypertrophy irked inIph


A. rickets ✅. B. familial rekits. C. renal osteodystrophy VitDRicketsbca.ptrHnk phosphate


oooo

ooo
famil HypophophetemiaRicketsb
Phosparca.vn rAikaiinepnosph

.......

$child (can't recall the age) presented with lower leg long bones angulation. Labs show high CA and low

phosphate. X ray shows distal bone hypertrophy: rickets✅. familial rekits. renal osteodystrophy



Baby delivered at home presented 65 days later with rt thigh bruises other exam

unremarkable ( PT high , PTT high , other normal ‫? د‬A. hemorrhagic diseases of newborn✅ Vit K Defeciency (factor X)

….

!Pt 5 days old baby with thigh bruises, home delivery, lab : high pt and ptt Dx: ‫ي‬

A. hemorrhagic disease of newborn. B. abo incompatibility C. Rh incompatibility
‫ ﻣن اﺳﻣﮭﺎ واﺿﺢ‬A ‫ اﺧﺗﺎروا‬bruses ‫… ﺗﺷوﻓون ﺑﺎﻟﺳﯾﻧﺎرﯾوا طﻔﻠﮫ ﺗوه ﻣوﻟود وﺣﺻل ﻟﮫ‬


conjugate bBilirubin isNevermore 20 than
gundiceon firstdayoflifepathologicJundice
1
TSBor 2mgIdl

Jaundice

!5 day with jaundice, mom said that his brother also was same sx in last delivery, what most important

question in history: = Mother blood type.
ABOincompatibilityJuridic onfirstdayoflifeindirectHyperBillrubinemia

… CriglerNajjarsyndrome
findirect Hyperbillrubinemia


child was delivered, and he developed jaundice on a 3rd day. He was treated for physiological ‫ز‬jaundice

but 2 weeks later jaundice became progressive with associated pale stool. Investigations conjugatedBilirubin
I

done and showed: Total bilirubin high, Direct bilirubin high. What's the diagnosis? Bilirubinem
maindirect directHyper

A. ABO incompatibility B. G6PD deficiency C. Rhesus is sensitization D. Biliary atresia ✅✅

Biliary atresia is a condition in infants in which the bile ducts outside and inside the liver are scarred and

blocked. Bile can't flow into the intestine, so bile builds up in the liver and damages it. The damage leads to

scarring, loss of liver tissue and function, and cirrhosis. Jaundiceclaystool
866T 9

…. phototherapy PALP
!8 weeks with president jaundice not relive by photophobia , with elevate ALP = Biliary atresia ✅!
darkurine

I
‫ وﻻن طﻔﻠﮫ ھذا اﻟﻣرض ﻏﺎﻟﺑﺎ‬jundice , ‫ ﻓﻲ اﻟدم ﻓﯾظﮭر اﻟﻣرﯾض اﺻﻔر ﯾﻌﻧﻲ ﻋﻧده‬billuropin ‫ ﻣﺎﯾﻘدر ﯾﻌﺑرھﺎ ﻓﯾﺗراﻛم‬bile ‫ﻟﻣﺎ ﯾﺣﺻل ﺿﯾق ﻓﻲ اﻟﻘﻧوات ال‬
‫ﯾﻛون ﻣﻌﮭم‬

. . .. ... .

Infant 2 weeks old. On examination he is jaundice, has large fontanel, cold extremities + hypotonia + large

toung to his mouth. What’s likely diagnosis?


A- Galactosemia. B- Bitouriceye. C_congenital hypothyroidism✅


hyperbilirubinemia, the raised with _congenital hypothyroidism = jundice


!large fontanel, cold extremities + hypotonia + large toung to his mouth + jundice

(hyperbilirubinemia)== Typical sign of congenital hypothyroidism!


.......

!2 month old mother notice bulging tongue , dry mouth, constipation , fhx of autoimmune dis , tSH high

22 , T3 low T4 low what mx: A. Give 1 month levothyroxine. B. give life long levothyroxine. C. repeat in 1

month

7Therapy for both acquired and congenital hypothyroidism consists of lifelong treatment with

levothyroxine (L-thyroxine) and regular check-ups to monitor disease activity.

... Hypotonia

!Baby 2 mo old with jaundice, floppy, protruding tongue there is positive family history of autoimmune,

labs show low thyroid hormone What is the treatment Thyroxine for life.

.........

!Newborn what is the SINGLE investigation you must to do? Thyroid function test!

‫ﺎ ﺗﻪ‬n‫ﻪ واﻋﻄﻴﻨﺎ ﻟﻪ ﻋﻼج راح §ﻄﻠﻊ ﻣﺘﺨﻠﻒ ﻃﻮل ﺣ‬¹‫ﺪ وﻣﺎ ﻟﺤﻘﻨﺎ ﻋﻠ‬Ò‫ اﻟﺜﺎﻳﺮو‬bd c ‫ ﻻااازم اي ﻣﻮﻟﻮد ﻧﻌﻤﻞ ﻟﻪ وﻇﻌﺌﻒ اﻟﻐﺪة ﻻن ﻟﻮ »ﺎن ﻋﻨﺪە اﻧﺨﻔﺎض‬.

‫ﺐ‬È‫ﺎﺧﺘﺼﺎر اﻟﺴ‬- ‫ﻫﺬا‬. ‫ﻞ ﻧﻌﻤﻠﻪ‬¹‫ﻪ ﻫﺬا اﻫﻢ ﺗﺤﻠ‬¹‫ ﻋﺮﻓﺘﻮا ﻟ‬.

.... .. .... 3 at Birth

%Newborn developed jaundice in first 12 hours , labs Low hb “9” Which test to order prior

A. HB electrophoresis. B. G6pd deficiency test. C. Fragility test✅% D. Tyrosine kinase

......... .........X X usuallyNorrudHbatBirth

%%Child with jaundice investigation low he, high total bilirubin, high indirect bilirubin,positive direct and

indirect comp test what is diagnosis = A.Spherocytosis B. Autoimmune hemolytic anaemia ✅✅

….. G vecoomb
!11 years old with with jaundice Lab test: Increase indirect bilirubin Increased total bilirubin Increase all

the lft ? A- obstructice B- gilbert C- autoimmune hepatitis directBilirubin

... x G
gqgpf.MN
Routine laboratory tests are usually normal in patients with Gilbert
the
Hemolysisoccurbeforeitenters

syndrome, except for unconjugated hyperbilirubinemia

!What indicate hemolysis ? A. increase conjugated B. Increase unconjugated Hepatocytelwherecunlugationoccu

7in extravascular hemolysis plasma levels of unconjugated bilirubin increase because the hepatocytes

cannot process the excess bilirubin.

….

!5d /o baby be jaundice asked about important q ask in Hx ??

A. APGAR score or B. mother transfused blood C. blood group ✅ ! Rh as more accurate

….

!16/ old with fever and RUQ pain and jaundice, No lab or radiology available:

A. Reassure and send home

B. Emergency consult a surgeon on phone

C. ask for lab results & tell him to come back tomorrow❌

D. admit to hospital & evaluate✅ Charcot triad of Ascending cholangitis (fever-RUQ pain-jaundice)

Answer is: B, CHECK NOTES

Note: should be admitted and investigate since its risk of septic shock.Then IV abx

followed by ERCP

……..

Pedia 4 days with jaundice and his brother had the same thing, Direct bilirubin and total bilirubin was high

what is the diagnosis: choledochal cyst

....

!Child 12 h , have jaundice , HCT high , pic of spherocytosis , what is the test u will do ? osmotic fragile

..........

$Pedia 4 days with jaundice and his brother had the same thing Direct bilirubin and total bilirubin was

high = $biliary atresia

........

!A newborn got jaundice and the mother mentioned that the other children had to get plasma

exchange or blood transfusion after the birth what is the important inv to do ? Mother blood group .

....... incompatibility RH

Exchangetransfusion

!8 weeks old baby with jaundice, not responding to phototherapy, what is the cause?
AdirectBilirubin

A. ABO incompatibility B. billiary atresia ✅

indirect ‫ﻂ‬i‫ = ر‬photo = ‫ﺮ ﺻﻮرة‬õ‫ = ﺗﺼ‬Biliri = ‫ﻣﻦ‬By = ‫» اﺧﺬ ﻟﻚ ﺻﻮرة‬Ö Ñ ‫ ﺧﻠﻴ‬Ê ø


Ö ‫ﻞ ﻣﺎ ﺗﻤ‬ô‫ ﻗ‬.

.........

! Pregnant with Rh-negative blood type her baby have Rh-positive blood type present with jaundice ask

about Pathophysiology =

autoantibodies against fetal RBCs (Mother’s antibodies attack fetus RBCs)

....

● Child k/c of SCA with pic chest X-ray? A. Acute chest syndrome

Lungopacitication
… Gallstonesbeenin sea

!Boy complaining of RUQ pain, he had a history of URTI couple of days ago, cbc shows low hgb and

increased retics, smear shows target cells and inclusionbodies, dx? SCA

e

!Sickle cell anemia child hb 3 severe pallor and long history given what mostly causes this:

a. Parvovirus B19✅ b. CMV. Aplastic crisis everything is low > triggered by parvovirus b19

%%SCD patient came with crises, What’s the appropriate next step to reach diagnosis (to differentiate

between aplastic and splenic sequestration?A. Reticulocytes B. CXR. C. US


Dactylitis

!Patient came with sudden pain in the hands and feet what’s the most likely diagnosis? SCD

! Boy complaining of RUQ pain, he had a history of URTI couple of days ago, cbc shows low hgb and

increased retics, smear shows target cellsand inclusion bodies, dx? SCD

Acutechestsyndrome

Prevent ACS in SCA ? hydroxyurea increaseHbg f

..

!Pic of CXR of rt lobe consolidation With long scenario of SCD pt presented with chest pain and dyspnea

and back pain, What is the Dx? Acute chest syndrome

....

!What is the medication will prevent these symptoms(Acute chest syndrome ) for the prev Q?

Hydroxyurea

....

SCD young patient came to the ER with toxic looking, enlarged liver andbspleen, drop in Hgb?-

sequestration crisis
.........

!A patient with acute chest syndrome and upper/lower limbs vaso occlusive crisis, what’s the effective

drug proven to reduce the frequency of painful crisis? Hydroxyurea!

... .

#Child with SCD most Important long tami treatment: Hydroxyurea

#Child with sickle cell anemia presented with shortness of breath and chest pain on

is the best initial step in A.lv fluid and analgesic✅

$6yr Sickler with fatigability for 2days. found anemic Spleen 6 Cm below costal margin

A. multiple transfusion B. Splenectomy✅ but hydration and transfusion first we will not do splenectomy from the first time
The role of splenectomy in major attacks is well established. We
ABC
first hydration and blood transfusion

… advocate splenectomy if the child develops one major attack


%patient with a decrease in all cell line WBC, Hgh, and plt (labs) asking about dx?
first
if recurrent we will choose B- splenectomy

Aplastic anemia%%

SCA child with hepatomegaly and splenomegaly 6cm below costal margin with pallor and fatigue for 10 days. Hb was 50( normal was more
than 100 i think) direct and indireIt bilirubin were high Normal platelets

A) regular blood transfusion B) splenectomy C) hydroxyurea D) fluids and analgesia


!Case of child studies given microcytic anemia he has high Hba2 on electrophoresis what type of anemia

Beta thalassemia minor

✴ Hg A2 higher in minor b thalassemia

✴HgF in major thalassemia

skeletaldeformaties ‫ﻂ =ﺣﺮف‬è‫ ر‬F = ‫ة‬í™‫= »ﺎرﺛﺔ ﻛﺒ‬major

Growthretardation

...
f Highforehead

12M fatigue, failure to thrive, stunt growth, large forehead or something?


prominant
zygomaticBoneandMaxilla

A. beta-thalassemia ✅ B.alpha-thalassemia C. SCA D. IDA

!Child lab showed microcytic and hypochoromic anemia with reticulocyte count high (2%) , ferritin

normal, and his 2 siblings have the same presentation what is the diagnosis ?

A. Sickle cell disease B. IDA C. Alpha Thalassemia traitif there is in choices Beta thalssemia choose it
..

!Child pale and lethargic, with no specific S/s , labs shows only microcytic anemia (low hb,lowMCV) what

is the diagnosis?A. thalassemia trait B. lead poisoning C. SCD

….
....

!pale child came with MCV : 68 + Lowe plattlet + low ferritin and have 2 siblings with same condition =

Thalasemia minor ( Dont choice iron defincy anemia)

‫ﻠﺔ وﻋﻨﺪە اﻟﺼﻔﺎااﺋﺢ‬ù‫ﻪ ﻧﻔﺲ اﻟﻤﺸ‬è‫ وﻫﻨﺎ ﻗﺎل اﻧﻪ ﻋﻨﺪە اﻗﺎر‬، ‫ة‬í™‫ ﻻن اﻟﺨﻼ§ﺎ اﻟﺤﻤﺮاء ﺗﻜﻮن ﺻﻐ‬، ‫ﺎ ﻧﻘﺺ اﻟﺤﺪ§ﺪ‬¹‫ﻤ‬¹‫ ﻣﻌﻬﺎ اﻧ‬C d Ö § ‫ﺎ‬¹‫ﻤ‬¹‫اﻟﺜﻼﺳ‬
c
‫ﻤﺎ‬¹‫ﺴ‬D‫ﺎ ﺛﻼ‬ú‫ ﻓﻐﺎﻟ‬، ‫ﻌﺾ‬- ‫ ﻧﺎزﻟﺔ واﻟﺤﺪ§ﺪد اﻻﺛﻨ™› ﻣﻊ‬.

......

!Child ingest nutritional supplement then black stool = iron

....

!child ingest iron tablets and come with symptoms= IV deferoxamine

$child 4 years old loss of weight for long time , most comon cause in this age :

A. lymphoma. B. leukema✅✅ (2_8 years), C. retinoplastoma. D. neuroplastoma

! Child eating paper = IDA!

….

! 58 days baby have G6PD they give lab value low HB level : what are the causes of this:

A. anemia of chronic disease. B. hemolytic anemia

..

! 2 yo girl with diarrhea and dehydration , splenomegaly , Hb is low , direct and indirect coomb is +ve , Dx

? A. spherocytosis B. fanconi anemia D. G6PD C. Autoimmune hemolytic anemia✅ .

..

! Ptn came for routine F/U Lab show: High RBC low HB Low MVC Normal Reticulocyte A-SCA B-G6PD C-

Anemia of chronic disease D-hemolytic anemia

Female had son with SCA. Remarried and for screening : A. Parental B.Mother C. father ✅

! Can be cured by splenectomy?

A. scd B. a thalassemia C. b thalassemia Answer is: ITP, Hereditary spherocytosis

! What is treated after splenectomy?

A. Alpha thalassemia, B. beta thalassemia, C. sickle cell trait D. ITP

..

!Case of child studies given microcytic anemia he has high Hba2 on electrophoresis what typeanemia ?

beta thalassemia minor a

...

!Baby born a few days ago at home presented with multiple bruises, diagnosis?

A. Von Willebrand. B. neonatal hemorrhage dis.✅✅٣

factorX VitKdef
%%A child with ALL came to the ER with febrile neutropenia, management? All septic workup with IV

ABx%%

...... .....

% A young boy came with hemarthrosis asking about dx? Hemophilia%

..

!child with hemarthrosis = Hemophilia

!Direct and indirect coombs test are positive: immune hemolytic anemia

!Classic ring-shaped/headphone-shaped trophozoites are seen in case of Malaria = Plasmodium

falciparum infection

!Pt with pallor (anemia) with splenomegaly Lab: high retic. Blood smear: microspherocytes ++

Anisocytosis + Dx? A. sickle cell anemia. B. hereditary spherocytosis

‫ﻢ‬Ë‫ ﻣﻦ اﺳﻤﻬﺎ ﻗﺎﻟ‬microspherocyte .

occiputlow
prominant setearsMicrognathiaclenchedfist

Syndrome vs'D


2
! prominence occipital, rocker bottom feet , cardiac = Edward’s syndrome

!Baby born with many Edward syndrome feature (rocker foot, etc) genetic

testing asking about possible diagnosis based on dysmorphic feature:. Edward

trimester

Test quadrabletestinthesecond

IAfp t3HCG test riot tinhibinA

mandible
Edward syndrome trisomy 18 small

y
prominent occipital -microcephaly - low set ear-cleft palate and lips -broad nose - microaganthia
- rocker bottom foot -horse shoe kidney - heart abnormality (TOF-VSD-ASD)

..... DX: in 2nd trimester quadruple test

Decrease free estriol ,AFP, inhibin A and BHCG


Prader willi syndrome (microdeletion at 15q11-q13)

- muscular hypotonia - Obesity -hyperphagia - short stature - cryptorchidism-


hypogonadism-facial dysmorphia-premature adrenarch -behavioral problem

Tx:
! long case of child doesn’t control his hungry : .Prader-Willi syndrome Caloric restriction

substitution of growth hormone and sex hormone


….

- Obese child mother complaining of hyperphagia on examination he had dysmorphic features +

hypotonia + ascended testes. What is the most likely diagnosis? -Prada

classic sign of Prader-Willi syndrome is a constant craving for food, resulting in rapid weight gain, starting

around age 2 years. Constant hunger leads to eating often and consuming large portions

….. sz‫ﻴ‬È‫ﻞ ﻛﺜ‬Í‫ﻞ ا‬n‫ =ﺑﺮاد =ﺑﺮادة اﻟﺜﻼﺟﺔ –ﺸ‬sz‫ﻞ ﻛﺜ‬Í‫ﻂ =ﻓﺠﺎﻋﺔ =ا‬i‫ ر‬9:;,<%,

……………..

!Child eats alot and he is obese with undecended testis , facial malformation and cleft plalte? pradder

willi syndrome

….

2cases Turner syndrome (one with primary amenorrhea and other phenotypical features and second one

presenting with short stature and the typical phenotype) ✅

c ™ ‫ ﻟﻬﺎ »ﻠﻤﺘ‬:
‫ ﻣﺘﻼزﻣﺔ‬Turner syndrom ‫› اﺳﺎﺳ‚ﺘﺎن‬

1_amenohhria 2_ short stature í™‫ﻃﻮﻟﻪ ﺻﻐ‬

........

! 17 years old hypertensive not menstruating yet at clinic by her parents ,she is short stature , short

neck Most appropriate diagnosis:- turner ✅

… Broad flat

....... NasalBridge

Parents are worried about the hight of their child on examinations the child look normal with deprsed

nose and short neck and large tongue what is the cause of his short stature ?

Trisomy 21
A.constitutional B. pubertal C. syndromale ✅✅

! Tall thin child patients (above the 95th percintile) and has flexible joints and pectus excavatum

diagnosis? A-turner == short B-marfan == Tall + Thin C-klinefelter D-pertman mostcommon As

ArsD vsD

…. AD VSD A

down syndrome: most common cardiac anomaly associated with DS?: -Endocardial Cushion Defect ✅

Endocardial cushion defects, more commonly known as atrioventricular (AV) canal

.. ...... .....

% Down syndrome endocrine association = Hypothyroidism = %

‫ = داون =اﺳﻔﻞ‬hypo

. . . .. . . ..

Common valvular heart disease in down syndrome? Endocardial cushion defect .(AVSD)

child with Down syndrome came with fixed S2, ejection systolic murmur and enlarged ventricles. What is

the most likely diagnosis? A.VSD B. AVSD ✅✅

….

$ Chart type for down syndrome? A. down syndrome growth chart

$Pic down syndrome?Trisomy 21$

….

!Low incidence in down syndrome: Mosaicism

‫ اﻟﻤﻮز ل اﺳﻔﻞ‬²ï‫ﻂ =ﻣﻮز = ﻧﻘ‬è‫ ر‬dowen

.........

.. a o
# investigation of down syndrome?high Bhcg, high inhipin, low AFP, low estradiol.

Mother 27 years I think had Down syndrome baby what Increases hor risk for

having another baby with Down syndrome = A. age B. father chromosome C. mother chromosomes

….

Infant months of age died , they took hx from the parents ( the infant was preterm, problem with lungs ,

parents are heavy smokers and he was sleep with them ) what is the cause of death ?

A- acute respiratory failure B- sudden death infant syndrome✅✅ C-o2 insufficiency

Sudden infant death syndrome (SIDS) is the unexplained death, usually during sleep, of a seemingly healthy

baby less than a year old. SIDS is sometimes known as crib death because the infants often die in their

cribs. Although the cause is unknown, it appears that SIDS might be associated with defects in the portion

of an infant's brain that controls breathing and arousal from sleep.

….

2months old infant was found dead by his mother. Mom said he was okay without any obvious thing. On

examination, no signs of fractures, bruises, or abuse. What is the important part of history to be asked?

A. social history (think about sudden infant death syndrome >> smoking) ✅

B.history of medication used by the mother during pregnancy

omEf
,….

Which of the following decrease RDS incident the most? A. parents not smoking near their infant✅

..

Baby 2 month died in his bed no other significant problems what is important in ?

a-Allergy ‫ﻌﺪوە‬úË‫ اﺳ‬Ñd c ‫ ﺗﻮە ﻣﻮﻟﻮد §ﻌ‬b-Maternal drug used in pregnancy ‫ﺐ ﻣﻮت ﻣﻔﺎجء‬È‫ﺴ‬±‫ ﻣﺎ‬C. Social hx✅ ‫§ﻤﻜﻦ‬

......

! 15 year complaining she doesn’t have menstruation yes on examination she is short with short neck

and posterior hair = Turner syndrome

.....

-coarctation of the aorta associated with a. Turner✅ b. down

..

-Girl with short stature webbed neck.. diagnosis a. Turner✅ b. familial

....

Dowd

!Clinical finding in pt with turner:


A. hypertonia + ankle clonus B. fold skin at nape of neck C. single palm crease y

!17 years old medically free brought to Gynecology clinic by her mother with

history of no menstruation. On examination there was low hairline, high BP and short stature. Both

mother and father were having short stature at her age.

What is the most likely diagnosis? Turner

$13 years old boy presented with cushing syndrome symptoms such as central obesity and striaLab test :

high cortisone in night and salivary ACTH What next steps: MRI pituitary✅. CT abdomen Aadrenaladenoma bACTH

If there’s high dose dexamethasone suppression of ACTH or CRH stimulation test choose it > its the confirmatoy
.......

! child present with central obesity + moon face + striat investigation? ACTH stimulations test

if there is 24 h urnie cortisol its best initial and more specific choose it

$Signs of turner syndrome=webbed neck.

........

2 Best

!Best lab to dx Turner syndrome= karyotype if there is = FSH choice it .


an

… rend or
ad T S

x15 y/o girl came with her mother complaining of never menstruated .. p/E ..

short stature + short neck + hypertension Parent stature below average Most likely dx?

A-Hypothyroidism B-Hypo pituitarism C-Turnur syndrome D-Familial short stature


.. O 0

aaron

Those so
!Case of digeorge syndrome( facial features, recurrent infection
give before Dx? all live vaccines
and tetany) ask about vaccines not to

....

!8 years old boy don't understand in school and have many involuntary movements, he is aggressive ..

Dx = There was no ADHA in choices

Note: DD: Tourette syndrome,if with communication impaired or Lesch-Nyhan Syndrome.

.....

$Dtap contraindications?

A. Seizure B. Encephalopathy within one week that couldn't be attributed to another cause ✅✅

! Malnutrition of african boy with Acitis or edema A. Kwashiorkor B. malesmas C. muscle atrophy

.....

!Child in poor area with central edema Muscle wasting = Kwashiorkor

‫ﻀﺎوي داﺋﺮي‬¹‫ﻠﻬﺎ ﺑ‬ù‫ﺐ »ﻠﻬﺎ ﺷ‬ë‫ﻮا‬ı‫ﺐ =اﻟ‬ë‫ =ﻛﻮا‬í™‫ﻂ =ﻛﻮاﺷﻜ‬è‫ = ر‬Edema .

....

!Pt child with brittle hair and abdominal destination diagnosed with kwashiorkor cause? Protein

malnutrition

.. Kwashiorkor is a form of severe


protein malnutrition characterized by

! Pt with ascites and edema, severe muscle wasting, hyperkeratosis, lab show: edema and an enlarged liver with fatty

slightly decreased glucose, significantly decreased albumin, your Dx: infiltrates. It is caused by sufficient
calorie intake, but with insufficient

A. severe protein deficiency (kwashiorkor) B. severe carbs deficiency (marasmus) protein consumption, which

IT distinguishes it from marasmus.


Kwashiorkor cases occur in areas of

!Malnutrition of african boy with Atrophy everywhere = marasmus famine or poor food supply.

… T

#Muscle wasting low protein = marasmus if no central edema Deficiency ofallNeutrition

....

!mumps in child most affect = parotid gland!

most common affected organ in pediatric with mumps: A- testes. B- parotid ✅✅

........ ...... ....‫ اﻛل = ﺑﺎروﺗﯾد ﻗرﯾب ﻣن اﻻﺳﻧﺎن ﻟﻠﮭﺿم‬mum = ‫ ﻣﺎم‬: ‫رﺑط‬

1Tumor

......

#4 year old boy most common cancer == leukemias MetaphysisotlongBon

…. IDistaltemuregproximattib

!child has swelling and redness above knee ( femur)=osteosarcoma

… painswelling tender

!Orthopedic tumor in pedia there is No limitation of movement? osteosarcoma

$Which lymph node indicate malignancy ? A. Cervical B. Supraclavicular✅ C. Submandibular

......N.B: gastric cancer

BycanceroncegBreastLung
secreted
ggggord

pPTHrP Hypercalcemia
!lung Ca, sudden onset of back pain, neurological Ex is normal ,ttt? Steroid then MRI

!Pt diagnosed with small cell lung cance, presented with dehydration, serum osmo low, urine osmo

high. ‫ ﻫﺬە اﻋﺮاض اﻟﺠﻔﺎف‬. Ttt:

Ox
Hypertonicsaline

A. 5% dextrose B. Normal saline C. Hypotonic saline SIAD

fluidRestriction 9 salt intake

saline symptomatic
Hypertonic

WilmstumorlaniridlGumalformation mental

WAGR
retardation

05Bestintialdiagnosis

#Flank mass on children = wilms (nephroblastoma ) Citideterminemetastases


(Wilms' tumor is a rare kidney cancer that primarily affects children. Also known as nephroblastoma, it's

↳most common intra abdominal malignancy in Peds


the most common cancer of the kidneys in children.) ↳WAGR Syndrome: wilms, Anirida, GU anomalies (cryptochridism), and mental retardation


↳displaced, Never cross midline
↳mets to lungs (hemoptysis)

!abdominal mass with lung nodule = wilms!(most ↳Hematuria common metastis wilms to lung )

Child presented to the ER with fever and abdominal pain. After a fall 1 day ago, the mother noticed

abdominal distention mainly on the right side. On examination he is pale and hypertensive , your dx ?

A. Liver contusion. B. Pyelonephritis C. Wilm's tumor. D. Neuroblastoma ↳ fever

…..
↳ Failure to thrive
↳ Loss of appetite & Vomiting

! mother while showering her sone noticed abdominal mass = ↳ HTN


↳ cross midline

if central = neuroblastoma If flank = Wilms tumor ↳ mets to eye "Racoon eyes"

.....

!Pt boy with abdominal mass noticed by mother while she was bathing him, on examination healthy boy

with rt. Flank mass 1st Q what is the investigation: A. US B. CT C. MRI

!2nd Q what is the dx: A. wilms tumor B. neuroblastoma


initial
505

... Best GT
!Woman was bathing her child and noticed a mass in his flank, which of the following invx is most

appropriate:

A. Abdominal radiography B. Abdominal CT C. Abdominal MRI D. Abdominal radiography and

ultrasonography

primaryhyperparathyroidism
....

!Boy came to ER has erosions bone in the hand and in the lab has high Ca, phosphate What the

treatment you will give ? Hydration + biophosphanate!

… Bisphosphate e.g. Pamidronate + zoledronate or zoledronic acid



Taq Dadenoma

1Milestones
Management of 1ry hyperparathyroidism
* if only hypercalcemia without Bone changes or osteoporosis = IV fluid + furosemide

*Bone changes or osteoporosis= IV fluid + furosemide+Bisphosphate e.g. Pamidronate +


zoledronate or zoledronic acid

! Child jump on 2 feet , what age in years? A. 1 B. 2 C. 3✅

… Holdhisheadsteadily

! 4 month old baby which of the following he can do= head lag

!Baby crawl and site and grasp things but unable to do pincer grasp: 7

Ñd Ö ‫ﻂ =§ﺤ‬è‫ = ر‬crawl = Ñd Ö ‫ﺪﻳﻦ »ﺎﻧﻪ §ﺤ‬¹‫ اﻟ‬, d í™‫ اﻟﺬراع اﻟﺼﻐ‬äd‫ﻠﻪ ﻟﻤﺎ ﻧﺨ‬ù‫ ﺷ‬7 ‫ رﻗﻢ‬.

.........

!child remove cloth by himself and say dada and tell stories = 4 years !

Child enters the dr’s clinic play w ball through it to doctor , catch a ball ,draw line, puts books together ?

A. 9m. B. 12m. C. 15m. D. 18m✅ ‫ﺣﻞ ﺟﻠﻮري‬

Child enters the dr’s clinic says “Hi”, mother gives him doll. Then he feeds the doll with milk bottle. Mother

moves his head then he says “No”. Mother does something and then he imitates her What’s the age?

A. 12 month. B. 15 month. C. 18 month✅. D. 24 month

&Child can say 3 words sentence == 3 years!

...... ....

!Baby speech understandable to stranger 75% ,says three word sentences = 3 years

!if 3 words > 15 months

&All reflex disappears after 4-6 months except stePPing at 2 months.&

….

! age the sucking reflex disappear? 6 m

#Sit without support = 6 month =‫ﻢ‬¹‫ﻞ اﻟﺮﻗﻢ ﻃﻔﻞ ﺟﺎﻟﺲ وﻇﻬﺮە ﻣﺴﺘﻘ‬ù‫ = ﻣﻦ ﺷ‬٦

Developmental milestones q: 5 words, hop on one leg ?

A. 48 m✅✅. B. 36 m. C. 24 m. D. 60 m

& hop in one leg is( 48m = 4 years)&

‫ﺔ واﻟﻔﺨﺬ‬ú‫ اﻟﺮﻛ‬،‫ اﻟﺴﺎق‬،‫ اﻟﻘﺪم‬، ‫ﺎء‬¹‫ اﺷ‬4 ‫اﻟﻘﺪم اﻟﻮﺣﺪة ﺗﺘﻜﻮن ﻣﻦ‬: ‫ﻂ‬è‫ر‬

. . . . . .. . . .. .

Ride tricycle can not draw square= 3 years

! Can say 3 sentence words can't climb up the stairs can run , milestone = 3y !‫ﺗﺎﻛﺪ أﺗﻮﻗﻊ ﺣﻠﮭﺎ ﺧﻄﺎ‬

….

!4 month milestone:

A. Sit without support B. Head controlC. Start trying to crawl D. Follow objects

...

$Can support his head, smile, follows objects with eyes...= 4 mo

...
4 months old boy what you can expect to his age?

A-try to crawl. B-smile. C-sit without support. D- fix his head

c ™ ‫ﺎء ﻋﻴﻨ‬¹‫ ـﻊ اﺷ‬è‫ﻪ ار‬¹‫ﺖ رأﺳﺔ ﻻن اﻟﺮاس ﻓ‬úE‫ ـﻊ ﺷﻬﻮر ﻳ‬è‫ار‬


‫ﻢ‬¹‫ ﺷﻬﻮر ﻻن ﺳﺘﺔ »ﺎﻧﻪ ﻃﻔﻞ ﻇﻬﺮە ﻣﺴﺘﻘ‬٦ ‫ﺖ ﻇﻬﺮە ﻟﻤﺎ ﻋﻤﺮە‬úE‫ ﺑ‚ﻨﻤﺎ ﻳ‬، ‫› واﻧﻒ وﻓﻢ‬

...

$Baby can move and fix his head when he in prone position , smile, reache object

A. 2m. В. 4m✅. С. 6m. D. 9 m

...

$Smile but not reach objects?. 8weaks✅

...

c ™ ‫ﻦ =ﺷﻔﺘ‬Ò¬‫ ﺷﻬ‬.
$ Child smile which month= 2 Month$ ›

...

! Child can run , stair , can’t use 3 word together age ? 18 month !

....

What is the age expected of ride tricycle? A. 2 B. 3 years✅ C. 4 D. 5

.....

8 Child feed the doll but can't us spoon what his age ? A. 12 months B. 15months C. 18 months ✅ D. 24

.......
Child can jump by two foots, Tell 2 words in sequences

A. 2 years ✅ B. 3 years C. 4years D. 5yeas

..........

!Child 4y old == 1/ can speak clearly. 2/ can draw squares and cycles !

Baby says dada, pincer grasp, walk by furniture, pulls himself to standing

position, sit without support A. 12 months B. 10 month C. 9month ✅. D. 8month

......

Child enters the dr’s clinic says “Hi”, mother gives him doll. Then he feeds the doll with milk bottle.

Mother moves his head then he says “No”. Mother does something and then he imitates her What’s the

age? 18 mo

...... .....

!can sit in his own, stand alone, walk by the furniture, say word of repetitive consonant sounds like

"dada" can hold objects between his thumb and index finger = 10 month .

‫ﻂ =ﻟﻤﺎ ﻧﻀﻢ اﺻﺒﻊ‬è‫ ر‬thump and index ‫ﻞ‬ù‫ §ﻈﻬﺮ »ﺎﻧﻪ ﺷ‬O ، ‫ وﻻﻧﻪ‬stand alone 10= ‫ ﺻﻔﺮ ورﻗﻢ واﺣﺪ‬، ‫ »ﺎﻧﻪ رﻗﻢ واﺣﺪ‬.

......

Baby can walk and say 2 to 3 words rather than mama dada he crawls the stairs age?A-1 ‫ﺎ اول‬-‫ﺎ‬-= ‫ﺳﻨﺔ =ﻣﺎﻣﺎ‬

ï
‫ﺎة ﻧﺘﻌﻠﻤﻬﺎ‬¹‫ﺎﻟﺤ‬- ‫ء‬î
d B-2 C-3 D-4

!baby which is 4 years old and can hop on one leg but can't stay still for 10 seconds, can draw a square,

has a friend, can tell a story the mom is worried about the developmental milestones?

A. The baby is normal no need for referral

B. He has a gross motor delay and fine motor delay

C. He has language and speech delay

$3 years old with normal milestones to his age except that he doesn't know how to

speak in future tense and can't say 3-word sentence, what would you tell the mom

A. reassure. B. he has a delay in speech only but other milestones are fine

Answer is: refer to ENT for audiogram

6-Question asking about best time to do red reflex examinations

A-All age B-Before school C-6 months and above D_ at birth and age of 6 weeks.✅✅

…..

!Same as this pic ask which reflexA. rooting B. suckling

....

#Rooting reflex when disappearing? 4m

! Pic of baby and physician hand. He was taping the left side of mouth When the primitive reflexes

disappear? A. 2 m B. 4m C. 6 m D. 12

!stepping reflex ===

! child can run towards you and follow 2 step commands= How old? 2years

...

!A child with pincer grip but cant put pallets in a bottle. How old? 8 month

...

! child whose 3 years old can talk to other children and strangers understand 75% of his speech. What

kind of delay? Or non? no delay

....

!A 4 year old child with a morning limb and knee arthritis and (-) ANA. When should see the

ophthalmologist to check for uveitis? 6months

.....

$what’s the time to say fever of unknown origin in pedia, per day : A. 7. B. 14✅ C. 21 D. 28

١٤ =٢ ‫ = ﺳﺑﻊ ﺣروف ﻧﺿرﺑﮭﺎ ب‬unknown = ‫رﺑط‬

….

checkPage85 87

Vaccine JtableofCItoeachvacciney

!newborn needs vaccine and his sister died of immunodeficiency disease. What should you do?

evaluation by immunology team

..

!newborn has a brother who died from immunodeficiency, regarding his vaccination? Defer all until

immunology result

..

! Child allergic to eggs. What vaccine should be avoided? Yellow fever

!Child has a brother who died from severe infection came for vaccines which will u defer till knowing

immunity status? A. varicellan B. influenza C. PCV

..... prodromes

#12 y child with mild fever f and vehicular rash in chest , trunk upper lower which test has higher dx

value : varicella zoster igm

.. serology'Igs

!Which vaccine is contraindicated in child with egg allergy?

A. MMR B. Pneumococcal C. Polio D. Influenza


yellowfever
..

!Baby present with with tea color urine with sore throat +140/80 Urine analysis done ( tea color, WBC,

19ANephropathy creatinineforken
erythrocyte ) next = creatinine.

damage
! Steroid dependent nephrotic syndrome patient needs vaccine?

A. wait 6 month after stopping the steroid B_wait 1 month after stopping the steroids

...

!Female has crohn's on biological and azathioprine, what about her baby vaccinations?

A. Delay All 6 months✅. B. Delay Live 12 months


*Postponing vaccinations*⏰

.... Delay live 3 months " Patient on low-dose systemic steroids for less than 14

!What is the absolute contraindication for DtaP vaccine:


days > *give him all vaccines.*

A. seizure (just like that)


" " Patient on high dose systemic steroids for more than
14 days> *delay all live vaccines for 1 month.*

B. hx of encephalopathy within 7 days following previous dose of DtaP " " " Patient on other immunosupresive drugs > *delay

... live vaccines for 3 months.*

!SCD patient took blood transfusion when should you give the vaccines? Varicella
Patient on biologics cytokine inhibitors > *Delay MMR &

A. give All vaccines B. delay 3 months C. delay 6 months

6 months.*

....
Patient received IVIG > delay *MMR & Varicella for 8
months.*

# Patient received blood transfusion: *delay MMR &

! DTP immunization expires in? varicella for :-*

Packed RBC’s: *3m*

A. 10 years B. 7years Whole blood: *6m*

Plasma & Platelets: *7m*

$ children has family members of with primary or


secondary immune deficiency: *give him all exept OPV.*


and

vision

eyelidgswelling erythemaReduced
...

! Baby missed vaccines 18/24.. 2 days swollen eye pain.. Low eye acuity.. What's next most appropriate..

A. Give vaccine.. B. Mri brain to see hemorrhage . C. CT for extend of disease stuff like that..

Answer is: C, ORBITAL CELLULITIS. proptosis ophthalmoplegia

….

#Hepatitis a vaccine for slckles who transfer blood = Give as scheduled ✅

!10 years old with abdominal pain and lab show high liver enzyme and indirect bilirubin.. diagnosis ?

A. Viral hepatitis B. Gilbert 9indirectBilirubin

otherwiseNormallabs
… Gfdirect Bilirubinb

!child with "reccurnt infection" with brother did due to septic shock what will give = Dont give live

vaccine !

. . .. course at least 2 3months

Child with nephrotic syndrome on steroid p give vaccine air steroid m i onth
for 3-6wks or 6 months has vaccine ?
A. Give thev accine B. Stop the steroids C. 3month and give appointment✅ D. Noneedtotreat

….
!Baby 3 hours of life. His brother died of immune condition =

A.give BCG B. Do not give BCG and only give hepatitis vaccine ✅ C.Give immunoglobulin

D.Book an appointment and review condition thoroughly


Ï c

d ‫ﻜﻮن ﺧﻼص ﻣﺎﻟﻬﻮدا‬¹‫ ﻟﻮ ﻋﺪى وﻗﺘﻬﺎ ﺑ‬Ùd Ö ‫ﺐ‬¹‫ﺴ™› ﻫ‬ë‫ ﻻن ﻓﺎ‬Ùd Ö ‫ﻦ اﻋﺘﻘﺪ ﺣﻠﻪ‬ı‫ﻘﺎ دي وﻟ‬-‫ﻣﺘﻜﺮر ﻫﺬا اﻟﺴﺆوال وﺗﻚ ﺣﻠﻪ ﺳﺎ‬

........

!Vaccination at age of school (6years) = Dtap , MMR, OPV, Varicella

.......

!Vaccines of 6 yrs old child (pre-school) in saudi arabia = OPV,MMR, dtap, varicella

!what to do for child came for vaccines but he on antibiotics course? DONT delay till finishing his

antibiotics.
giveHim
most kids, taking antibiotics for a mild illness (like an ear infection) shouldn't keep them from getting

their vaccinations on schedule. Antibiotics do not interfere with the ingredients in vaccines or cause a bad

reaction in a child who has just been vaccinated.

.......

!Neonate was born the mother says she had a baby died due to probably immune deficiency and ask

about vaccines for the new baby?

A.Defer all vaccines until immuno results show B.Order the vaccines and follow up with immunology

C. Take hep B only ✅✅ D.Take all vaccines as usual

!2 year child didn't complete his vaccination present with fever, unable to swallow , dysphasia , enlarged

tonsils , your dx ?. Diphtheria ✅✅

!Mother brought her Newborn to vaccination and she said his sister died at age 3yrs immune-

compromised disease What accurate ? A. give BCGA B. Don’t give live vaccination

C. not give and refer to immunologicalteam✅

….

$Pregnant negative varicella antibodies= A. IViG B. give vaccine, C. avoid exposure ✅

….
!Born of baby with brother die due to Immunocompromised

A.Order vaccine and follow with immunity team B.Defer vaccine until immune team evaluate ✅
.....

$Vaccine avoid in low immune? Live g g g f's


chickenpox in immunocompromised

exposureprophylaxisfor
post

f passiveimmunization VEE
1
patient

a similar case happened at school weeks ago, the parents were concerned about another

immunodeficient child at home what to do with him ?

A. Injection af specific immunoglobulin to the immunodeficient child✅

B. Start antiviral therapy to immunodeficient child

C. Vaccination. (Contraindication live vaccin) .

D.Reassure cause the condition is self limiting


inhibitor Waitfor 6

month
…. Biologiccytokine

$Pregnant lady on chemo what vaccine not to give to child after delivery

A. do not give the child vaccine until he is 1 year old

B. Start vaccination at the age of 6 months✅‫ ﺗﺮﺟﻊ ﻟﻪ اﻟﻤﻨﺎﻋﺔ )ﺣﻞ ﺟﻠﻮري‬Ñi‫ ﺷﻬﻮر ﺣ‬٦ ‫ﻠﺔ ﻋﻨﺪە ﻓﻨˆﺘﻈﺮ‬¹‫(ﻻن اﻟﻤﻨﺎﻋﺔ ﻗﻠ‬

C. Vaccinalions given at 2 month Live attenuated viral vaccines are 

..........

- measles, mumps, rubella
$Vaccines of 4 months baby: -varicella, zoster

hepb, dtp,Hib,MMRx month

D -BCG
hepb, dtp, Hib, PCV ✅ -yellow fever

hepb, dtp, hepA, x PCV -rotavirus,

18month -influenza (intranasal).


............

$(immunodeficiency ) What vaccine contraindication = vareclla$

$2 year sickle cell anemia took one dose of hepatitis A vaccine and came for second dose but she had

history of blood transfusion before 3 weeks due to vasoocclusive crisis what you will do :

A. Give the second dose✅ B. Give it after 3M C. Give it after 6M D. Check the hepA antibodies

!6 months with diarrhea ONLY, no vomiting or fever = vaccinations?Give all.

‫ﺎﺧﺬ اﻟﻠﻘﺎح‬v ‫ﻤﻨﻊ اﻟﻄﻔﻞ‬v‫ﻋﺎدي اﻻﺳﻬﺎل ﻟﺤﺎﻟﻪ ﻣﺎ‬.

.....

Child presented with vesicular lesions all over body Parents are concerned about another child who is

immunocompromised and contacted wih his sick brother give = special IVIG for immunocompromised

patients.

c ™ è&‫ﻪ اﻣﻴﻮن ﺟﻠ‬¹‫ ﻃﻔﻞ ﻋﻨﺪە ﺿﻌﻒ ﻣﻨﺎﻋﺔ ﻧﻌﻄ‬immungloplin = immuncompromise.


‫› =ﻣﻦ اﺳﻤﻬﺎ‬

....

!1 YO vaccination : OPV, MMR, PCV, varicella meningococcal

)>‫ﺑس( ﻋﯾﺑﮭم ﺻوﺗﮫ اﻟﻌﺎﻟﻲ = ﺑس )اﺳﻛت‬Pcv ( ،) ‫ﻛﻠﻣﺔ )أوﺑﺎ ﻟﻠطﻔل‬Opv ، ‫ اﻟطﻔل ﻋﻣره ﺳﻧﺔ ﯾﻧطق ﻣﺎﻣﺎ‬، ‫ ﻣن ﻣﺎﻣﺎ‬M ‫رﺑط = ﺣرف ال‬

….

vaccine to give in 2mo old baby? IPV hbv HIB PCV rota DtaP ‫»ﻠﻬﻢ‬

! 4 months old vaccine =IPV, HBV, Hib, DTaP, PCV

…. live

!Mother crohns and take biologic drugs and azathioprine what regard baby vaccinations? delayed all

6 months
a

‫» ﺗﺮﺟﻊ ﻣﻨﺎﻋﺘﻪ‬t‫ ﺷﻬﻮر ﺣ‬٦ ‫ اﻻ |ﻌﺪ‬Ô Ñ z ‫ﻪ ﻓﺎ¸ﺴﻴ‬n‫ﻂ اﻟﻤﻨﺎﻋﺔ ﻓﻤﺎ ﻳﻨﻔﻊ اﻋﻄ‬ôð‫ﺬا ﻳ‬P= ‫ﺮاﻳﻦ‬iµ‫ﺜ‬Ã‫دواء از‬

…..

!Immunodeficient baby, What vaccine should you avoid: Varicella

….

! Boy with throat thrush, fever and weak Which vaccine will you not give = Varicella

A. PCV. B. Varicella✅. C. IPV

% What vaccine contraindicated in immunocompromised patients ? live vaccines%

child in vaccination clinic. His brother has immunodeficiency and died. Which vaccine to defer until this

child seen by allergy/immunology unit? Varicella

proptosisorbitalcellulitis


p e
!Child missed 12 m and 18m vaccination came with bulging eyes and diplopia what to do = CT to see

infection infiltrate

..

!Mother on biological drug: Delay live vaccine 6m

!mother is not vaccinating her child what to do? -Explain and clear the myth

child , his brother died of immunodeficiency, what vaccinations to hold until confirm immunity of the

child: Varicella )*+(live attenuated)✅✅

!Child K/C BA, didn't take his vaccines, parents refused vaccines cause it is harming their child = Explain

!child with no vaccinnation , now he has ( pharyngitis + cervical lymphoadenopathy = Diphyheria !

‫ﻂ‬Ò‫ = ر‬pharyngitis = Diphyheria

. . .. . . .

#2yoars old with fever cough SOB her mother confirms that she had all vaccines

without any mIssIng what is the causative organism

A. strep pneumoniae✅ B. H.Influenza C. Aspergillus pneumonia D. Klebsiella

.....
of
mostcommoncause pneumonia
!A newborn baby just delivered, mother states that she had a baby that died of immunodeficiency, whats

the protocol regarding vaccination of this baby in saudi arabia

A. don't give bcg vaccine B. give and observe in the clinic after 1 week

! New born to mother on adalimumab, regarding vaccine:

A. give all B. dely for 2 months C. delay live attenuated 6 month D. delay live attenuated 12 month

! child came for vaccination had tonsillitis and lethargy since 4 days was started on antibiotics and is

now better what will u do== a. defer 2 weeks b. vaccinate✅ c. all except dtap

.. .

!-Child has brother who died from severe infection came for vaccines which will

u defer till knowing immunity status a. varicella b. influenza c. PCV

! baby was born in 35 wk, parents asking about vaccines?

A. Delay them to 2 months B. Don’t give live vaccines C. Give all ✅✅

! Case of pediatric patient known to have Seizure, came with syndromic feature hypotonia large head

asking for vaccinations: A. Give all B. Stop all live vaccine C. Stop Dtap✅

Contraindications to DTaP unstable neurologic disorder such as uncontrolled seizures .

….

!Vaccine contraindicated with pregnancy, severe illness? Varicella

….

! 2month infant GA at birth was 27 weeks 1.7 kg and spend 2 weeks in nursery. presented now to the

clinic for first time doing good and gaining weight appropriately best action:

A. delay all vaccines 2 months B. give all vaccines same dose C. give all vaccines half dose

D. dtap instead of Dtap (something like this)

....

! Child scheduled for vaccines, mother mention old baby was having septic shock and died after vaccines,

which vaccines will delay: A. varicella B. injectable poly


T

!most common minor side effects of routine DTP vaccination?

A. injectable site erythema. B. generalized pain C. fever 38.3. D. vomiting

!Father refused to vaccinate his son. What to do?try to counsel him by benifits of vaccine

unvaccinated kid and his dad said he doesn’t believe in vaccinations, what to do:

A.Inform child protection B. Talk to the father about the importance of vaccinations ✅

!baby with diarrhia give vaccin or not give ? Its okay u can give all vaccine , Diarrhia not contraind !


CI if hx of encephalopathy in 7daysAbsolute or see
#Regardingonly

iurerelative
dtap vaccines , Mother present with her 6 month, she reported that after the 4months

e
vaccines her baby developed rash ,fever ,What is the precaution ?

Give prophylaxis antipyretic and antihistamines ✅

…….

Respiratory

weeks old baby with strong cough and 2 episodes of him losing consciousness. on exam there is

intercostal retractions. 02 sat was 90 . What do you do? pHasNorde


Bronchiolitis

A. Inhaled steroids B. Ventilator support C. Antibiotics D. B2 agonist ✅

...... Txsupportive

AscitesEnlargedliverespleenEdema VD

$Pt with tachypnea orthopnea.. (Symptoms and signs) of Rt sided heart failure on lab investigations Liver

enzymes are very high abnormal ? A. Heart failure✅✅ B. Renal failure C. Liver failure

….

$Newborn after CS have respiratory symptoms , CXR shows fluid in the horizontal fissure , what’s the dx

:A. acute distress syndrome B. meconium aspiration C. Transient tachypnea of the newborn (TTN)✅

...........

! Horizonal line in x ray of infant = Transient tachypnea of newborn


seen
deliveredby
or
innewborn 35Wklfullterm

… preterm

!Baby born 27 weeks after 30 mins started having tachypnea and gruntingsz‫ﺷﺨ‬

most common cause?

a. RDS✅ b. TTN c. Meconium aspiration

Respiratory distress syndrome (RDS) is when the neonate has difficulty breathing faced by preterm

infants and is directly related to structurally immature and surfactant deficient lungs

.....

! 30-min aged baby preterm of 27 weeks, developed tachypnea and grunting , Dx:

A. Respiratory distress syndrome B. TTNn C. pulmonary embolism D. meconium aspiration

...

Newborn withnasal flaringandsubcostal retraction? -respiratory distress syndrome

c ™ ‫ اول ﻣﺎ *ﺸﻮف ﻫﺬە اﻟ”ﻠﻤﺘ‬، ‫س‬íi‫ﺴ‬±‫ ﻫﺬە واﺿﺤﺔ اﻋﺮاض د‬.


‫ ﻃﻮل ﻧﺨﺘﺎر اﻟﻤﺘﻼزﻣﺔ ﻫﺬە‬ä‫› ﻋ‬

... ..... .....

%%Patient known case of SMA (spinal myotonic atrophy), develops respiratory muscle fatigue and needs

intubation , mother refuses as one of her children died of the same condition what will you do?

A-consult ethical committee B-intubate her ✅✅ ‫ ﻻن ﺣﺎﻟﺘﻬﺎ ﻃﺎرﺋﺔ ﻣﺎ§ﺤﺘﺎج اﺻﻼ ﻣﻮاﻓﻘﺔ‬%%

. . . .. . .

Transient

! Another case with term baby, that show *Horizontal fissure* on X- ray: TTNtachypnan
of

....... Newborn

!Baby with x ray show ( TTN ) + symptoms of pneumonia + Lap high Neutrophils Ttt= Oral amoxicillin 7

day✅

....

!Term baby with Respiratory distress after CS Delivery CXR : fluid in horizontal fissure = Transient

tachypnea of the newborn TTN.

….

% 7 year old Child ingested 20 tablet baby aspirin ,what you expect regarding acid base balance ?

Respiratory alkalosis > metabolic acidosis.% A. Respiratory alkalosis > respiratory acidosis

B. Metabolic alkalosis > respiratory acidosis. C. Respiratory alkalosis > metabolic acidosis.✅✅✅

D. Metabolic acidosis > respiratory alkalosis. www

.....
Is

metacidosis
!child was in picnic‫ﺔ‬P‫ ﻧﺰ‬sudden he devlop respiratory symptom = causes = lung collap( due to forgin

body) !

….

$A 2-hr old baby = What intervention would minimize disability in the first 6 hours?

A. Respiratory support✅ B. Mild hypothermia


term

… repost
!A newly born presents with meconium aspiration. He was stabilized by intubation and given IV

inotropic fluid. After that he developed respiratory distress. His preductal O2 sat was 92% and post ductal
O2 sat was 83%. What is the next step in managing the patient? nitric oxide Txsurfactant
treatment

lavagea
surfactant

..... dit
PalmH I
support
ventilator
TN ventolatlon : A-ER observation
severnitric
oxidea

!3m baby with respiratory distress when to admit for non invasive

B-pedia ward C-pICU

!2 months boy with bronchiolitis On ( non invasive mechanical ventilation) need admission... A.

Pediatric ward B. neonatal care unit C. Pediatric ICU D. ER

....

! 2 months baby with Bronchiolitis need non invasive respiratory support wher to keep the baby ?

PICU

....

# 18 m k/c of cerebral palsy presents with respiratory compromise Then put under MV Given ABG ,Pco2

normal,Po2 low ,PH 7.3 Ask what type of respiratory failure? A. hypoxic B. Hypercapnic✅ C. hypocapnic

Answer is:B since he is under MV

Neurogenic brainstem, respiratory center Neurogenic will cause hypercanic type ll

.....

! Child with Cerebral palsy on mechanical ventilation , readings, what type of respiratory failure:

A. hypoxic RF B. hypercapnic RF D.Neurogenic Hypercapnic


Ñ t ‫ﻓﻤﻨﻄ‬

cant solve it ‫ ﻻن ﻋﻨﺪە‬cerebral pulsy ÉÖ ‫ ﺧﻠﻞ‬brain ‫ﺴˆﺐ ﻧﻴﻮروا‬ı ‫ اﻟﺮﺋﺔ ﺗﺘﺎﺛﺮ‬Û Ö


....

without ABG
!CP baby on mechanical ventilation ABG ( all within normal even normal Co2 + normal o2 ) Type of

respiratory failure ? Neurogenic ( not hypoxia or hypercapnia)‫ﯾﺣﺗﺎج ﻧﺗﺎﻛد ﻣن اﻟﺣل‬

.....

!18 months old with cerebral palsy had episodes of vomiting and unable to breathe. Then he was moved

to ICU. He was on 100% FIO2. ABG showed NORMAL PH AND CO2 but LOW O2. what is the diagnosis?

hypoxic Rispiratory failler .

....
Hypocophia

! young girl presented with palpitation and hypercapnia. Father mentioned issues with schools and

lead Hypocapnia it

exams. What is the diagnosis? Hyperventilation syndrome

.... to

!Child with pneumonia hemolysis + positive agglutination test, what is the organisation

A- sterpt P B-moraxella C-mycoplasma D- H influenza


Mycoplasmapneumonia 1
subclinicalHemolytic

... Anemia

Atypical

pneumonia
multiform
1
2 coldagglutinins IgM

papularrash erthyma

! child vaccinated up to date has high grade fever and cough, x ray shows bilateral infiltration , and

auscultation bilateral crepitation more in rt lower lobe, what is diagnosis

A - viral infection B- atypical pneumonia C- sterpt pneumonia

.. Middlelope

Boy came with respiratory infection symptoms, by auscultation: crackle, by precaution: stony dullness

direct under the crackles sound, Diagnosis? A. Pneumothorax B.pleural effusion C. Collapsed lung

... duetopneumonia

!neonate presenting after 2 days of delivery with history of seizure and hypertonic extremities, most

likely

....

!Decrease mortality in first few hours after delivery of hypoxic ischemic encephalopathy baby=Mild

hypothermia

...

!Most beneficial action in the first 6 hours for the baby in the previous Q is?

Total body hypothermia or total body cooling

....

!Pre term baby diagnosed with hypoxic ischemic encephalopathy (HIE) what's the risk factor : Low birth

weight less than 1500 gm

... MNeonete

!child came with seizure and they give CSF value " all normal " Dx? Hypoxic ischemic encephalopathy

...

!Mother had obstructed and difficult labor, she gave birth to a child who wasn't crying and cynosed. His

ABG: pH 6.9, HC03 7, 02 8. What's the diagnosis? A. Hypoxic ischemic encephalopathy ✅

....

! 6y old child k/c of asthma came with asthma exacerbation , lab showed RR = 7/minutes , Hypercapnia

best initial ? A. IV MG B. Intubation C. Albuterol D. Theophylline

.....
Al Butera SABA
$pt on ventolin not improved, what to add ? A.plow
dose
ICS✅. B. Increase dose of ventolin

….

$Infant diagnosis with VSD present with symptoms and sign of heart failure ( hepatomegaly, SOB,

Cardiomegaly ) what's the next step in management?

oxygen
a) Admitted to ICU for cardiac b) catheterization c) Give diuretics ✅

‫ﺪ وﻫﺬ‬úı‫ اﻟ‬bd c ‫ﻘﺔ ﻧﻔﺲ و ﺗﻀﺨﻢ‬¹‫ §ﻘﻮﻟﻚ ﻋﻨﺪە ﺿ‬، ‫ﺐ‬Òíi‫ﺎﻟ‬- ‫˝ ﺧﻄﻮات‬
‫ﻪ ﻣﺪرات ﻟﻠﺒﻮل‬¹‫ﺐ ﻓﺸﻞ اﻟﻘﻠﺐ ﻧﻌﻄ‬È‫ اﻟﺮﺋﺔ ¯ﺴ‬ä‫ﺐ ﺗﺮاˇﻢ اﻟﺴﻮاﺋﻞ ﻋ‬È‫ا ¯ﺴ‬furosemide ï
d ‫ﻧﻤ‬

….

$ Mother brought her child for a check up, he is asthmatic on inhaled ventolin, she told the doctor he is

having minimum 2 cough attacks per week, and she came to the ER 2 times last month. Management?

A. continue same management B. Increased dose of Ventolin C. Inhaled corticosteroids

….

!Patient with recurrent dyspnea attack due to inhaling dust or perfumes, on examination has inspiratory

wheeze. Asks about initial management: Ventolin Neb

..

Asthmatic patient on inhaled corticosteroid and short acting beta 2 agonist which he use 3 times daily.

What is next step in the management?

A. oral steroid. B. Long acting beta2 agonist✅. C. Interleukininhibitors. D. Mastcellstabilizer

… lastResort

$Child with asthma and his mother also she had poor technic for inhaler use the child have extirpation

despite the treatment what is the cause= poor technic

….

!child with bronchial asthma her mother concerned about the prognosis of her child disease? it’s mostly

will get out in children ‫ﺮوح ﻣﻨﻬﻢ‬Ã‫ﺠﻴﻬﻢ ازﻣﺔ و‬v sz‫» اﻃﻔﺎل ﻛﺜ‬Ö Ñ ‫ﻌ‬v .


mediumdoseIcs dose Ics
high

SABA LowdoseIcs LABA er oral steroid


Leukotriene
Cmontelukastifpregnent

!A child who had flu-like symptoms, then has a typical picture of asthma exacerbation What is the first

line of management? Beta agonist ( not Anti-viral)

!Pregnant lady, just delivered and she's known to have bronchial asthma. Which of the following

uterotonic medications you would avoid giving? carboprost F2 alpha

!Child has Asthma , and father is smoking , what to do? Counseling parents of smoking cessation

Child with asthma & mitral regurge , presented with symptoms of pneumonia ( fever ) :give abx &

reevaluate later✅

!child has asthma symptoms but when do splrometry was normal what is the best : Methacholine

challenge test Second



linediagnostictest
Afterspirometry

!child had mild dyspnea, when exposed to dust diagnostic test of asthma:

A. Spirometer B. Methacholine challenge test

...

! 12 yo girl complaining of nausea and vomiting if the flight was more than 1 hour what is the best

antiemetic for her ?

A. ginger B. 2 drugs end with steron C. diphenhydramine D. Dimenhydrinate

Note: The Dx is Motion Sickness > Tx with Anticholinergic agent like Scopolamine

Or with Antihistamines agent like Dimenhydrinate

,,,,, failure to

#5 year boy with bronchial asthma on 2 medication, has syrr mother concern about him not gainingthrivemy

weight. Was : Less than 5th percentile weight and At 25th percentile height choices were:? cystic

A. reassure mother and refer to a dietitian fibrosis

B. investigate more for possible asthma mimickers ✅ cysticfibrosis AR

......

% 4 y child diagnosed with BA , has wheezing , failure to thrive = . refer for possible asthma

mimickers✅✅%
‫ﺐ‬È
‫ﺴ‬Ó ‫ اوﻻ اﻻزﻣﺔ ﻻ‬failler to thrive ، ‫ﺐ‬È‫ﺴ‬8‫ §ﻌﻤﻞ ﻧﻔﺲ اﻋﺮاض اﻻزﻣﺔ و‬¤d‫ﺶ اﻟﻤﺮض ا‬±‫ﺐ ا‬¹‫ ﻃ‬failler thrive ‫ ؟ ﻫﻮ‬cystic

fibrosis , ‫ »ﻠﻤﺔ‬Ñc‫ ف ﻣﻌ‬:

Asthma mimickers
‫ﻪ اﻋﺮاض
اﻻزﻣﺔ‬ú‫ﺸ‬Ó ‫ﻘﺔ ﻣﺎﻋﻨﺪە ازﻣﺔ ﻋﻨﺪە ﻣﺮض اﻋﺮاﺿﻪ‬¹‫ اﺳﺎس اﻧﻪ ازﻣﺔ واﻟﺤﻘ‬ä‫ﻜﻮن اﻟﻄﻔﻞ ﺷﺨﺺ ﻋ‬Ò‫ و‬، ‫ﻪ ا اﻻزﻣﺔ‬ú‫ﺸ‬± ‫ ﻣﺮض‬Ñd c ‫§ﻌ‬

‫ وﻣﻨﻬﺎ‬cystic fibrosis
... .... ..

M child C/O dry cough, has FHx of BA, he was given bronchodilator but hasn't improved, what is the most

indicating factor of cystic fibrosis ?A- Not responding to bronchodilator B_ Poor weight gain✅✅

!S & S of cystic fibrosis and asked about which sign u suspect to find ? Nasal polyp

Child presenting for check up clinic when to be concerned about growth ?

A-Body wt persisting at 10th percentile ✅ ‫ة‬²ï‫ رﻗﻢ ﻋ‬B-At 15th percentile C-Progressing from 5th to

!Criteria to help diagnose HTN in a 12 YO girl? A. BP >120/80 B. BP>140/90

C. >90th percentile for age and sex

D. >95th percentile for age and sex

!During the evaluation of adolescent she was on the 95th percentile for age and sex, what of the

following action should be taken

A. Initiate multidisciplinary team B. Confirm size is more than average for age and sex C. Give a list of low

calorie food D. Avoid talking about weight

$Growth chart (similar to the pic) showing normal at birth, increasing but below the 3rd centile,

Diagnosis? failure to thrive $

....

1Autoimmune

!Pediatric pt with ptosis and dlplopia and knee hyperreflexia with no fasciculation symptoms worse

through the day and improve next morning , what is the pathophysiology : increased

A- Anti-bodies against thyroid preoxidase SX worsewith


andfwrest
muscleuse

B- anti-bodies against Acetyle cohen receptors ‫ﯾﺣﺗﺎج ﺗﺎﻛد ﺣل‬

ptosisdiplopiaButyrin
MyastheniaGravis

strabismus dramblyopia
positivecovertest

C- genetic mutation in specific gene ' c+s)

.... 1- Hydroxychloroquine and corticosteroids

strabismus 2- Hydroxychloroquine and Methotrexate


!Child with +ve cover test -) stabismus 3- Hydroxychloroquine and azathioprine(refractory to meth)

Hydroxychloroquine and Mycophenolate mofetil


This is an Important risk for (or major concern or something) as it can cause :

lazyeye
Hydroxychloroquine and cyclophosphamide

A-amblyopia✅ B-strabismus C-leukocria

Note:+ve cover test Diagnosis ? Strabismus

Mostseriouscomplicationof S LE motheronBaby

iscomplete Block 2 1 bab


Heart
!Baby with AV block... Which of the following Maternal conditions is associated with the

presentation? SLE others skinrashgthrobocytopenia

improveswithActivity LFTabnormal
C

!Active arthritis for 4 months with morning f RA stiffness 2 h with x Ray findings

= Hydroxychloroquine and methotrexate weekly ✅


e
ginza

!Child with URTI then complaining of bleeding from nose, gum and bruising the treatment is ? ITP

A. Prednisolone ( for mild bleeding + less than 30, 0000 + like this case all symptom is "mild bleeding " .

✅✅✅ When a rapid rise in platelet count is desired (eg, planned surgery or procedure that is likely to induce blood loss, head trauma without ICH, severe unexplained headache), we suggest treatment with IVIG

B. IVIG (Bez there is no sever bleeding , sever bleeding mean there is intracrania Hemorhgae )
Treatment of ITP: ‫تلخيص للزبده من اب تو ديت‬

C. Antibiotics ‫ﻣﺎﻟﻪ دﺧﻞ‬ • If cutaneus Bleeding only, such as Bruising of petechia and (plt>30000) —> SUPPORTIVE

D. Supportive ‫ف ﻣﺎﻳﻨﻔﻊ‬íc c ‫ ( اﻻن ﺟﺎﻟﺴﺔ ﺗ‬we choice when there is no bleeding )


• If cutaneus Bleeding only, such as Bruising of petechia and (plt<30000) —> STERIOD or ( IVIG When a rapid rise in platelet count is desired (eg, planned surgery or procedure
that is likely to induce blood loss, head trauma without ICH, severe unexplained headache), we suggest treatment with IVIG)

• If severe mucosal bleeding or suspect internal hemorrhage BUT without hemodynamic instability!—> combination therapy of steroid and IVIG

…. when cutaneus Bleeding only, such as Bruising of petechia • If life threatening bleeding (Hemodynamically instability) —> Plt transfusion + steroid + IVIG

4 y old girl with history of limping and movement restriction for 2 months, ANA negative, she is otherwise
normal What is the screening for Uveitis ?

if ANA +ve

1) every 3 month. 2)every 6 month✅✅ 3) every 9 month. 4) no screening b/c she is ANA -ve.

Uveitis Screening in Patients with Juvenile Idiopathic Arthritis

Uveitis is a serious complication of juvenile idiopathic arthritis (JIA). Approximately 6% of all cases of uveitis
occur in children, and up to 80% of all cases of anterior uveitis in childhood are associated with JIA

، ‫ ﻻن اﻟﻣوﺿوع ھذا ﻛﺑﯾر‬، ‫طﯾب ﺑﺷرح ﺑﺷﻛل ﻣﺧﺗﺻر ﻣرة‬

‫ ھذه‬، (( Uveitis )) ‫ ھو اﻧﮫ ﯾﺣﺻل ﻟﮭم‬Juvenile Idiopathic Arthritis ‫اوﻻ ﻣن ﻣﺿﺎﻋﻔﺎت اﻟﻲ ﻛﺛﯾﯾر ﺑﺗﺣﺻل ﻟﻼطﻔﺎل اﻟﻲ ﻣﺻﺎﺑﯾن ب‬

‫ وطﺑﻌﺎ ھذا ﺧطر ﻻن ﺑﯾﺟﻲ ﻟطﻔل وﻣﻣﻛن ﯾﺳﺑب‬uvitis ‫ ﯾﺻﺎﺑوا ب‬JIR ‫ ﺑﺎﻟﻣﯾﺔ اﻟﻲ ﺟﯾﮭم اﻟﺗﮭﺎب اﻟﻣﻔﺎﺻل‬٦ ‫ﻣن اﺣد اﻟﻣﺿﺎﻋﺎﻓﺎت اﻟﻲ ﺗﻘرﯾﺑﺎ‬

‫ ﻛل ﻓﺗرة‬، ‫ ﻓﺣص دوري‬Screeing ‫ ﻻزم ﻧﻌﻣل ﻟﮭم‬Juvenile Idiopathic Arthriti ‫ ف ﻗﺎﻟوا اي اﺣد ﯾﺻﺎب ب‬، ‫اﻟﻌﻣﻰ ﻟو ﻣﺎ ﺗﻌﺎﻟﺞ‬

‫ طﯾب ﻋﻣﻠوا ﺗﻘﺳﯾﻣﺔ وﺟدول ﻛل ﻛم ﺷﮭر اﻋﻣل ﻓﺣص وﺳﻛرﯾن ف اﻟﺟدول ﻣﻘﺳم ﻋﻠﻰ ﺣﺳب اﻟﻌﻣر‬، ‫ﻣﺣددة ﺣﺗﻰ ﻧﻛﺗﺷﻔﮭﺎ ﺑدري وﻧﻌﺎﻟﺟﮭﺎ‬

٦ ‫ ﻗﺎﻟوا ﻟو اﻟطﻔل ﻋﻣره اﻗل ﻣن‬، ‫ طﯾب ﺧﻠﯾﻧﺎ ﻋﻠﻰ ﻗد اﻟﺳؤوال وراح اﺿﻊ ﻟﻛم اﻟﺟدول‬، negative ‫ وﻻ‬postive ‫ اذا‬ANA ‫وﻋﻠﻰ ﺣﺳب‬

month6 ‫ ﻓﺎﻟﺟواب ﺑﺎﻟﺳؤوال‬. ‫ ﯾﻛون ﻛل ﺳﺗﺔ ﺷﮭور‬screeing ‫ ﻓﺎل‬Negative ‫ ﻋﻧده‬ANA ‫ﺳﻧوات و‬

…..

3 years old boy his mom complained that she noticed blood spotting in the dipper,

there was another episode 3 months ago, no abdominal pain or other manifestation, on examination

patient looks pale and mucosal dryness, what’s the most likely diagnosis:

A. Juvenile polyps✅

B. Mickle diverticulum = Mainly present with abdominal pain


C. Intusscesption = (red jelly-like stools), sometimes mixed with mucus.)
2years Age of

Juvenile polyposis syndrome is a disorder characterized by multiple noncancerous (benign) growths

, typically develop polyps before age 20; These growths occur in the gastrointestinal tract, typically in the

large intestine (colon). The number of polyps varies from only a few to hundreds, even among affected

members of the same family. . . Polyps may cause gastrointestinal bleeding, a shortage of red blood cells

(anemia).

t.pro

% juvenile idiopathic arthritis case : both knees arthritis , nothing else, type?Oligoarthritis%

… y35joints involving Join

!Pediatric Knee+elbow pain increase in morning ? Juvenile rheumatoid idiopathic!

…. weightsBearing

Redswollenjoint FeverBush Joint

Uveitis nailchanges Butitsasymmetric

Earlymorningstiffness

synovialthickeing

!12 y boy with right knee and wrist swelling and subcutaneous nodule = Juvenile rheumatoid arthritis

!Pediatric patient with rt elbow pain and lt knee pain, worse in the morning and

improve with day A. juvenile idiopathic arthritis✅. B. reactive arthritis

juvenil .‫ ﺑدون ﺗﻔﻛﯾر اﺧﺗﺎروا‬joint pain ‫ﻻن ﺑﯾدﯾﺎﺗرك وﻋﻧده ﻛذا ﻣﻛﺎن ﻓﯾﮫ‬

!Neonatal heart block, what is the most likely condition the mother had? SLE ( not Rubella ❌)

….
! BabyZylowith painless per rectal bleeding in the diaper Hx of same complaint 3 month?Dx?

A. Juvenile polyp B. Hirschberg dis. C. Mechels diverticulum ✅

‫ ﻣﻄﻌﻢ ﻣﺎك‬mac ‫ﺧﻠﻔﻴﺘﻪ ﻟﻮﻧﻪ اﺣﻤﺮ‬

!if there is familly history = Juvenile polyp

!pedia 3y pt have painless bleeding per rectum ?

A. Juvenile polyp B. Meckel's diverticulum (age)=typically appear before the age of two years.

….. Baby diagnosed with cystic fibrosis

!Eye Cover test ? Strabismus Amb lyopia with + sweat chloride test his

…. brother is normal, how to confirm

Pediatric Patient Dx with cystic fibrosis, sibling no hx What appropriate management? diagnosis of cystic fibrosis?

1) test siblings‫ﺎء‬i¯‫ اﻗ‬for chloride sweat test.✅ 2 test parents for chloride sweat test. A. Ctfr gene in parent

3) genetic test for CFTR gene for the sibling 4) genetic test for CFTR gene for parents B. Ctfr in sibling
C. Chloride test parent


D. Chloride test in sibling

! Baby with +ve sweat chloride test?

A. Sibling sweet chloride. B. Parent sweet chloride C. for brothers and parents

.....

! Pediatric patient presented with sudden apnea, has a history of upper respiratory infections, on ✅

examination the patient takes coughs several hacking cough then takes a breath with inspiratory wheeze,

between coughing she looks fatigued and exhausted. No fever. Best investigation

A. Sweet chloride text B. CXR C. ABG


cough,wheezing , recurrent infection, poor feeding and poor weight gain

..

Exactly this picture:

A. measles B. chicken pox C. henoch-schonlein purpura

a r

TetradSX1palpablepurpura2Arthritis
Arthralgia

Abdominalpain4Renaldisease

…. 3 Nephritegyndrom

#Child presented to ER with parents c/o hematuria and urine Incontinence with previous hx 2 weeks ago

with constipation and fever . Lab show Wbc; 8000 with low Hb, UA show : + protein and +++Rbcs, What's

most likely diagnosis??

A. henoch-schOnlein purpura ✅ B. Autoimmune hemolytic anaemia C. Post streptococcal

glomerulonephritis

.......

!A child presented with joint pain and hematuria. Mother reported history of URTI 4 weeks ago. On

exam there was petechial rash involving buttocks and thigh, otherwise normal. Platelets normal. What is

the appropriate Diagnosia? Henoch-Schonlein purpura

‫ ﻣﻌﻪ‬Cd Ö ¹‫ﺔ ﺗ‬¹‫ﻫﺬا اﻟﻤﺮض ﻳﺘﻜﻮن ﻣﻦ ﺛﻼث اﻋﺮاض اﺳﺎﺳ‬

Joint pain + abdominal pain + petecha

diarrheaprecedes
Ask about Treatment = mainly Suppurative. But with syptom active = Steroid . Bloody

ifsevere
Thieommobffittothee

……. HOSIStodays Renaltuna


!Young pt have hx of bloody diarrhea, after 1 week he developed petechiae rash, he have hematuria3and

low platelet - Pt & ptt normal=====A. HUS ✅ B. TTP C. ITP D. DIC

(HUS) is a condition that affects the blood and blood vessels. It results in the destruction of blood platelets

(cells involved in clotting), a low red blood cell count (anemia) and kidney failure due

.....

Child has bloody diarrhea after a week develop petechial rash + hematuria and low plt Pt and ptt normal

? A- hemolytic uremic syndrome. B-TTP C-ITP D-DIC

✴HUS= It results in the destruction of blood platelets (cells involved in clotting), a low red blood cell count

(anemia) and kidney failure due to damage to the very small blood vessels of the kidneys.

…. tplatlet I Hemoglobin
$years old boy with petechiae all over his body. Lab results: low platelets and high

creatinine level. What is the diagnosis? A. ITP. B. TTP. C. HUS✅✅. D. UTI


ABIN l creatinine

Hemolytic-uremic syndrome (HUS) often occurs after a gastrointestinal infection with E coli bacteria

(Escherichia coli O157:H7) = 3 thing happen ( low plattler + low RBC ( anemia ) + high creatinin (kidny

failler )

'''''' Has

!thrombocytopenia and uremia in studies and has fever and headache A. HUS B. TTP C. ITP

e e
… Neuro FeverNeurologicalManifestation

T
!Child with fever hematuria headache ask dx = TTP Thrombocytopenia

.... HA Renalt

!A child complains of Headache and petechiea , labs show decreased plt , normal creatinin.There is

attaches pic shows schizocytes. What is the probable diagnosis : TTP

.....

Rx pf TTP? plasma exchange


… TTP

schistocytes

les microangiopathic hemolytic

Anemia ofTTP

What is the dx: TTP

.....

!Treatment of thrombotic thrombocytopenic purpura = mainly by Plasmapheresis with or without

steroid

...

! Sick pt with fever and coagulation profile all high: A. TTP B. ITP C. HUD D. DIc

#.7 years old child day 7 post appendectomy come with fever and bad general .condition but her in

mechanical ventilation . After that do ct contrast . The child bleed from lvlne trachea and wound site ,

What is the Diagnosis ? DIC✅

!Dehydration baby what to found= Low pulse pressure or low urine output.

.......

!case of child with moderate dehydration what other sign looks at it= Lower urine output ✅

! Child previous URTI came with petechiae and abrasion Every thing is normal except platelets 15000 ask
for chronic> 3 to 6 months & not adequately controlled using 1st line

pm
about ttt ==A. Splenectomy B. Platelets transfusions (ITP CASE) ✅ C. Ivig
observational rs oid wecant Aeroidot
if I MzgrEhB
….

give
↳ only in life threatening Bleeding" hemodynamically instability”

supportive pedia
q8Bpd

....
!Clear case with GBS and some sever manifest ion ask about mange?
DIG

A. immunoglobulin B. plasm exchange symmetrical Ascending flaccidparalysis


... IVIG GIortRTI Bilateral

Following

!Boy post URTI a fews back develops Guillain-Barré syndrome symptoms viral

(similar to the above scenario) asking about the most like diagnosis:

A. Guillain-Barré syndrome

...

!GBS case with involvement of all lower limb and respiratory symptoms : IVIG

........

!prognosis of GBS = full recovers !

%Adult boy with previous history of infection presents with progressive lower limb weakness? GBS (Gullin

barr syndrom)

Child presented with erythematous pharynx, with cervical lymph nodes and rapid strplysin test negative

and low grade fever with positive EBV . It next step ?

A. Give antibiotics and anti-pyretic ‫ﺎ‬Òí™‫ﻜﺘ‬- ‫ﺲ‬G‫ ﻫﺬا ﻓﺎﻳﺮس وﻟ‬B. Give antipyretic and fluids✅

C. Docultureandsensitivity D. Give Acyclovir

No effective antiviral therapy is available for Epstein-Barr virus (EBV) infectious mononucleosis in

immunocompetent persons. Acyclovir and ganciclovir may reduce EBV shedding but are ineffective

clinically. Treatment of immunocompromised patients with EBV lymphoproliferative disease is

controversial A child came back from a visit to Africa, lethargic presented with facial and generalized body weakness. He is irritable and
has mild neck stiffness. Lymphadenopathy not mentioned, CSF is turbid what the causative organism is:

….. A. EBV B. CMV C. polio

#Child came from a visit to Africa. neck stiffness . can not elevate head or limbs : EBV ✅

‫ﻂ‬è‫ = ر‬ebv = ‫ﺎ‬¹‫ﻘ‬Ò¬‫ﻌﺪ ﻋﻦ اﻓ‬-‫ا‬.

polio

...

$11 years old child had severe diarrhea 3 weeks ago, now the child presented with

bilateral lower limbs weakness and numbness, dx?

A. Poliomyelitis B. Guillain Barre syndrome✅✅✅ C. Muscular dystrophy D. Cerebral palsy

….

!Child post-GI infection a few weeks back develops ascending symmetrical muscle weakness associated

with loss of reflexes. (It’s Guillain-Barré) how will you treat this child? A. Steroids B. Immunoglobulins

Child Herpes gingivostomatitis sx (lips, gums, tongue, palate vesicles) and he can’t feed orally. What will

you give him?

A. Antiviral (i'm not sure if they write acyclovir or antiviral)

B. IV fluid and antiviral✅✅ ‫ ﻓﺎﻳﺮال‬Ñd i ‫ﺎﻟﻔﻢ ﻓﻼزم ﺗﻐﺬ§ﺔ و»ﻤﺎن اﻧ‬- ‫ﺪ ﻻن ﻣﺎˇﺎن §ﻘﺪر §ﺎˇﻞ‬Ò&‫ﻻزم ﻓﻠ‬

C. Abx

......

! A 12 month old child developed painful vesicles around his lips, tongue, gu and hard palate. He is unable

to tolerate orally. What is the most appropriate management?,,

A. oral acyclovir B. IV acyclovir + fluids C. IV abx + fluids

! child with gum bleeding, erythema papules in mouth. Swab showed ( multinucleated giant cell ) on

Tzanck smear , organism = Herpes simplex!

!White vesicles around mouth and gum with cervical lymphadenopathy, diagnosis? HSV

… chickenpox

A pruritus Trunk whole


!5 years old presented with rash (picture provided) the rash started as one scratch and then became Body

generalized over the body. Inguinal and axillary lymph nodes are swollen. What will you give? Acyclovir
immunocompromised
indiation

Andyoung
aspirin
therapy

! A young child presented with tonsillar ulcer and painful lesion in the back of her mouth and soft palate

, what the dx ? A. Herp angia !

… Coxsackie virus

! Child with fever 39 ,sore throats on examination there is white exudate over the tonsils most

appropriate step to reach the diagnosis ? A. Throat culture B. rapid antigen detection test


fluctuantpainless swelling
we transilluminetion

1Hernia Resolves spontaneously


withinGmonths dullachingpain
vetransinuminition

!Neonate , palpable mobile mass non tender , what is it ? A.Hydrocele ✅ B. Varicocele

‫ﻂ‬è‫ = ر‬hydro = ‫ﺐ اﻟﻢ‬È‫ﺴ‬±‫ﺪروا ﻣﻌﻨﺎﻫﺎ ﻣﺎء =اﻟﻤﺎء ﻣﺎ‬¹‫ﻫ‬

............

-Child came with testicular swilling wasn't painful positive transillumination normal sensations in preanal

area? A-Testicular torsion. B-Hematocele. C-Hydrocele✅✅. D-Indirect hernia

(hydrocele is a sac filled with fluid that forms around a testicle. )

….

!testicular asymmetry and was tender on palpation = testicular torsion = A testicle that's positioned

higher than normal or at an unusual angle. !

….
TX orchidopexy

5 years old child found to have one testes in the scrotum and the other in the ingunal area what to do?

A-orchiopexy ✅. B-orchiectomy. C-wait tell puberty. D-GnRH

(Orchiopexy is a surgery to move an undescended testicle into the scrotum and permanently fix it there.)..

... ..... .. .... ...

!Baby with painless scrotal swelling bilaterally , neg transillumination test, Dx?

A. Testicular torsion B. Orchitis C. Idiopathic scrotal edema

‫ ﻻن ﻗﺎﻟﻚ‬painless ‫ﺎرات –ﺴˆﺐ‬n‫ﻞ اﻻﺧﺘ‬-‫ و‬pain

.....

15 years old with scrotal pain and absent cremasteric reflex diagnosis?

A-varicocele = ‫ ﻫﺬا ﻟﻮ‬cemasteric reflex postive.

B-tusticular torsion✅ C-epididmaitis í™‫ﺔ وﻫﺬا اﻟﻮﻟﺪ ﺗﻮە ﺻﻐ‬¹‫§ﺠﻞ ﻣﻊ اﻧﺘﻘﺎل ﻏﺪوى اﻻﻣﺮاض اﻟﺠˆﺴ‬

D-indirect hernia

Epididymitis:

It's usually caused by a bacterial infection or a sexually transmitted disease (STD). The condition usually

improves with antibiotics.

...... ...... ...... ...... ptesticulartorsion


12 years old male with testicular pain (horizontally lying, mildly elevated, no swelling or erythema of

scrotum) what to do next: -surgical exploration ✅‫ اﺳﺘﻜﺸﺎف‬-rehydration

‫ﻋﺔ‬²¯ ‫ ﻃﻮل ﻣﻦ اﻻول ﺟﺮاﺣﺔ‬ä‫ﻨﺎ ﻋ‬Ò¬‫ﻟﻦ ﻧˆﺘﻈﺮ ﻋﺸﺎن §ﺤﺼﻞ ﻟﻬﺎ ﻏﺮﻏ‬

!child complain of "unilatral scrotal swelling " dose not transillium whats nexst step ?

Discharge pt

Ÿ
Give antibiotic
X varicocele
Do laprotomy X
Ÿ

Ÿ
x
Ÿ U/s and think about surgery ✅ ‫اﺻﺤﻬﻢ‬

10 month girl , her parents noticed lump in the girl while she crying, disappeared in sleeping

(umbilical hernia) she is otherwise healthy. Management?

1) reassurance ✅ 2 ‫ة‬í™‫ )ﺗﻮﻫﺎ ﺻﻐ‬mech repair 3) simple repair. 4)laproscopic repair

6 90% will close spontaneously, if not surgery intervention after the age 4-5 years
! 18 month baby complain of umbilical hernia become huge with cry = wait and watch✅

….. Strangulated

!Child with swelling, red, tender hernia. Dx? Incarceration (red inflammed tender is incarcerated. )

Baby abounded by his mother as he’s result of out ofillegal


wedlock pregnancy

Pediatric surgeon asses him he has inguinal hernia decided he need inguinal

hernia repair. Pt need consent What’s appropriate next step?

A-refer the police B- refer to social worker B- you do the consent for the baby. C- call the ethical

committee ✅ Respdistress intubatebreechvent

chest


f
Bmj.iednsdnfif zNGT
Diaphragm hernia what’s the management? 3SX thoracotomy
JABA

or laparotomy

3 Surgery = if ask about treatment . 2NG tube = if ask about whats nexst step .
‫ ﻏﺎﻟﺑﺎ اﻟﻛﻔل ﯾوﻟد‬، ‫ وﻧﺎادر ﻧﻼﻗﯾﮫ ﺑﺎﻟﻛﺑﺎر‬congintal ‫ ﻏﺎﻟﺑﺎ ﯾﺟﻲ‬diaphragm hernia ‫ اوﻻ‬، ‫طﯾب ﻻن اﻟﺳؤوال ھذا ﯾﺗﻛرر ﻛﺛﯾر ﺧﻠﯾﻧﺎ ﻧﺷرح ﻧﻘﺎط ﺑﺳﯾطﺔ ﻋﻧﮫ‬

‫ ﻟﻛن ﻗﺑل‬، ‫ ﺑﺎﺧﺗﺻﺎر اﻟﻌﻼج ﻧﺳوي ﻟﮭم ﺟرااﺣﺔ ﻓﻲ اﻗرب وﻗت‬، ‫ طﯾب ھﻲ ﺣﺎﻟﺔ ﺧطﯾؤة وﻣﻣﻛن ﺗؤدي ﻟﻠوﻓﺎة ﻟو ﻣﺎﻟﺣﻘﻧﺎ ﻋﻠﯾﮭﺎ‬، ‫وﻋﻧده ھذا اﻟﻣرض اﻟﺧﻠﻘﻲ‬

‫ ف ﻋﻠﻰ ﺣﺳب اﻟﺳؤوال ﻟﻣﺎ ﯾﻘوﻟك‬، ‫ ﻧودﯾﮫ ﻟﻠﻌﻣﻠﯾﺎت‬stablizantion ‫ ﺑﻌد ﻣﺎ ﻧﻌﻣل ﻟل ﻣرﯾض‬NG tube ‫اﻟﺟراﺣﺔ ﻧﻣﺳك ﺧطوة ب ﺧﻛوة اول ﺷﻲء ﻧﻌﻣﻠﮫ ھو‬

next step .‫اﻟﻌﻼج ﻟﻠﺣﺎﻟﺔ ﯾﺧﺗﻠف ﻋن ﻛﻠﻣﺔ‬

Congintal diaphragm hernia :

Immediately after delivery, the infant is intubated (bag-mask ventilation is avoided). A nasogastric tube is

passed to decompress the stomach and to avoid visceral distention.

How is a diaphragmatic hernia treated? Both congenital and acquired diaphragmatic hernias typically

require urgent surgery. Surgery must be performed to remove the abdominal organs from the chest and

place them back into the abdomen.

In this condition, there's an opening in your baby's diaphragm. This allows some of the organs that should

be found in your child's belly to move up into the chest cavity. This condition can cause serious breathing

problems. It is life-threatening.

. .. . . . . .. .

!neonate with hiatus hernia .. ?! NGT!

The first step in management is nasogastric tube placement and securing the airway (intubation). The baby

will usually be immediately placed on a ventilator.

!baby with diaphragmatic hernia what will you do after stabilization? NGT

….

!Baby with diaphragmatic hernia (pic provided) what is your management ?

A. Chest tube. B. Immediate Nasogastric tube C. Immediate hernia repair

‫ ﻣﺛل اﻟﺻورة وااھﮭﮭﮭﮭم ﺧطووووة اذا ﺷوﻓﺗوا ھذا اﻟﻣرض ﻧﻌﻣل ﻟﮫ‬، ‫ ﺗﻛون اﻻﻣﻌﺎء ﻣوﺟودة ﺑﺎﻟﺻدر‬diaphegramtic hernia ‫ﺷوﻓوا‬

.‫ واﺿﺢ ﺑﺎﻻﺷﻌﺔ‬NGT ‫ﺣﺗﻰ ﺗﻼﺣظوا اي ﺻورة ﻓﯾﮭﺎ دﯾﺎﻓﯾﺟراﻣﺗك ھﯾرﻧﯾﺎ ﺑﺟوﺟل ﻻزم ﺗﻼﻗون ن ﺗﯾوب‬NGT

…..

$congenetal diaphragmatic hernia, after stabilization, management is: - large NGT ✅

$Full term infant of diagnosis as diaphragmatic hernia. Voyageurs support done at delivery... TTT is

A. SURGICAL REPAIR URGENT B. ANTIBIOTICS C. CHEST TUBE

….

$Child with bilateral inguinal hernia what is the treatment : A. Herniotomy .✅( bez child) B.

laparoscopic mesh repair

1GIT
!Baby 6 weeks, direct bilirubin high. Dx? A- Crlglar najjar B- ABO C- Rh Incompatibility D- Choledochal

cyst ✅

… Indirect
# child w/ gastroenteritis and they asked diagnostic test = Stool antigen✅

!Scenario about baby fall down on his RIGHT abdomen from 1 day then develop abdominal pain and

fever= liver contusion


..
incomplete 2

!Pt baby with abdominal distention and difficulty passing stool , on PR examination empty rectum, but

after finger pulled out there is diarrhea, DX: A. hirschsprung B. intussusception

… MyEnterocolitis

#3 Days neonate passed meconlum and since then he is passing seedy yellow

stool and on breastfeeding he has intestinal obstruction what is the diagnosis? Hirschsprung Disease

...

!Sx of hirschsprung disease with plc asking for a dx = Delayed passed meconium = Diagnostic by =Rectal
Biopsy

Teated by resection and anastomoses, leveling colectomy

.......

! Patient came with gastroentrietis When start normal diet after the ORT( in pedia

A. After 24 h B. After 5 days

!Patient complained of Rt groin pain for 2 weeks and exacerbated by exercise, physical examination

showed normal groin and genitalia, next step?

A. CT abdomen B. MRI abdomen C. Us D. Re-evaluate after 2 wks

‫ﺎ ﺣﺼﻞ ﻟﻠﻌﻀﻠﺔ ﺷﺪ‬ú‫ﺎﺿﺔ ف ﻏﺎﻟ‬Ò¬‫ﺪ ﻣﻊ اﻟ‬Ò$‫ﻗﺎﻟﻚ اﻧﻪ ﻳ‬

overstretching the muscles during sport.t'= ‫ﻣﺎ§ﺤﺘﺎج اﺷﻌﺔ‬

..

! How to decrease the risk of airbag injury in children younger than 12 years: Restrain to back seat

hittin't
j

gg

What’s your management: A. open cyst deerofing. B. albendazole

….

! Child with his parents start to develop fever lethargy, parents mention 2 days before changing his eye

color to yellow to he became ictrus, which type of viral hepatitis he had:

A. Hepatitis A B. Hepatitis B C. Hepatitis C D. Hepatitis D

!Child with attacks of severe midline abdominal pain with facial pallor. Poor appetite .. hx 2 = sisters with

migraine abdominal migraine

….

!10 years old boy with fever and lethargy and mild abdominal pain + recurrent diarrhea sometimes

bloody + loss of weight and on Examination child looks pale = ulcerative colitis(bloody diarrhia + anemia

go more with UC not crohns )

….

$When say this pain from organ ( organic pain) ?

A. Befor sleep time. B. Before awakening✅. C. Two choices I forgot it


a

$A child presented with 3 days of vomiting and stooling. Other information were given. What will be the

electrolyte derangement?

A. Hyperchloremic metabolic acidosis

My

B. Hypochloremic matabolic alkalosis✅


C. Hyperkalemia matabolic alkalosis

D. Hypokalemic metabolic acidosis D


magiasndtriiEate9FsIhEtahiIvhs

M confirmetorgasyjonfo.mn
! gastric cancer what is of high diagnostic value? fasting gastrin level !

the choiceschoose it

C EA or CA1a a if its in

10 y With diarrhea and sometimes bloodyl

A. Celiac ‫ ﻣﺎ§ﻌﻤﻞ اﺳﻬﺎل دم‬B. Cohns c. uc ✅ D. Dysentery

...........

celiac , what food should avoid? Barely

....

!12 year with maldigestion. He has greasy foul smelling diarrhea and trouble gaining weight+ feels

lethargic most of the time. BMI: 16, Hg low the diagnostic test=

Antiendomysial antibodies (presint in celiac disease)

‫ﺎك‬¹‫ﻠ‬¹‫ﻞ =ﺳ‬¹‫ﺴ‬±‫ﻂ ﻣﺎ‬è‫ر‬

......

...

!Caucasian pediatric presenting with failure to thrive, abdominal pain, and distention. He was normal

until he was started on regular food‫ﻨﺎ‬P ¤‫ﻠﻤﺔ اﻟ‬-. diagnosis = Celiac disease

‫ﺎك =ﺳﻠﻖ =اﻻˇﻞ‬¹‫ﻠ‬¹‫ﻂ =ﺳ‬è‫ ر‬.

...

! celiac diseas , indicates activity of the disease= Anti Tissue transglutaminase Ab

c ™ ‫ = ﻣﻦ اﺳﻤﻬﺎ ﺟﻠﻮﺗ‬glutin ‫ ال‬Hc ‫ﻣﺮ‬ciliac ›


c ™ ‫ﺎﻟﺠﻠﻮﻟﺘ‬- ‫ﻠﺔ‬ù‫ﻋﻨﺪﻫﻢ ﻣﺸ‬

......

! 13 Years old girl has type 1 dm , her weight 40 kg ( below 50th percentile ) and his hight 150 cm (

below 95th percentile ) , she has no signs of secondary sexual characteristics of puberty, you want to

perform annual screening in clinic for ? (common Q ) ( celiac dis screening) !

.............

!16 years old male with 3 years history of diarrhea ( food containing )= Anti endomysial antibodies ✅

Q was high risk illness baby, from whom do you take details ? A.Father B. Mother ✅. C. House

% Signs of obstruction in a child best initial modality of diagnosis = A- US✅. B- chest x-ray C- CT

… Ultrasound dose not have rules to detect intestinal obstruction like x ray
these are signs of obstruction and we should do X ray for intestinal obstruction , if no x ray and there is CT choose it

Pediatric patient with PUD, what’s your management? A. PPI. B. PPI + Amoxacillin + lmidazole ✅✅

f … Antifungal
4 years old presented with preforated divertcular. laprtomy cleaning done. after days the patient

presented with fever, abdomen was soft, wound was .clean in DRE found anterior bulging and apropreate
management:

A- us drainage (mostly abscess and need drainage not only antibiotics )✅

B- abdminal tract watchout. C- iv antibiotic. D- iv paracetamol

initial management is ABx But the best is drainage + Regarding the size, its all about if the patient needs OR
drainage or US

…. lifestylemeditation best to choose and most accurate


agents with central anticholinergic
&GERD CASE symptom increase at night only what to give her?Bed elevation & PPI!

activity (e.g. diphenhydramine,


… benztropine)

rs cholinergic

$ Pediatric had gastroenteritis and she took metoclopramide that leads to involuntary movement facial

grimace and tongue protruding what to give: A. Diphenhydramine✅. B. Epinephrine. C. Cyproheptadin.

= . ‫ = دوﻟﻔﯾن = ﯾﻌﻣل ﺣرﻛﺎت ﻛﺛﯾرة = ﺗﺷﻧﺟﺎت‬diphen‫داﯾﻔﯾن‬

(Acute dystonic reaction, common side effect of metacroplomide. treated by IV anticholinergic such as

benztropine or diphenhydramine)

.........
Obesity complications if pediatric = hypertenstion

‫ﻢ‬¹‫ﻤ‬ù‫ﺔ ﺗ‬¹‫ﻋﺔ او ﻋﻤﻠ‬²¯ ‫ ﻫﻨﺎ ﻧﺨﺎف وﻻزم ﻳﻨﺤﻒ‬¤d‫ﺐ ﻣﻀﺎﻋﻔﺎت ؟ اذا ﻟﻘﻴﻨﺎ ﺿﻐﻂ اﻟﺪم ﻋﺎ‬¹‫ﺪﺋﺖ ﺗﺠ‬- ‫ﺎﻻﻃﻔﺎل‬- ‫ اﻗﻮل اﻧﻪ اﻳﻮا اﻟﺴﻤﻨﺔ‬Ñi‫ ﻣ‬.

$ 4week old neonate full term Co projectile vomitting- Ex there is mass olive shape Investigation the:

A. SONOGRAPHY ✅ ‫ ﻫﻮوﻧﻔﺴﻪ اﻻ§ﻜﻮا‬B. SINCITY C. ENDOSCOPY

… 02
e

&Projectile non-bilious vomiting ,olive like abd mass: dx? A-pyloric stenosis ✅

….
$ Olive mass :Pyloric stenosis
......
fluid
riignIYYIII.eu tbk
O
Q
F IJ

$Case of pyloric stenosis : pylomyotomy. After siblization Yes pylomytomy

!Child with projectile non biliary vomiting + mass in epigastric most accurate investigation? US what

treatment?pyloromyotomy

...

stenosis

pespyloric
! Child came constipation and bilious vomiting for 1 week pain and mild tenderness in left side and

abdominal distension and ancient bowl sounds ,the reason for surgical referral ?

A. absent bowel sounds B. bilious vomiting C. abdominal distension D. constipation

pyloric stenosis what occurs the diagnosis? - failure to thrive✅

….

!Pyloric stenosis what is electrolytes abnormalities:-hypochloremia hypokalemic metabolic alkalosis

….

!Baby withprojectile vomitingand ugre to drink ( feeling hungry)== pyloric stenosis ✅

….

Child with repetitive vomting and laps shows hypocholrmic hypokalemic metabolic alkilosis and eant the

daignosis Pyloric stenosis Child has SOB, recurrent infection, failure to thrive, vomiting.

What does he have? A. Hypertrophy of pyloric muscles


….

B. Hypertrophy of esophagus sphincter


&hypochloremic hypokalemic metabolic alkalosis = pyloric stenosis& C: relaxation of the LOWER esophageal sphincter

D: relaxation of the UPPER esophageal sphincter


!Baby 2 yer CameGERD with apnea -vomiting - dehydration- lethargic-FTT- not gaining wt what is the

IT

problem: A) not relaxed Upper esophageal sphincter B) not relaxed lower esophageal sphincter
C) not relaxed pyloric muscles D) hypertrophy of pyloric m

…. MI if

2ndto 7thweekofAge
!scenario of pyloric stenosis , best diagnostic? A. Barium swallow B. Ct chest or abdomen C. US✅

!Child with Metronidazole and Omeprazole for eradication of H.Pylori drug you want to add =

Clarithromycin

liability of diagnosed necrotizing enterocolitis : A.male sex. B. pre- term ✅ C.young age

most important risk factor for Necrotizing enterocolitis: - birth weight < 1.5 kg ✅

Baby preterm 32 weeks , wight 1200g .. symptoms of bloody diarrhea, he has necrotizing enterocolitis ..

what related to that ? A. Male B. Bloody diarrhea C. Weight below 1500g


....

!NEC MT preterm
case asking about factor If its absent risk for NEC ?? Full term .
as

!&continuous bilious vomiting , Abd destention, Passes meconium after birth, now yellowish thin

diarrhea===.,Colic a-hirschsprung disease b-toxic megacolon c-enterocolitis✅✅

‫ﻂ‬ú‫ﺎﻟﻀ‬- ‫ﻫﺬە اﻋﺮاض‬. == necrotizing enterocolitis &!

Necrotizing enterocolitis (NEC) is a medical condition where a portion of the bowel dies. It typically occurs

in newborns that are either premature or otherwise unwell.

….
firstuurs

Toxic 7.5 logo support.ve


1Over dose s anurs

!acetaminophen ingestion they mother says that the bottle had a small amount and he spilled some of it

on his clothes after 4 hours observation in ER he is still stable

A. Discharge home with instruction. B. acetylcysteine. C. Watch for another 4 hours

N.B: Should be serum paracetamol level

Boy came with history of alkali causative ingestion with no symptoms. What is the management:

A. Induce vomiting B. put nasogastric tube. C. neutralization by water. D. close observation



www.g.at gyypgmm.ge


if
symptomatic upperEndoscopy

y y

9 years girl ingested two boxes (each 20 tablets) of paracetamol one dya ago after fight with her mother?

e
Mx: -n-acetlcystine. -gastric lavage. -observation and discharge pt ✅

‫ﻞ‬ú‫ ـﻊ ﺳﺎﻋﺎت ﻗ‬è‫ اﻗﻞ ﻣﻦ ار‬bd c ‫ ﻟﻮ ﺣﺎت‬، ‫ ﺳﺎﻋﺎت »ﺎن ﻣﻤﻜﻦ اﻋﻜﻴﻬﺎ اﻟﺪواء‬٨ ‫ء‬îï c
d I‫ﺎر ﺧﻼص ﺧﻠﺺ وﻗﺘﻬﺎ ﻟﻮ ﺟﺎت اﻗ‬¹‫ اﺧﺘ‬Ùd ‫ﻻن اول وﺛﺎ‬

ï c
‫ﺖ‬¹‫ﺔ وﻧﻄﻠﻌﻬﺎ ﻟﻠﺒ‬Ò&‫ﻪ ﻟﻠﻌﻼﻣﺎت اﻟﺨﻴ‬ú‫ء ﻧˆﺘ‬î d ‫ ف اﻫﻢ‬، ‫ﻞ ﻣﻌﺪة ¯ﺲ ﻣﻼص ﻓﺎت اﻻوان‬¹‫ ﻟﻼﻣﻌﺎء »ﺎن ﻣﻤﻜﻦ اﻋﻤﻞ ﻟﻬﺎ ﻏﺴ‬I
d ‫ﻣﺎ اﻟﻤﻌﺪة ﺗﻔ‬

......

$Child ingested iron Serum iron 90 Came with nausea and vomiting I think 9 hours ago, Mx?

A. Gastric lavage B. IV deferoxamine✅

$Paracetamol poisoning with symptoms ttt? N-acetyl salicylate

child not talking Arabic, before operation he seems very anxious, how should you act as an intern:

auscultate his ches then let him auscultate yours ✅

- paracetamol overdose for a child stage 1-2, one day ago, management:

A- don’t give anti dote cause it will have no effect by now ✅✅


‫ ﻻن ﻏﺳﯾل اﻟﻣﻌدة ﯾﻛون اﻗﺻﻰ ﺷﻲء ارﺑﻊ ﺳﺎﻋﺎت‬، ‫ﺑﺎراﺳﯾﺗﺎﻣول ﻣرت ﻋﻠﯾﮭﺎ ﯾوم ﻛﺎﻣل وھو اﺧذ اﻟﺟرﻋﺔ اﻟزﯾﺎدة ف ﺧﻼص ﻣﺎ ﯾﻧﻔﻊ ﻣﻌﮭم ﻏﺳﯾل ﻣﻌدة‬

‫ ﺳﺎﻋﺎت ﻗﺑل ان‬٨ ‫ ﯾﻧﻔﻊ ﻓﻘط ﻟو ﺧﻼل‬N-acetylcysteine ‫ اﻟﻲ ھو‬antidote ‫ ودواء‬، ‫ ﻻن اﻟﻣﻌدة ﺧﻼص ﺗﻔﺿﻲ ﺑﻌد ارﺑﻊ ﺳﺎﻋﺎت‬، ‫ﻣن ﺗﻧﺎول اﻟﺟرﻋﺔ‬

. ‫ ﺳﺎﻋﺔ ف ﻣﺎ ﻧﻌطﯾﮫ ﺷﻲء‬٢٤ ‫ ف ﺧﻼص ھذا ﻣر ﻋﻠﯾﮫ‬، ‫ﯾﺻل اﻟدواء ﻟﻠﻛﺑد وﯾﺣﺻل ﻟﮫ اﻣﺗﺻﺎص ﻓﻲ اﻟدم‬

..... .....

● Female ingested 20 tablets of acetaminophen then came with N/V and right UQP what to do?

N-acetylcysteine. ✅✅. B.No need for antidote.

… depends on type symptoms

6 years old drank a bleaching agent 30 mins ago, how will u manage ‫ﻜﺲ‬Ò‫˙ب »ﻠﻮر‬ï?

1- CloseMif Asymptomatic
observation. 2- Cleaning stomach with NGT. 3- drinking water to dilute the bleaching agent

4- endoscopy ✅

if symptomatic

! 8 years old presented with nausea vomiting diarrhea she was (dehydrated) then they do rapid fluid

correction after that she deteriorate and become comatose. How to replace her fluid deficit?

A. over 48 h B. Over 24 h C. Over 12 h D. Over 6 h ....NB: If with electrolytes correction > 48..

… tape ds.ve
!12 years old boy ingested bleach‫ﺾ‬nÕ‫ |ﻠﻊ ﻣﻮاد ﺗ‬by mistake they started IV fluids and he is vitally stable

close observation as i remember Gastric irrigation or something like this I totally forgot but there is other

hrsi
reasonable which? observation

inEry 1Kidney

disease

Gpcauslice
!Dialysis requirement (high potassium unresolving to treatment) =Ca-gluconate!

..

!25 yo male his sister has adult polycystic kidney asking about screening for him

a. anti cysteine b. Ultrasound abd. c. CT abdomen

… supportiveTx foredema
Nephriticsyndrome PSGN

Dark urine, Proteinuria 2+, HTN. ttt? A- Steroidvv B- Diuretic Frusemide✅. D- Antibiotic

….

child i think 4y with fever 39.9 , dark urine , irritability, o/e tender abdomen with no organomegaly,

investigation( WBC 16.000 , urinalysis show proteinuria +2 and erythrocyte 18 normal is 0-3 or 2 .. what

you will give the child : A- furamide B- ceftriaxone ‫ﺪ‬úı‫ﻖ اﻟ‬Ò¬‫ﺴﺘﻘﻠﺐ ﻋﻦ ﻃ‬± C- steroid D- salt restriction

….

!Young with fever and dark urine has history of constipation , urinalysis: high leukocyte, high

erythrocytes, high protein = pyelonephritis ( not acute glomerulonephritis )

● Pediatric pt c/o of dark urine and fever BP ,Was 125/80 ,Urine analysis ,Protein +2 Diagnosis

?Polynerites✅

….

!child came with his parents to ED complaining of fever and dark urine the parents gave hx of the pt have

constipation on ex pt have abdominal tenderness on urinalysis +wbcs +Rbcs +proteins What’s most likely

dx? A. Hypertensive nephropathy B. acute glomerulonephritis C. pyelonephritis D. cystitis

…..

Child with edema in eyelid and ankle with pale and dark urine . What you need in hx ?

A-impetigo in last month ✅B-SCA

PSGN, For GABH ) usually in IO days

But for skin infection it takes 3weeks

#An infant presents with a 2 day hx of lethargy, malaise, and dark urine. What is tho Important thing in

hx that you must ask? A. Recurrent UTls B. Recent impetigo contagiosa

.....

! Patient with impetigo when to do nursy what is best answer

A. Drug continues five days. B. Drug continues 3 days. C. Drug continues until dry

….

What’s the diagnosis? Impetigo

Ddx: IgA nephropathy: during or immediately following(1-2


Ddx: nephritic syndrome days) a respiratory or gastrointestinal infection

poststreptococcal glomerulonephritis (Postinfectious glomerulonephritis)

prior infection with group A beta-hemolytic streptococci

+Nephritic syndrome symptoms


URTIorskininfeen

Hematuria 7 todays
Hypertension

Edema

... Tx: supportive only

$child developed generalized edema with fever and dark urine, all labs normal except low calcium and

low albumin, what is dx?

A. Minimal change disease✅ B. Mesangial Disease C. Nephropathy D. May be Glomerulonephritis

........

most common type of nephrotic syndrome in children? Minimal change disease

‫ﻂ‬è‫ = ر‬minimal = child

....

!most common in Adult =Focal segmental glomerulosclerosis

.....

%child with puffy eyes, hypoalbuminemia? Minimal change glomerulonephritis%

..

- Treatment of patients with minimal change GN? . Steroid

X
! 10 years boy c/o puffy eyes for 1 week with a history of recent infection, no edema no urine changes

and labs completely normal what to give him?strong steroid✅ everything is Normalwhysteroid

.....

ofdailycortisone
!How to know steroid resistant nephrotic syndrome: A. after 4 wk B. 4 months

!child about 12 or 13 y with edema bilateral periorbital edema.diagnose with nephrotic syndrome ..

what’s the management medications : prednisone .

….

! Patient child with nephrotic syndrome, contraindication use: = NSAID

….

! Child e Nephrotic syndrome What medication to avoid in management:

A. aspirin B. amoxicillin C. acetaminophen D. ibuprofen edges


#Child came lethargic ,fever,abd pain hls family say they have history of mldotrlnlan disease ,Labs normal

except ratlculocytes high which of tho following drug contralndlcatad = aspirin . Macrolides and verapamil

….

!pediatric nephrotic syndrome with edema ( Face edema ) and proteinuria without hematuria =Steroid

trial!

! Q about Steriod resistnce nephrotic syndrome :- no remission in 4 w !

! when you said that child have steroid resistant =Induce remission in 4 weeks

….

$A case of pediatric nephrotic syndrome with edema (Face edema) and proteinuria without hematuria,

what will you give him? A. Steroid trial✅. B. Diuretic. C. Antibiotics. D. Biopsy

!Child with nephrotic (diagnoses given), after ttt given when can you confirm the baby completely

resolve?

A. After normal dipstick 3 Consecutive time. B. After normal depstic 5 Consecutive time. C.edema resolved

...

!Another Same Q but asking about effect of ttt when? A- 2 weeks B- 4 weeks

....

child diagnosed as nephrotic syndrome and given steroids, question is what's the best indicator for

remission ?

a-edema resolved b-urine analsyis negative for protien for 3 days✅ c-same as choice 2 but for 5 days

!- child with facial and lower limb edma with protinurea on thiasid and other medication i don’t

remember wt best next step?- switch thiaside to fursmide✅!

! pediatric nephrotic syndrome with edema ( Face edema ) and proteinuria without hematuria , what

will you give him ? A. Steroid trial✅✅ B. Diuretic. C. Antibiotics D. Biopsy

….

! Child with tonsillitis.. I think the question was about complication.

A. pharyngitis B. bronchitis C. glomerulonephritis

.....

!A 16 year old presents with cola colored urine and has fever that followed a rash. Urinalysis was

provided + creatinine was high, Most likely diagnosis is ,, A. acute glomerulonephritis. B. IgA nephropathy

.... Fever Rash HSPassociated Nephropathy

%%Child had UTI one day next develop hematuria' ? Poststreptococcal glomerulonephritis ( Not IgA

nephro) %%

:‫ =ﺗﯾﺟﻲ ﺑﺳﺑﻲ اﻧواﻟﻣرﯾض ﻋﻧده اﻣراض ﻣﻧﺎﻋﯾﺔ ﻣﺛل‬IgA nephropathy

as rheumatoid arthritis and systemic lupus erythematosus (SLE)

، ‫ ﻓﻲ أي ﻣﻛﺎن ﺑﺎﻟﺣﺳم ﺳواء اﻟرﺋﺔ‬infection ‫ ﻣن اﺳﻣﮭﺎ ﺗﯾﺟﻲ ﺑﻌد اﺻﺎﺑﺔ اﻟﺟﺳم ب‬Poststreptococcal glomerulonephritis ‫ﺑﯾﻧﻣﺎ‬

NSAIDs linestrepto . %% ‫ ﻟﻛن ﻧوع اﻟﺑﺗﻛﯾرﯾﺎ ﻣن ﻓﺻﯾﻠﺔ و‬، ‫اﻟﺣﻠق‬
first
….. Reactive

Arthritis Resort

rs EFaedum
sstffefia.dz
plast
!Baby with ankle and knee arthritis, he had UTI prior 2 wk what to give ? A. Azathioprine B. methotrexate

0
asymmetric
m igratory
j
f
o o_0

!Pediatric complain of arthralgia, SOB, rash in lower extremity , fever past hx of dental procedure before

X
2 month and sore throat before 2 week . on lap proteinuria and hematuria , On examination murmur ..

what diagnosis A. infective endocarditis. B. post- streptococcal infection C. SLE

posimplecystitis
!Girl 7 years with suprapubic pain No rebound no guarding. Tx? discharge with oralqABx ( Not admitted)

....

!Child has abnormal movement and hematuria, had skin infectioni weeks ago. Now he looks drowsy,

Labs: Creatinine high, BUN high, C3 Low Urine: Protein +, RBC +++ == PSGN ( its come after skin or URTI by

1 to 3 week.)

↳most common intra abdominal malignancy in Peds

..... n
WAGR syndrome (cryptochridism), and mental retardation
↳WAGR Syndrome: wilms, Anirida, GU anomalies

!Child has abdominal mass, aniridia and undescended testicle =wagner disease ↳displaced, Never cross midline

↳mets to lungs (hemoptysis)


↳Hematuria

n
$URTI after 3 day develop hematuria = lgA nephropathy $a respiratory or gastrointestinal infection
during or immediately following(1-2 days)

Boy with glomerulonephritis ( hematuria ), after week he developed hemoptysis what is the dx ?
A. Henochschenolein Purpra. B. Good pasture syndrome✅✅. C. Rapid deterotion D. IgANephropathy
… Ddx: Wegner's disease (granulomatosis w/polyangitis)

r
Clinical features:
ENT involvement

!Child with glomerulonephritis then develop hemoptysis ? goodpasture syndrome Chronic rhinitis/sinusitis / saddle nose deformity

ne
Lower respiratory tract

pnpm
G AntiGBM
cough, dyspnea, hemoptysis

……
Renal involvement
Pauci-immune glomerulonephritis→ rapidly progressive
(crescentic) glomerulonephritis (RPGN)

- Child 6 years old present with asymptotic hematuria and every thing normal What u should to do
A)repeated urinanalysis ✅ ‫ﺎرت‬¹‫اﺻﺢ وﺣﺪة ﻣﻦ اﻻﺧﺘ‬. B)cystoscope. C) biopsy

!Asymptomatic hematuria in a child Urinanalysis : +ve RBC What to do next ?
A. Repeat urinalysis ✅B. Urine cytology C. Kidney biopsy D. Cystoscopy

!Child presents with hematuria. history of constipation for 1 week and presents with hematuria and
urinary incontinence. Upon examination he had tender abdomen, Labs: Low Hb, RBC in urine, Protein in
urine, WBC in urine == UTI (not PSGN)
...... this is a symptom
!Case of uti and asks what go with urinary tract infection as a risk factor: Fever

not a Riskfactor
a
! Most common cause of vomiting in pediatrics ? A. Celiac dis B. UTI C. DKA D. cyclic vomiting syndrome
.... ‫ ﯾﻛون ھو اﻻﺻﺢ‬GERD ‫…… ﻟو ﻓﯾﮫ ﺑﺎﻻﺧﺗﯾﺎرات‬.
!Most common couse of vomiting in infant? GERD , gastroenteritis

1Skin lesion
! Lower lid swelling, painful erythema not affect vision : A- Cellulitis B- Subperiosteal hematoma

Sole spots

!This child with same condition group of children in school .. (i suggest its chicken box)
A. sole spot B. skin scratch C. tonsillitis
..
!Pedia present with case of impetigo ask when can return to day care:
A. not contagious return now If I
Children can return to school 24 hours after beginning an effective

c
B. not contagious return after 3 days
antimicrobial therapy. Draining lesions should be kept covered.
C. contagious return after 5 days
D. contagious return when it gets dry.
‫ اﯾﺎم ﻣن اﻟﻌﻼج ﯾﺑدء‬٣ ‫ ﻣﻌدﯾﺔ وﯾﻌد‬imperiga ‫ ﺳﺎﻋﺔ ﺧﻼص ﻣﺎﯾﻛون‬٢٤ ‫ ﻣﺗﻰ ﻣﺎ ﯾﻛون ﻣﻌدي ؟ ﺑﻌد اﺧذ اﻟدواء ب‬impetiga ‫ﺑﺎﺧﺗﺻﺎر‬
.‫ﯾﺗﻌﺎﻓﻰ اﻟﻣرﯾض‬
…..
!Mom comes with her boy who had body rash for 2 days which was eruptions then became vesicular
then crusted with yellow secretions, asks when she can let him go to nursery? What you tell here?
A. The total is 5 days so she can let him go after 3 days B. she can let him go after 5 days from now
C. it is not contagious and she let him go any time
D. the vesicles secretions are contagious and she can let him go to nursery when they get dry
...
Boy has a cat developed itching for a month with red eye and watery with discharge no lymphadenopathy
and general exam normal ? A. Cat scratch. B. Dermatitis allergic✅ C. Conjunctivitis D. Seborrheic

!Child with mass on her face = hemangioma !
… Neonate —>Erythema toxicum neonatorum
e developed truncal rash= Reassure!
!Baby

-Oral thrush and napkin dermatitis what to give ? A-Oral 7 dys , B-oral and topical 5 dys , ✅ ✅. C-systimc
napkin dermatitis

!Neonate presents with erythematous rash all over the body, asking about the management ?
Reassure! ons Erythema toxicum neonatorum is a common rash in neonates. It appears in up to half
of newborns carried to term, usually between day 2–5 after birth; it does not occur

.... outside the neonatal period. Erythema toxicum is characterized by blotchy red spots
on the skin with overlying white or yellow papules or pustules.

!Child with poor oral intake reveals oral thrush and diaper rash, what to do :
5 days systemic antifungal
oral topical
..
!Child was brought to hospital with air way swelling and skin lesion all over the body the mother stated
that he was in a party at his friends house:=-Food allergy✅
….
!Kid with stridor and rash on birthday = allergy food !

! kid at party had (symptoms and sign of allergy) what cause? food allergy

! 6 - year - old boy went to sleep with friends home and returned with swelling around his eyes and feet ,
diagnosis? allergy!

!Child with vascular malformation of lower limb , when to interfere :
A. Pain?✅B. Claudication?
(Pain,pressure sores and ulcers, and possibly bleeding indicates rapid progression of AV malformation
…..
_neonate _ less than one year common area eczema==== * scalp *
....
_Child more than one year area and Adults common area eczema=== - *Flexors* -
....
!9 years old boy (long senior..not related to question) what is most part effect for eczema in the age
Goup? Flexor
‫ = ﻛرﺳم ﻓﻛس ﻟﻌﻼج اﻛزﯾﻣﯾﺎ‬flexer = ‫رﺑط = ﻓﯾﻛس‬
......
6 month old long senior..not related to question) what is most part effect for eczema in the age Goup?
A.Flexor B.extensor C.cheeks D. scalp ✅ ‫اﻟﻌﻣر ﯾﻔرق ﺑﺎﻟﻣﻛﺎن ﺳﻛﺎﻟب = ﺳﺗﺔ ﺷﮭور‬...
....... Infancy(first 6months): face, scalp,(6-12 months) extensor surfaces of extremities /Older, long-standing disease: flexural aspects
Eczema:
#9 yo with eczema + strong calmly hx of it , where is the commenters site for eczematous plaques ?
Flexors
......
!Eczema’s counseling regard bathing? non soap is better
...
Baby with eczema with a positive family history, what are tha common sites involved at this age? - scalp first 6months
….
Eczema in 10 y old boy where is most location: ‫ي‬
1) back head. 2) extensor. 3 ) fece and cheek. 4) dorm of foot. 5) Flexors✅✅
….
! Parent’s of 6 month child they come complaining that he has erythema and plaques all manifestation of

Be
eczema > what’s common area for eczema in this age : A. diaper area B. Upper back C. Flexor D. Dorsal of
foot
....

!child has eczema topical steroid i think now eczema get worse what next?
A. Medications name. B. Referral to derma. C. Parent Complince medication
....
!Most common place of psoriasis in childhood? A-Scalp✅ B-Diapers place C-Planter
(Elbow, knee, scalp)

Calculantion
T
800 with liver dis. And 800 normal doing test :Test +ve 200 with dis. And +ve 100 normal What is the
sensitivity of this test
A. 25%✅. B. 33%. C. 60%.
Sensitivity = TP/ TP+FN x100
200/800x100
%800 with liver dis. And 800 normal doing test :Test +ve 200 with dis. And +ve 100 normal = sensitve =
25% %
....
! Neonatal needs D10 fluids.. what’s the dose/kg? 2 ml/kg!
!child =5ml/kg for children!
%%Calculate the maintenance IVF for pediatric pt weight 18kg = Answer is : 1400/day , 56/hour === 4-2-
1 role roo so b
… ioxioo o.se IEEE
Eius g seminar
% baby for primigravida vomiting after each meal? - physiological GER%

Dehydrated patient sunken eye decrease skin turger and depressed fontanile percentage?
A)Mmild
5. B) 10✅ moderate. C) 15. D) 20 => Sever usually altered mental state and hypotension
… moderate
serge hypotension
!calculation hight in a boy ( 178 father hight + 155 mother hight ) = 173 !
… predicted height forchildren fafherheighttmotssrheightftbifboyllB.fm
...
Z
#maintenance for child 10kg /24 hour?
1000(10×100=100)
Ñ Ñ Ñ
‫ وﻧﺠﻤﻊ‬50 ‫"ب‬Ñ ‫ﺔ‬n‫ة اﻟﺜﺎﻧ‬¤ø‫ واﻟﻌ‬، ١٠٠ ÉÖ ‫ة‬¤ø‫ اول ﻋ‬٢٠ ‫ﻪ ﻣﺜﻼ‬n‫ ﻟﻮ ﻓ‬،‫ة‬¤ø‫ﺲ ﻋ‬ı ‫ﻓﻬﻨﺎ وزﻧﻬﺎ‬, 100 ÉÖ ‫ ـﻬﺎ‬i¸Ñ‫ اﻟﻮزن ﻧ‬ÉÖ ‫ة‬¤ø‫اول ﻋ‬
٢٠ ‫ ـﻬﺎ ب‬i¸Ñ‫ة ﻧ‬¤ø‫ ﺛﺎﻟﺚ ﻋ‬٣٠ ‫ وﻟﻮ ﻣﺜﻼ وزﻧﻪ‬، Ô Ñ z ‫ اﻟﻨﺎﺗﺠ‬.
100 first 10 kg , 50 scond 10 , 20 third 10
.....
#maintenance 18 kg per hour? 56 minutes
0 2 L
5
! Child with percentage of burn and asks about the duration of dehydration replacement (deficit should

X be replaced over?): A. 24 hrs B. 48 hrs C. 6 hrs D. 16 hrs



Fetal- bradykardia with sinusoidal what causes ?
shydropsfetariscrusensitiveadying fetus
a-Anemia b-Head compression c-Utroplacental Note: sinusoidal > anemua
!Fetal- bradykardia with sinusoidal what causes ?anemua!

! Young with vaginal itching and few vaginal bleed?A. foreign body B. sexual abuse
...
!Same but with fecal incontinence:
Sexual abuse
....
! I think 3m young pt with ear drum perforation.. what infection cause?
A. adenovirus B. rhinovirus C. rotavirus D. coronavirus

......
Case of chiled his weight is 10 Kg calculate the maintenance : 1000ml ( 10 × 100 = 1000)
! The screening for haematological diseases in children who are low risk , age in months = A. 6 B. 12 C.
X 24 D. 48

! Fluid replacement for Dehydrated child about 3 questions Maintenance
Answer is: First 10 kg > 10*100. Second 10 kg> 10*50. The rest * 20

# after slnusitis surgery affect sensation in lower eye and upper lip nerve ? Infra-orbital

1Diabetic inspidis
!Child after brain trauma developed polyuria . Investigations shows elevated serum osmolality and
decreased urine osmolality .A. Central DI
….
$Child had Head trauma that develop polyuria = Central DI

!Case about a child with increased water consumption and going to the toilet, not to mention Urine and
serum osmolality, not mention any electrolyte, with no response to water deprivation test, no other
complain:
A. central DI B. peripheral DI C. SIADH
D. medical terminology sound like drinking too much water.!

... E Compulsive water drinking


or psychogenic polydipsia
‫ ﻣﺎ اﻗﺪر اﺷﺨﺺ‬DI ‫ |ﺪون ﻣﺎ اﻋﺮف‬Osmolorty

!child after RTA , admitted in ICU , labs show High serum osmo and low urine osmo Dx ?
A. Central DI ✅ B. Nephrogenic DI C. SIADH

! Case about a child with increased water consumption and going to the toilet, didn’t mention Urine and
serum osmolality, with no response to water deprivation test, no other complain:
A. central DI B. peripheral DI C. SIADH Compulsive water drinking
... or psychogenic polydipsia

!Child came to emergency dehydrated, history of increased thirst and wets diapers a lot after recent
seizure attack, Urine osmolality low. Blood osmolality high. Whats management?
Desmopressin ✅✅!(Diapetis insipids)
vasopressin
.. .Tx
. . .for
. central MADH or
!11 years old boy , wetting his bed. All discussion and rewards done. What to do ? Desmopressin
…...
! 7 years old boy , mother concern because he still wet his bed despite he is fully toilet trained.. he is a
shamed by that and ask for help (the boy ), what is the best management :
A. Avoid banishment B. Desmopressin C. Bed Alarm and positive reinforcement D. Family centered
approach

$8-7 years old child with bedwetting came asking for help with his mom:
A. don’t punish him B. positive re enforcement and alarm✅ C. intranasal desmopressin
........
!Bedwetting is considered normal until age: A. 5 B. 6 C. 7 D. 8
....
! Mother came with her son, and concern about recurrent bed wetting, the treatment effect will be on
which muscle: A. Gluteus B. Perianal C. Pelvic Floor
1Ethic
! BA child and her father smoking, your action? -Education to the father

!Mom is worried bout her son who has lost his friend 3 months ago and he is talking about killing
himself but he will not actually do it action ?
A-do nothing normal grief B-take detailed hx to prevent suicide ✅
..
! 2 Q about child with separation anxiety and school phobia
Q1 A. Focus on unlikely bad things could happen (i
B. punish him when react with sym of anxiety
C. stay at home 3 months and don't go to school
D. wait for supe
….
! Which of the following most Induced Fabricated child illness? Mother

# 7 years old child complains of pain that last for 15 minutes and then resolved. It is always in the
morning (school time) what it the advice for the mother.
positive enforcement by forcing on the things the child does rightat school✅

‫ﺎرات‬L‫ اﻻﺧﺘ‬bd c îï c i
d ‫ﺸﺎرك وﻻ‬M‫ اﻟ”ﻼس ﻣﺎ‬bd ‫ و‬، ‫ﺖ‬Ë‫ﻄﻪ ﺻﺎر ﻣﺸ‬L‫ە ¯ﺴ‬í‫ﻔ‬- ‫ﻌﺪﻫﺎ‬- ‫ﻪ‬L‫ﻠﺪە ﺛﺎﻧ‬- ¤‫ﺸﻪ واﻧﺘﻘﻞ ا‬K‫ﺎن اﻟﻤﻌ‬ù‫ ﻣ‬í™‫ﻃﻔﻞ ﻏ‬: ‫ﺆال‬c ‫ ﺳ‬bd
c
bd ‫»ﺎن‬
A. depression B. Hypomania
C. Dysthymia D. 4th option Adjustment disorder
......

Girl 8 yrs old with bald area on head. Mother says she keeps picking her hair due to exams and stress.
(trichotillomania) What medication will u give? a. Olanzapine b. Fluoxetine✅
(No CBT or psychotherapy in options)

!2 mo infant is was found dead in his bed , was born with no complications, no medical illness ,mother
denies any family hx of sudden death , what would you ask more about?
A-Social hx✅ B-Family hx C-Mother meds during pregnancy

!Radiologist suggest something for pt case.. who should take the pt consent for radiologist opinion?
A. radiologist B. Head nurse C. Medical resident D. Any one of treatment team
....
!70 yo man, present with skin change, report revealed malignant melanoma. Who can make the pt
comply to your medical advice:
A. this is a serious issue. B. you are professional so he trusted you C. explain to the pt by medical term.
D. take time to tell him and try to gain his trust.
...
! Pt came with s&s of violence what do:
A. admit call social worker. B. treat and find the cause of her sadness. C. focus Hx and PE.
...
!When you should consider during examination of other gender:
A. ask for nurse B. expose necessary area. C. be professional
...
!40 yo female, pregnant at 20wks gestation, complain of fever and lower abd pain, she didn’t seek for
medical advice, what is the reason: A. neglect B. fear of physician. C. cost of consult
...
!Elderly patient with Cancer, their family ask the doctor to increase the dose of analgesia, Doctor respond
after clarify to them that may cause patient Death .. principle of what:
A. Totality B. subsidity C double effect

! doctor explaining to patient and ask does everything make sense ?
Check understanding of pt

$A scenario of a 7 year old girl child brought on account of passage of blood stained stools, bedwetting
after attaining prior continence and complaining of vulval itch. Perineal examination revealed some labial
bruises. What's mosst likely diagnosis?
A. Candidal infection of vulva. B. Bacterial vaginosis. C. Pinworm infection. D. Abuse✅
… ''''
Psychiatrist asking a patient with depression about the possibility of him having a mental illness, what is he
assessing? insight
...
$The scenario talks about how the child always complain about abdominal pain
everytime he goes to school and what to inform the parents/teacher on how to
deal with this school phobia by positive reinforcement?
A. Let him have more TV watch time
B. campliment him on the good work he has done✅ ‫ﺪ‬¹‫ﻌﻤﻞ ﺟ‬- ‫ﻪ اﻟﻤﺪرس اﻧﻪ ﻗﺎم‬¹‫ ﻋﻠ‬Ñd c ‫§ﺠﻠﺲ ﻳﺜ‬
c. give him less or moderate work
D. Mark the absence from school days
........
!child his parent brought to the ER with 2nd degree burns after accidentally jump to hot water and splash
it, O/E u notice other healed injuries of defiant times. What is the step-by-step guide to Child Protection?
A.Observe, document, report, self care✅
B. Observe, report, referral, follow up.
C. Document, confirm, report, consoling parents.
D. Observe, report, referral, follow up.
‫ﺎب اﻟﺤﻤﺎ§ﺔ‬ú‫ﺒﻮرت ﺛﻢ ﻧﺎﺧﺬ ب اﺳ‬Ò‫ﻪ ر‬¹‫ﻨﺎە وﻧﻌﻤﻞ ﻋﻠ‬-‫ ﻛﺘ‬¤d‫ ﺛﻢ ﻧﺮﺳﻞ ا‬، ‫ ﻻﺣﻈﻨﺎە و*ﺴﺠﻠﻪ‬¤d‫ ﺛﻢ ﻋﺸﺎن ﻣﺎ ﻧˆ˝ ﻧﻜﺘﺐ ا‬، ‫ء ﻧﻼﺧﻈﻪ‬î ï
d ‫اول‬
‫ﺔ ﻋﺸﺎن ﻣﺎ§ﻘﻊ ﺑﻨﻔﺲ اﻟﺨﻄﺎ‬-‫ ﻣﻦ اﻻﺻﺎ‬.
......
,,,,,
Female child with vaginal discharge and fecal incontinence most likely diagnosis:
A-Sexual abuse ✅ B- FB

Pregnant with indication of CS she refused? A- Husband consent ✅ ‫ اﻟﺰوج اﻫﻢ‬Ùd N ‫ ﻫﺬا ر‬Ñi‫ﺣ‬
......
Case of amputation, pt refused. What to do?
A- refer. B- discharge because he refused ✅ No respect wish in choices
.....
Where 12 year old must set in car to avoid injury in accidents?
A-in the front unrestrained ‫ ﻻ §ﺠﻠﺲ وراء‬B-in the back restrained✅✅‫ﺪ‬¹‫ ﻣﻘ‬C-in child chair in the
back‫ اﻻﻃﻔﺎل ﻻ‬î d ‫ﻛﺮ‬
….
! What are the 4 principles for effective reporting in child abuse?
A-observe, document, report, self protection✅ B-observe, document, referrals, follow up
C- Document, confirm, report, conference parents.
….
7 y Child with vomiting dx as school phobia and GAD ( Generalize anxity disorder) tx? Cognitive
behavioral therapy

Female child with vaginal discharge and fecal incontinence most likely diagnosis:
Sexual abuse.
.... ..... .....
% Female child with sexual ababuse.. site of injury? At 6 o’clock%

%A mother comes to the clinic she is concerned about her son for the past 3 months after the death of his
friend. He one time said to her “i wish i was dead, but I won't do anything about it”. What to do?Ask
detailed Q about suicidality for prevention.%
….
...........
%Child Head trauma subarachnoid, hge, periorbital edema, bruises and LOC Father said he found her like
this, what to do? Call child protection %%
........
Wife and her husband in the clinic for some test, the doctor finds out that the husband is HIV positive who
should he inform:
A-The wife B-The husband ✅✅ C-ignore the result D-The ethics committee

......
Most sexual harassment‫ ﺗﺤﺮش‬for girl in work place ?
A. Coworker
B. supervisor ✅
C. Clients
D. Owner or boss
%%Most sexual harassment‫ ﺗﺤﺮش‬for girl in work place = supervisor%%
.....
Women wearing revealed clothes talking to you in quiet tone with unknown language the sitting nurses
didn’t understand her language?
A-find nurse she understands her language ✅
B- call for security
.....
!postive rinforcement = focous on the right things he is doing in the class !
‫ﺔ وﻧﻌﺰزە ؟‬Ò&‫ اﻟﻄﻔﻞ ﻗ‬äd‫ﻒ ﻧﺨ‬¹‫ ﻛ‬Ñd c ‫§ﻌ‬
....
$Child with symptoms of ADHD( Attention Deficit Hyperactivity Disorder ) interrupting the class room
and neighbor what is the treatment= dexmethylphenidateMethylphenidate
**if the Child 6 years and older is methylphenidate
** If less than 6 years CBT
.....
Mif byearse
!attention deficit hyperactivity disorder treat by = Methylphenidate !

!Young child with bruises in genital area , bleeding = abuse !



! Most Child fabrication symptoms by A. Mother B. Father C. Step mother D. Care provider

!Child with poor weight gain mother is anxious and busy with the other children in poor socioeconomic
status: Refer for social worker

!Child complaining of abdominal pain on school days to avoid going to school, what to do? Focus on
good things he did ✅
….
..
!Patient child with signs and symptoms in (clinical finding ) neurofibromas type1 “Did NOT mention the
diagnosis” and mother side a lot his family with same signs:
A. Family Consultation. B. Referral to genetic test. C. start counseling
….
!What is the problem of a 12 old child response to her parent's illness according to her developmental
stage:
A. difficulty to express emotions
B. Refractory behaviour: fussing, violences
C. Imitating her parents symptoms when she plays
D. Does not understand their illnesses nature, cause, diagnosis, treatment
….
!Girl 8 yrs old with bald area on head. Mother says she keeps picking her hair due to exams and stress.
(trichotillomania) What medication will u give? A. Olanzapine B. Fluoxetine C. (No CBT or psychotherapy)

!First sign of female puberty A. menarche B. Thelarche
… SSRI first line
! first sign of male puberty?scrotum enlargement✅

TABLE 4-1. Contraindications and precautions(a) to commonly used vaccines used vaccines
Vaccine Citation Contraindications Precautions
DT, Td (4) Severe allergic reaction (e.g., anaphylaxis) after a previous dose or GBS <6 weeks after previous dose of tetanus-toxoid–containing
to a vaccine component vaccine
History of Arthus-type hypersensitivity reactions after a previous
dose of diphtheria-toxoid—containing or tetanus-toxoid–
containing vaccine; defer vaccination until at least 10 years have
elapsed since the last tetanus-toxoid-containing vaccine
Moderate or severe acute illness with or without fever
DTaP (38) Severe allergic reaction (e.g., anaphylaxis) after a previous dose or Progressive neurologic disorder, including infantile spasms,
to a vaccine component uncontrolled epilepsy, progressive encephalopathy; defer DTaP
Encephalopathy (e.g., coma, decreased level of until neurologic status clarified and stabilized
consciousness, prolonged seizures), not attributable to another GBS <6 weeks after previous dose of tetanus-toxoid–containing
identifiable cause, within 7 days of administration of previous dose vaccine
of DTP or DTaP History of Arthus-type hypersensitivity reactions after a previous
dose of diphtheria-toxoid–containing or tetanus-toxoid–containing
vaccine; defer vaccination until at least 10 years have elapsed
since the last tetanus-toxoid–containing vaccine
Moderate or severe acute illness with or without fever
Hepatitis A (39) Severe allergic reaction (e.g., anaphylaxis) after a previous dose or Moderate or severe acute illness with or without fever
to a vaccine component
Hepatitis B (40) Severe allergic reaction (e.g., anaphylaxis) after a previous dose or Moderate or severe acute illness with or without fever
to a vaccine component
Hypersensitivity to yeast
Hib (41) Severe allergic reaction (e.g., anaphylaxis) after a previous dose or Moderate or severe acute illness with or without fever
to a vaccine component
Age <6 weeks
HPV(b) (42) Severe allergic reaction (e.g., anaphylaxis) after a previous dose or Moderate or severe acute illness with or without fever
to a vaccine component, including yeast
IIV (43) Severe allergic reaction (e.g., anaphylaxis) after previous dose of GBS <6 weeks after a previous dose of influenza vaccine
influenza vaccine or to vaccine component. Moderate or severe acute illness with or without fever
Egg allergy other than hives, e.g., angioedema, respiratory
distress, lightheadedness, recurrent emesis; or required
epinephrine or another emergency medical intervention (IIV may be
administered in an inpatient or outpatient medical setting and
under the supervision of a health care provider who is able to
recognize and manage severe allergic conditions).
IPV (44) Severe allergic reaction (e.g., anaphylaxis) after a previous dose or Pregnancy
to a vaccine component Moderate or severe acute illness with or without fever
LAIV(c) (43) Severe allergic reaction (e.g., anaphylaxis) after a previous dose or GBS <6 weeks after a previous dose of influenza vaccine
to a vaccine component Asthma in persons aged 5 years old or older
Concomitant use of aspirin or aspirin-containing medication in Medical conditions which might predispose to higher risk of
children and adolescents complications attributable to influenza(d)
LAIV4 should not be administered to persons who have taken Moderate or severe acute illness with or without fever
oseltamivir or zanamivir within the previous 48 hours, peramivir
within the previous 5 days, or baloxavir within the previous 17 days.
Pregnancy
Children aged 2 through 4 years who have received a diagnosis of
asthma or whose parents or caregivers report that a health care
provider has told them during the preceding 12 months that their
child had wheezing or asthma or whose medical record indicates a
wheezing episode has occurred during the preceding 12 months.
Persons with active cerebrospinal fluid/oropharyngeal
communications/leaks.
Close contacts and caregivers of severely immunosuppressed
persons who require a protected environment.
Persons with cochlear implants (due to the potential for CSF leak,
which might exist for some period of time after implantation.
Providers might consider consultation with a specialist concerning
risk of persistent CSF leak if an age-appropriate inactivated or
recombinant vaccine cannot be used).
Altered Immunocompetence
Anatomic or functional asplenia (e.g. sickle cell disease)
MenACWY (45) Severe allergic reaction (e.g., anaphylaxis) after a previous dose or Moderate or severe acute illness with or without fever
to a vaccine component, including yeast
MenB (46, 47) Severe allergic reaction (e.g., anaphylaxis) after a previous dose or Moderate or severe acute illness with or without fever
to a vaccine component Pregnancy
MMR(e),(f) (1) Severe allergic reaction (e.g., anaphylaxis) after a previous dose or Recent (≤11 months) receipt of antibody-containing blood product
to a vaccine component (specific interval depends on product)
Pregnancy History of thrombocytopenia or thrombocytopenic purpuraNeed
for tuberculin skin testing or interferon-gamma release assay
(IGRA) testing(i)
Known severe immunodeficiency (e.g., from hematologic and solid
tumors, receipt of chemotherapy, congenital immunodeficiency,
long-term immunosuppressive therapy(g) or patients with HIV Moderate or severe acute illness with or without fever
infection who are severely immunocompromised)

Family history of altered immunocompetence(h)


MPSV4 (48) Severe allergic reaction (e.g., anaphylaxis) after a previous dose or Moderate or severe acute illness with or without fever
to a vaccine component
PCV13 (49) Severe allergic reaction (e.g., anaphylaxis) after a previous dose of Moderate or severe acute illness with or without fever
PCV13 or any diphtheria-toxoid–containing vaccine or to a
component of a vaccine (PCV13 or any diphtheria-toxoid–
containing vaccine), including yeast
PPSV23 (50) Severe allergic reaction (e.g., anaphylaxis) after a previous dose or Moderate or severe acute illness with or without fever
to a vaccine component
RIV (43) Severe allergic reaction (e.g., anaphylaxis) to any component of the GBS <6 weeks after a previous dose of influenza vaccine
vaccine Moderate or severe acute illness with or without fever
Rotavirus (6) Severe allergic reaction (e.g., anaphylaxis) after a previous dose or Altered immunocompetence other than SCID
to a vaccine component Chronic gastrointestinal disease(j)
SCID
Spina bifida or bladder exstrophy(j)
History of intussusception
Moderate or severe acute illness with or without fever
Tdap (51) Severe allergic reaction (e.g., anaphylaxis) after a previous dose or GBS <6 weeks after a previous dose of tetanus-toxoid–containing
to a vaccine component vaccine
Encephalopathy (e.g., coma, decreased level of Progressive or unstable neurological disorder, uncontrolled
consciousness, prolonged seizures), not attributable to another seizures, or progressive encephalopathy until a treatment regimen
identifiable cause, within 7 days of administration of previous dose has been established and the condition has stabilized
of DTP, DTaP, or Tdap
History of Arthus-type hypersensitivity reactions after a previous
dose of diphtheria-toxoid—containing or tetanus-toxoid–
containing vaccine; defer vaccination until at least 10 years have
elapsed since the last tetanus-toxoid–containing vaccine

Moderate or severe acute illness with or without fever


Varicella(e),(f) (52) Severe allergic reaction (e.g., anaphylaxis) after a previous dose or Recent (≤11 months) receipt of antibody-containing blood product
to a vaccine component (specific interval depends on product)
Known severe immunodeficiency (e.g., from hematologic and solid Moderate or severe acute illness with or without fever
tumors, receipt of chemotherapy, congenital Receipt of specific antiviral drugs (acyclovir, famciclovir, or
immunodeficiency, long-term immunosuppressive therapy(g) or valacyclovir) 24 hours before vaccination (avoid use of these
patients with HIV infection who are severely immunocompromised)(e) antiviral drugs for 14 days after vaccination)
Use of aspirin or aspirin-containing products(k)
Pregnancy

Family history of altered immunocompetence(h)


Zoster (53) Severe allergic reaction (e.g., anaphylaxis) after a previous dose or Moderate or severe acute illness with or without fever
to a vaccine component Receipt of specific antiviral drugs (acyclovir, famciclovir, or
(Live zoster vaccine only) Known severe immunodeficiency (e.g., valacyclovir) 24 hours before vaccination (avoid use of these
from hematologic and solid tumors, receipt of chemotherapy, antiviral drugs for 14 days after vaccination, for zoster vaccine live
congenital immunodeficiency, long-term immunosuppressive only)
therapy(g) or patients with HIV infection who are severely
immunocompromised)(e)

Pregnancy
Abbreviations: DT = diphtheria and tetanus toxoids; DTaP = diphtheria and tetanus toxoids
and acellular pertussis; DTP = diphtheria toxoid, tetanus toxoid, and pertussis; GBS = Guillain-
Barré syndrome; Hib = Haemophilus influenzae type b; HIV = human immunodeficiency virus;
HPV = human papillomavirus; IIV = inactivated influenza vaccine; IPV = inactivated poliovirus;
LAIV = live, attenuated influenza vaccine; MenACWY = quadrivalent meningococcal conjugate
vaccine; MMR = measles, mumps, and rubella; MPSV4 = quadrivalent meningococcal
polysaccharide vaccine; PCV13 = pneumococcal conjugate vaccine; PPSV23= pneumococcal
polysaccharide vaccine; SCID = severe combined immunodeficiency; RIV=recombinant
influenza vaccine; Td = tetanus and diphtheria toxoids; Tdap = tetanus toxoid, reduced
diphtheria toxoid, and acellular pertussis.

You might also like